Slot System
Featured Buckets
Featured Buckets Admin

Should you screen for postpartum depression?

Article Type
Changed
Tue, 07/14/2020 - 15:31
Display Headline
Should you screen for postpartum depression?
Practice changer

Make depression screening a routine part of the postpartum visit.1

Strength of recommendation

B: Based on a single blinded randomized controlled trial (RCT).

Yawn BP, Dietrich AJ, Wollan P, et al. TRIPPD practices. TRIPPD: a practice-based network effectiveness study of postpartum depression screening and management. Ann Fam Med. 2012;10:320-329.

 

Illustrative case

A 20-year-old patient comes in at 6 weeks’ postpartum for routine care. Should you screen her for postpartum depression?

The incidence of depression in the first 3 months’ postpartum is estimated at about 14%,2,3 and the consequences can be severe. A new mom with a mood disorder in the first year of her child’s life can disrupt the mother-infant relationship, thereby contributing to both short- and long-term adverse outcomes for the child. These include behavior problems, low self-esteem, poor self-regulation, and an increased risk of impaired mental and motor development.4,5

Postpartum depression often goes undetected

Despite this correlation, postpartum depression is both under-recognized and undertreated.6 A prospective randomized study of 5169 women who were screened for postpartum depression bears this out. Researchers found that about one in 4 (26%) of the 674 mothers who had positive screens were not asked about their emotional state by their clinicians.2

This may be due to a lack of evidence of the efficacy of screening for postpartum depression. In the Healthy Start Depression Initiative (n=1336), universal screening—with referrals to mental health care outside of the primary care physicians’ offices—did not alter either the level of depressive symptoms over a 10-month period or depression treatment.7

The American College of Obstetricians and Gynecologists’ Committee on Obstetric Practice noted in 2010 (and reaffirmed in 2012) that there is insufficient evidence to support a firm recommendation for universal postpartum screening for depression.8 Neither the US Preventive Services Task Force nor the American Academy of Family Physicians has a specific recommendation regarding postpartum depression.

STUDY SUMMARY

Screening—with follow-up—leads to better outcomes

The study by Yawn et al included 28 family medicine practices in 21 states, randomized to either usual care or intervention.1 Eligible practices had to have provided well-baby or maternity care to more than 30 patients in the previous year, but not to routinely screen for postpartum depression. A total of 2343 women aged 18 years or older—all of whom were between 5 and 12 weeks’ postpartum and planning on continuing care at the family medicine practice at the time they entered the study—were enrolled.

Staff at the 14 intervention sites received training in postpartum depression screening and diagnosis. They were also given a set of tools to facilitate management of postpartum depression, including an outline for follow-up visits and scripts for nurses to use for telephone calls relating to antidepressants. Therapy options were left to the discretion of the physician and the patient, with the help of tools that detailed the doses and adverse effects of various medications and described cognitive behavioral therapy.

Low socioeconomic status has been shown to be a strong risk factor for the development of postpartum depression.At the start of the study, participants at all 28 sites were given survey packets containing the Edinburgh Postnatal Depression Scale (EPDS) and the 9-item Patient Health Questionnaire (PHQ-9). Patients subsequently received surveys by mail at 6 and 12 months’ postpartum for self-reported outcomes. Clinicians at the intervention sites had routine access to the EPDS and PHQ-9 scores; those at the usual care sites did not.

The primary outcome was ≥5 point drop in the PHQ-9 score from baseline at 6 or 12 months’ postpartum, considered to be an indicator of clinical improvement and/or response to therapy. The PHQ-9 is a validated measure of depression severity, with the proven ability to detect changes over time.9,10

Of the 2343 women initially enrolled in the study, 1897 (81%) provided outcome information and were included in the analysis. The rates of women with elevated depression scores (EPDS and/or PHQ-9 ≥10) at the start were comparable between the intervention and usual care groups (29.5% vs 25.8%, respectively).

Of those whose scores were initially elevated, 219 women in the intervention group and 178 women in the usual care group returned surveys at 12 months and were included in the final analysis. The results: 45% of those in the intervention group met the primary outcome—a decline in self-reported depressive symptom levels, as indicated by a PHQ-9 decrease ≥5 points—compared with 35% of the women in the usual care group (odds ratio, 1.8; 95% confidence interval, 1.14-2.9; P=.001). Not surprisingly, medical record review also indicated that those in the intervention group who initially had elevated depression scores were more likely to have received a diagnosis (66% vs 41%; P=.0006) and therapy (20% vs 11%; P=.02) for postpartum depression.

 

 

 

WHAT'S NEW?

We now have evidence of the efficacy of postpartum screening

This is the first large study of a primary care-based approach to screening, diagnosis, and management of postpartum depression to show any improvement in maternal outcomes at 12 months. Prior universal screening and referral support in the Healthy Start program was done by paraprofessionals, who referred women with positive screens for mental health care outside of the primary care setting and did not reduce the rate of depression in perinatal women.7

CAVEATS

Dropout rate, socioeconomic status may affect results

Among the women who initially were found to be positive for postpartum depression, 38% did not return questionnaires at 12 months’ postpartum. While this loss to follow-up is high, it is comparable to that of most effectiveness trials11 with similar rates in the intervention and usual care groups.

Within the intervention group, there was no statistical difference between women who did and did not return the questionnaires with regard to marital status, history of depression, income, or uninsured status. However, women in the usual care group who did not return the 12-month questionnaire were more likely to be poor (89% vs 57%; P<.01) and uninsured (49% vs 29%; P<.01) than those who did return the questionnaire.

The impact of these differences and the loss to follow-up in this study is unknown. However, low socioeconomic status has been shown to be a strong risk factor for the development of postpartum depression.12 The authors of the study suggest (and we agree) that the difference in socioeconomic status in women who did not return the questionnaire may underestimate the positive impact of this screening approach.

CHALLENGES TO IMPLEMENTATION

Screening requires extra work

Personnel at the intervention sites received a half day of training in postpartum depression screening, diagnosis, and nursing telephone follow-up. The workload at these sites also increased, as most women found to have postpartum depression received one to 2 follow-up telephone calls and an average of one to 2 follow-up visits after the start of therapy. These measures, while seemingly modest, could pose a challenge to implementation. This could potentially be alleviated by the additional payments for care coordination promised in the Patient Protection and Affordable Care Act.13

ACKNOWLEDGEMENT

The PURLs Surveillance System was developed in part by Grant Number UL1RR024999 from the National Center for Research Resources, a Clinical Translational Science Award to The University of Chicago. The content is solely the responsibility of the authors and does not necessarily represent the official views of the National Center of Research Resources or the National Institutes of Health.

Files
References

1. Yawn BP, Dietrich AJ, Wollan P, et al; TRIPPD practices. TRIPPD: a practice-based network effectiveness study of postpartum depression screening and management. Ann Fam Med. 2012;10:320-329.

2. Horowitz JA, Murphy CA, Gregory KE, et al. Best practices: community-based postpartum depression screening: results from the CARE study. Psychiatr Serv. 2009;60:1432-1434.

3. Gaynes BN, Gavin N, Meltzer-Brody S, et al. Perinatal Depression: Prevalence, Screening Accuracy, and Screening Outcomes: Summary. AHRQ Evidence Report Summaries. Rockville, MD: Agency for Healthcare Research and Quality; Feb, 2005.

4. Murray L, Cooper PJ. The impact of postpartum depression on child development. In: Goodyer I, ed. Aetiological Mechanisms in Developmental Psychopathology. Oxford, England: Oxford University Press; 2003.

5. Goodman SH, Gotlib IH. Risk for psychopathology in the children of depressed mothers: a developmental model for understanding mechanisms of transmission. Psychol Rev. 1999;106:458-490.

6. Pearlstein T, Howard M, Salisbury A, et al. Postpartum depression. Am J Obstet Gynecol. 2009;200:357-364.

7. Yonkers KA, Smith MV, Lin H, et al. Depression screening of perinatal women: an evaluation of the healthy start depression initiative. Psychiatr Serv. 2009;60:322-328.

8. American College of Obstetricians and Gynecologists. Committee on Obstetric Practice. Committee opinion no. 453: Screening for depression during and after pregnancy. Obstet Gynecol. 2010;115(2 pt 1):394-395.

9. Löwe B, Kroenke K, Herzog W, et al. Measuring depression outcome with a brief self-report instrument: sensitivity to change of the Patient Health Questionnaire (PHQ-9). J Affect Disord. 2004;81:61-66.

10. Kroenke K, Spitzer RL, Williams JB. The PHQ-9: validity of a brief depression severity measure. J Gen Intern Med. 2001;16:606–613.

11. Ribisl KM, Walton MA, Mowbray CT, et al. Minimizing participant attrition in panel studies through the use of effective retention and tracking strategies: review and recommendations. Eval Program Planning. 1996;19:1-25.

12. Dolbier CL, Rush TE, Sahadeo LS, et al; Community Health Network Investigators. Relationships of race and socioeconomic status to postpartum depressive symptoms in rural african american and non-Hispanic white women. Matern Child Health J. 2013;17:1277-1287.

13. Centers for Medicare and Medicaid Services. The Affordable Care Act: Helping Providers Help Patients. Available at: http://www.cms.gov/Medicare/Medicare-Fee-for-Service-Payment/ACO/downloads/ACO-Menu-Of-Options.pdf. Accessed September 6, 2013.

Article PDF
Author and Disclosure Information

Andrew H. Slattengren, DO
Shailendra Prasad, MBBS, MPH
M. Altaf Kaiseruddin, MD

North Memorial Family Medicine Residency, University of Minnesota, Minneapolis (Drs. Slattengren and Prasad); Department of Family Medicine, The University of Chicago (Dr. Kaiseruddin)

PURLs EDITOR
Bernard Ewigman, MD, MSPH
Department of Family Medicine, The University of Chicago

Issue
The Journal of Family Practice - 62(11)
Publications
Topics
Page Number
E1-E3
Legacy Keywords
Andrew H. Slattengren; DO; Shailendra Prasad; MBBS; MPH; M. Altaf Kaiseruddin; MD; postpartum; depression; screening; postpartumdepression; PURLs; Healthy Start Depression Initiative; Patient Protection and Affordable Care Act
Sections
Files
Files
Author and Disclosure Information

Andrew H. Slattengren, DO
Shailendra Prasad, MBBS, MPH
M. Altaf Kaiseruddin, MD

North Memorial Family Medicine Residency, University of Minnesota, Minneapolis (Drs. Slattengren and Prasad); Department of Family Medicine, The University of Chicago (Dr. Kaiseruddin)

PURLs EDITOR
Bernard Ewigman, MD, MSPH
Department of Family Medicine, The University of Chicago

Author and Disclosure Information

Andrew H. Slattengren, DO
Shailendra Prasad, MBBS, MPH
M. Altaf Kaiseruddin, MD

North Memorial Family Medicine Residency, University of Minnesota, Minneapolis (Drs. Slattengren and Prasad); Department of Family Medicine, The University of Chicago (Dr. Kaiseruddin)

PURLs EDITOR
Bernard Ewigman, MD, MSPH
Department of Family Medicine, The University of Chicago

Article PDF
Article PDF
Practice changer

Make depression screening a routine part of the postpartum visit.1

Strength of recommendation

B: Based on a single blinded randomized controlled trial (RCT).

Yawn BP, Dietrich AJ, Wollan P, et al. TRIPPD practices. TRIPPD: a practice-based network effectiveness study of postpartum depression screening and management. Ann Fam Med. 2012;10:320-329.

 

Illustrative case

A 20-year-old patient comes in at 6 weeks’ postpartum for routine care. Should you screen her for postpartum depression?

The incidence of depression in the first 3 months’ postpartum is estimated at about 14%,2,3 and the consequences can be severe. A new mom with a mood disorder in the first year of her child’s life can disrupt the mother-infant relationship, thereby contributing to both short- and long-term adverse outcomes for the child. These include behavior problems, low self-esteem, poor self-regulation, and an increased risk of impaired mental and motor development.4,5

Postpartum depression often goes undetected

Despite this correlation, postpartum depression is both under-recognized and undertreated.6 A prospective randomized study of 5169 women who were screened for postpartum depression bears this out. Researchers found that about one in 4 (26%) of the 674 mothers who had positive screens were not asked about their emotional state by their clinicians.2

This may be due to a lack of evidence of the efficacy of screening for postpartum depression. In the Healthy Start Depression Initiative (n=1336), universal screening—with referrals to mental health care outside of the primary care physicians’ offices—did not alter either the level of depressive symptoms over a 10-month period or depression treatment.7

The American College of Obstetricians and Gynecologists’ Committee on Obstetric Practice noted in 2010 (and reaffirmed in 2012) that there is insufficient evidence to support a firm recommendation for universal postpartum screening for depression.8 Neither the US Preventive Services Task Force nor the American Academy of Family Physicians has a specific recommendation regarding postpartum depression.

STUDY SUMMARY

Screening—with follow-up—leads to better outcomes

The study by Yawn et al included 28 family medicine practices in 21 states, randomized to either usual care or intervention.1 Eligible practices had to have provided well-baby or maternity care to more than 30 patients in the previous year, but not to routinely screen for postpartum depression. A total of 2343 women aged 18 years or older—all of whom were between 5 and 12 weeks’ postpartum and planning on continuing care at the family medicine practice at the time they entered the study—were enrolled.

Staff at the 14 intervention sites received training in postpartum depression screening and diagnosis. They were also given a set of tools to facilitate management of postpartum depression, including an outline for follow-up visits and scripts for nurses to use for telephone calls relating to antidepressants. Therapy options were left to the discretion of the physician and the patient, with the help of tools that detailed the doses and adverse effects of various medications and described cognitive behavioral therapy.

Low socioeconomic status has been shown to be a strong risk factor for the development of postpartum depression.At the start of the study, participants at all 28 sites were given survey packets containing the Edinburgh Postnatal Depression Scale (EPDS) and the 9-item Patient Health Questionnaire (PHQ-9). Patients subsequently received surveys by mail at 6 and 12 months’ postpartum for self-reported outcomes. Clinicians at the intervention sites had routine access to the EPDS and PHQ-9 scores; those at the usual care sites did not.

The primary outcome was ≥5 point drop in the PHQ-9 score from baseline at 6 or 12 months’ postpartum, considered to be an indicator of clinical improvement and/or response to therapy. The PHQ-9 is a validated measure of depression severity, with the proven ability to detect changes over time.9,10

Of the 2343 women initially enrolled in the study, 1897 (81%) provided outcome information and were included in the analysis. The rates of women with elevated depression scores (EPDS and/or PHQ-9 ≥10) at the start were comparable between the intervention and usual care groups (29.5% vs 25.8%, respectively).

Of those whose scores were initially elevated, 219 women in the intervention group and 178 women in the usual care group returned surveys at 12 months and were included in the final analysis. The results: 45% of those in the intervention group met the primary outcome—a decline in self-reported depressive symptom levels, as indicated by a PHQ-9 decrease ≥5 points—compared with 35% of the women in the usual care group (odds ratio, 1.8; 95% confidence interval, 1.14-2.9; P=.001). Not surprisingly, medical record review also indicated that those in the intervention group who initially had elevated depression scores were more likely to have received a diagnosis (66% vs 41%; P=.0006) and therapy (20% vs 11%; P=.02) for postpartum depression.

 

 

 

WHAT'S NEW?

We now have evidence of the efficacy of postpartum screening

This is the first large study of a primary care-based approach to screening, diagnosis, and management of postpartum depression to show any improvement in maternal outcomes at 12 months. Prior universal screening and referral support in the Healthy Start program was done by paraprofessionals, who referred women with positive screens for mental health care outside of the primary care setting and did not reduce the rate of depression in perinatal women.7

CAVEATS

Dropout rate, socioeconomic status may affect results

Among the women who initially were found to be positive for postpartum depression, 38% did not return questionnaires at 12 months’ postpartum. While this loss to follow-up is high, it is comparable to that of most effectiveness trials11 with similar rates in the intervention and usual care groups.

Within the intervention group, there was no statistical difference between women who did and did not return the questionnaires with regard to marital status, history of depression, income, or uninsured status. However, women in the usual care group who did not return the 12-month questionnaire were more likely to be poor (89% vs 57%; P<.01) and uninsured (49% vs 29%; P<.01) than those who did return the questionnaire.

The impact of these differences and the loss to follow-up in this study is unknown. However, low socioeconomic status has been shown to be a strong risk factor for the development of postpartum depression.12 The authors of the study suggest (and we agree) that the difference in socioeconomic status in women who did not return the questionnaire may underestimate the positive impact of this screening approach.

CHALLENGES TO IMPLEMENTATION

Screening requires extra work

Personnel at the intervention sites received a half day of training in postpartum depression screening, diagnosis, and nursing telephone follow-up. The workload at these sites also increased, as most women found to have postpartum depression received one to 2 follow-up telephone calls and an average of one to 2 follow-up visits after the start of therapy. These measures, while seemingly modest, could pose a challenge to implementation. This could potentially be alleviated by the additional payments for care coordination promised in the Patient Protection and Affordable Care Act.13

ACKNOWLEDGEMENT

The PURLs Surveillance System was developed in part by Grant Number UL1RR024999 from the National Center for Research Resources, a Clinical Translational Science Award to The University of Chicago. The content is solely the responsibility of the authors and does not necessarily represent the official views of the National Center of Research Resources or the National Institutes of Health.

Practice changer

Make depression screening a routine part of the postpartum visit.1

Strength of recommendation

B: Based on a single blinded randomized controlled trial (RCT).

Yawn BP, Dietrich AJ, Wollan P, et al. TRIPPD practices. TRIPPD: a practice-based network effectiveness study of postpartum depression screening and management. Ann Fam Med. 2012;10:320-329.

 

Illustrative case

A 20-year-old patient comes in at 6 weeks’ postpartum for routine care. Should you screen her for postpartum depression?

The incidence of depression in the first 3 months’ postpartum is estimated at about 14%,2,3 and the consequences can be severe. A new mom with a mood disorder in the first year of her child’s life can disrupt the mother-infant relationship, thereby contributing to both short- and long-term adverse outcomes for the child. These include behavior problems, low self-esteem, poor self-regulation, and an increased risk of impaired mental and motor development.4,5

Postpartum depression often goes undetected

Despite this correlation, postpartum depression is both under-recognized and undertreated.6 A prospective randomized study of 5169 women who were screened for postpartum depression bears this out. Researchers found that about one in 4 (26%) of the 674 mothers who had positive screens were not asked about their emotional state by their clinicians.2

This may be due to a lack of evidence of the efficacy of screening for postpartum depression. In the Healthy Start Depression Initiative (n=1336), universal screening—with referrals to mental health care outside of the primary care physicians’ offices—did not alter either the level of depressive symptoms over a 10-month period or depression treatment.7

The American College of Obstetricians and Gynecologists’ Committee on Obstetric Practice noted in 2010 (and reaffirmed in 2012) that there is insufficient evidence to support a firm recommendation for universal postpartum screening for depression.8 Neither the US Preventive Services Task Force nor the American Academy of Family Physicians has a specific recommendation regarding postpartum depression.

STUDY SUMMARY

Screening—with follow-up—leads to better outcomes

The study by Yawn et al included 28 family medicine practices in 21 states, randomized to either usual care or intervention.1 Eligible practices had to have provided well-baby or maternity care to more than 30 patients in the previous year, but not to routinely screen for postpartum depression. A total of 2343 women aged 18 years or older—all of whom were between 5 and 12 weeks’ postpartum and planning on continuing care at the family medicine practice at the time they entered the study—were enrolled.

Staff at the 14 intervention sites received training in postpartum depression screening and diagnosis. They were also given a set of tools to facilitate management of postpartum depression, including an outline for follow-up visits and scripts for nurses to use for telephone calls relating to antidepressants. Therapy options were left to the discretion of the physician and the patient, with the help of tools that detailed the doses and adverse effects of various medications and described cognitive behavioral therapy.

Low socioeconomic status has been shown to be a strong risk factor for the development of postpartum depression.At the start of the study, participants at all 28 sites were given survey packets containing the Edinburgh Postnatal Depression Scale (EPDS) and the 9-item Patient Health Questionnaire (PHQ-9). Patients subsequently received surveys by mail at 6 and 12 months’ postpartum for self-reported outcomes. Clinicians at the intervention sites had routine access to the EPDS and PHQ-9 scores; those at the usual care sites did not.

The primary outcome was ≥5 point drop in the PHQ-9 score from baseline at 6 or 12 months’ postpartum, considered to be an indicator of clinical improvement and/or response to therapy. The PHQ-9 is a validated measure of depression severity, with the proven ability to detect changes over time.9,10

Of the 2343 women initially enrolled in the study, 1897 (81%) provided outcome information and were included in the analysis. The rates of women with elevated depression scores (EPDS and/or PHQ-9 ≥10) at the start were comparable between the intervention and usual care groups (29.5% vs 25.8%, respectively).

Of those whose scores were initially elevated, 219 women in the intervention group and 178 women in the usual care group returned surveys at 12 months and were included in the final analysis. The results: 45% of those in the intervention group met the primary outcome—a decline in self-reported depressive symptom levels, as indicated by a PHQ-9 decrease ≥5 points—compared with 35% of the women in the usual care group (odds ratio, 1.8; 95% confidence interval, 1.14-2.9; P=.001). Not surprisingly, medical record review also indicated that those in the intervention group who initially had elevated depression scores were more likely to have received a diagnosis (66% vs 41%; P=.0006) and therapy (20% vs 11%; P=.02) for postpartum depression.

 

 

 

WHAT'S NEW?

We now have evidence of the efficacy of postpartum screening

This is the first large study of a primary care-based approach to screening, diagnosis, and management of postpartum depression to show any improvement in maternal outcomes at 12 months. Prior universal screening and referral support in the Healthy Start program was done by paraprofessionals, who referred women with positive screens for mental health care outside of the primary care setting and did not reduce the rate of depression in perinatal women.7

CAVEATS

Dropout rate, socioeconomic status may affect results

Among the women who initially were found to be positive for postpartum depression, 38% did not return questionnaires at 12 months’ postpartum. While this loss to follow-up is high, it is comparable to that of most effectiveness trials11 with similar rates in the intervention and usual care groups.

Within the intervention group, there was no statistical difference between women who did and did not return the questionnaires with regard to marital status, history of depression, income, or uninsured status. However, women in the usual care group who did not return the 12-month questionnaire were more likely to be poor (89% vs 57%; P<.01) and uninsured (49% vs 29%; P<.01) than those who did return the questionnaire.

The impact of these differences and the loss to follow-up in this study is unknown. However, low socioeconomic status has been shown to be a strong risk factor for the development of postpartum depression.12 The authors of the study suggest (and we agree) that the difference in socioeconomic status in women who did not return the questionnaire may underestimate the positive impact of this screening approach.

CHALLENGES TO IMPLEMENTATION

Screening requires extra work

Personnel at the intervention sites received a half day of training in postpartum depression screening, diagnosis, and nursing telephone follow-up. The workload at these sites also increased, as most women found to have postpartum depression received one to 2 follow-up telephone calls and an average of one to 2 follow-up visits after the start of therapy. These measures, while seemingly modest, could pose a challenge to implementation. This could potentially be alleviated by the additional payments for care coordination promised in the Patient Protection and Affordable Care Act.13

ACKNOWLEDGEMENT

The PURLs Surveillance System was developed in part by Grant Number UL1RR024999 from the National Center for Research Resources, a Clinical Translational Science Award to The University of Chicago. The content is solely the responsibility of the authors and does not necessarily represent the official views of the National Center of Research Resources or the National Institutes of Health.

References

1. Yawn BP, Dietrich AJ, Wollan P, et al; TRIPPD practices. TRIPPD: a practice-based network effectiveness study of postpartum depression screening and management. Ann Fam Med. 2012;10:320-329.

2. Horowitz JA, Murphy CA, Gregory KE, et al. Best practices: community-based postpartum depression screening: results from the CARE study. Psychiatr Serv. 2009;60:1432-1434.

3. Gaynes BN, Gavin N, Meltzer-Brody S, et al. Perinatal Depression: Prevalence, Screening Accuracy, and Screening Outcomes: Summary. AHRQ Evidence Report Summaries. Rockville, MD: Agency for Healthcare Research and Quality; Feb, 2005.

4. Murray L, Cooper PJ. The impact of postpartum depression on child development. In: Goodyer I, ed. Aetiological Mechanisms in Developmental Psychopathology. Oxford, England: Oxford University Press; 2003.

5. Goodman SH, Gotlib IH. Risk for psychopathology in the children of depressed mothers: a developmental model for understanding mechanisms of transmission. Psychol Rev. 1999;106:458-490.

6. Pearlstein T, Howard M, Salisbury A, et al. Postpartum depression. Am J Obstet Gynecol. 2009;200:357-364.

7. Yonkers KA, Smith MV, Lin H, et al. Depression screening of perinatal women: an evaluation of the healthy start depression initiative. Psychiatr Serv. 2009;60:322-328.

8. American College of Obstetricians and Gynecologists. Committee on Obstetric Practice. Committee opinion no. 453: Screening for depression during and after pregnancy. Obstet Gynecol. 2010;115(2 pt 1):394-395.

9. Löwe B, Kroenke K, Herzog W, et al. Measuring depression outcome with a brief self-report instrument: sensitivity to change of the Patient Health Questionnaire (PHQ-9). J Affect Disord. 2004;81:61-66.

10. Kroenke K, Spitzer RL, Williams JB. The PHQ-9: validity of a brief depression severity measure. J Gen Intern Med. 2001;16:606–613.

11. Ribisl KM, Walton MA, Mowbray CT, et al. Minimizing participant attrition in panel studies through the use of effective retention and tracking strategies: review and recommendations. Eval Program Planning. 1996;19:1-25.

12. Dolbier CL, Rush TE, Sahadeo LS, et al; Community Health Network Investigators. Relationships of race and socioeconomic status to postpartum depressive symptoms in rural african american and non-Hispanic white women. Matern Child Health J. 2013;17:1277-1287.

13. Centers for Medicare and Medicaid Services. The Affordable Care Act: Helping Providers Help Patients. Available at: http://www.cms.gov/Medicare/Medicare-Fee-for-Service-Payment/ACO/downloads/ACO-Menu-Of-Options.pdf. Accessed September 6, 2013.

References

1. Yawn BP, Dietrich AJ, Wollan P, et al; TRIPPD practices. TRIPPD: a practice-based network effectiveness study of postpartum depression screening and management. Ann Fam Med. 2012;10:320-329.

2. Horowitz JA, Murphy CA, Gregory KE, et al. Best practices: community-based postpartum depression screening: results from the CARE study. Psychiatr Serv. 2009;60:1432-1434.

3. Gaynes BN, Gavin N, Meltzer-Brody S, et al. Perinatal Depression: Prevalence, Screening Accuracy, and Screening Outcomes: Summary. AHRQ Evidence Report Summaries. Rockville, MD: Agency for Healthcare Research and Quality; Feb, 2005.

4. Murray L, Cooper PJ. The impact of postpartum depression on child development. In: Goodyer I, ed. Aetiological Mechanisms in Developmental Psychopathology. Oxford, England: Oxford University Press; 2003.

5. Goodman SH, Gotlib IH. Risk for psychopathology in the children of depressed mothers: a developmental model for understanding mechanisms of transmission. Psychol Rev. 1999;106:458-490.

6. Pearlstein T, Howard M, Salisbury A, et al. Postpartum depression. Am J Obstet Gynecol. 2009;200:357-364.

7. Yonkers KA, Smith MV, Lin H, et al. Depression screening of perinatal women: an evaluation of the healthy start depression initiative. Psychiatr Serv. 2009;60:322-328.

8. American College of Obstetricians and Gynecologists. Committee on Obstetric Practice. Committee opinion no. 453: Screening for depression during and after pregnancy. Obstet Gynecol. 2010;115(2 pt 1):394-395.

9. Löwe B, Kroenke K, Herzog W, et al. Measuring depression outcome with a brief self-report instrument: sensitivity to change of the Patient Health Questionnaire (PHQ-9). J Affect Disord. 2004;81:61-66.

10. Kroenke K, Spitzer RL, Williams JB. The PHQ-9: validity of a brief depression severity measure. J Gen Intern Med. 2001;16:606–613.

11. Ribisl KM, Walton MA, Mowbray CT, et al. Minimizing participant attrition in panel studies through the use of effective retention and tracking strategies: review and recommendations. Eval Program Planning. 1996;19:1-25.

12. Dolbier CL, Rush TE, Sahadeo LS, et al; Community Health Network Investigators. Relationships of race and socioeconomic status to postpartum depressive symptoms in rural african american and non-Hispanic white women. Matern Child Health J. 2013;17:1277-1287.

13. Centers for Medicare and Medicaid Services. The Affordable Care Act: Helping Providers Help Patients. Available at: http://www.cms.gov/Medicare/Medicare-Fee-for-Service-Payment/ACO/downloads/ACO-Menu-Of-Options.pdf. Accessed September 6, 2013.

Issue
The Journal of Family Practice - 62(11)
Issue
The Journal of Family Practice - 62(11)
Page Number
E1-E3
Page Number
E1-E3
Publications
Publications
Topics
Article Type
Display Headline
Should you screen for postpartum depression?
Display Headline
Should you screen for postpartum depression?
Legacy Keywords
Andrew H. Slattengren; DO; Shailendra Prasad; MBBS; MPH; M. Altaf Kaiseruddin; MD; postpartum; depression; screening; postpartumdepression; PURLs; Healthy Start Depression Initiative; Patient Protection and Affordable Care Act
Legacy Keywords
Andrew H. Slattengren; DO; Shailendra Prasad; MBBS; MPH; M. Altaf Kaiseruddin; MD; postpartum; depression; screening; postpartumdepression; PURLs; Healthy Start Depression Initiative; Patient Protection and Affordable Care Act
Sections
PURLs Copyright

Copyright © 2013 Family Physicians Inquiries Network. All rights reserved.

Disallow All Ads
Alternative CME
Use ProPublica
Hide sidebar & use full width
render the right sidebar.
Conference Recap Checkbox
Not Conference Recap
Clinical Edge
Display the Slideshow in this Article
Article PDF Media
Media Files

Mediterranean diet: Higher fat but lower risk

Article Type
Changed
Tue, 07/14/2020 - 15:25
Display Headline
Mediterranean diet: Higher fat but lower risk
Practice changer

Counsel patients at high risk for cardiovascular disease and stroke to follow a Mediterranean diet, which is associated with a 30% risk reduction.1

Strength of recommendation

A: Based on one well-design randomized controlled trial (RCT).

Estruch R, Ros F, Salas-Salvado J, et al. Primary prevention of cardiovascular disease with a Mediterranean diet. N Engl J Med. 2013;368:1279-1290.

 

Illustrative case

A 62-year-old patient with diabetes, obesity, and a family history of early onset coronary artery disease is motivated to make significant lifestyle changes. You recommend moderate aerobic exercise for 30 minutes 5 times a week, but wonder whether a low-fat diet or a Mediterranean diet would be more effective in lowering her risk.

Cardiovascular disease (CVD), including heart disease and stroke, is the leading cause of mortality in the United States. CVD accounts for one in every 3 deaths,2 and stroke is a leading cause of long-term disability.2 The direct cost of treating CVD is estimated at $312.6 billion annually.2

Many modifiable risk factors contribute to CVD, including smoking, sedentary lifestyle, obesity, alcohol consumption, and poorly controlled chronic disease, as well as an unhealthy diet. A recent report from the American Heart Association suggests that 13% of deaths from CVD can be attributed to poor diet.2

Focus counseling on patients at risk
Primary care providers (PCPs) often struggle to effectively counsel patients on behavior change strategies, but face many barriers. Chief among them are the lack of time, training, and confidence in their counseling techniques, as well as a lack of patient motivation and readiness to change.3 In recognition of these barriers, the US Preventive Services Task Force recently recommended that PCPs focus behavioral counseling efforts on patients at high risk for heart disease.4

Large observational studies have found an association between trans fat and an increased risk of CVD, as well as a decreased risk of CVD in patients adhering to a Mediterranean diet.5-11 This type of diet typically includes a high intake of olive oil, fruit, nuts, vegetables, and cereals; moderate intake of fish and poultry; and low intake of dairy products, red meat, processed meats, and sweets. It also includes wine in moderation, consumed with meals.

Data on the physiologic properties of olive oil, including its antioxidant, vasodilating, and antiplatelet effects—as well as its effects on low-density lipoprotein cholesterol (LDL-C) that may inhibit atherogenesis—support the link between a Mediterranean diet and a decreased risk of CVD found in the observational studies.12,13 Until recently, however, no RCT had compared the effect of a Mediterranean diet with that of a low-fat diet for primary prevention of CVD.

STUDY SUMMARY: Mediterranean diet significantly lowers risk

Prevencion con Dieta Mediterranea (PREDIMED) was a large RCT (N=7447) comparing 2 variations of a Mediterranean diet with a low-fat diet for primary prevention of CVD. This Spanish study enrolled men 55 to 80 years of age and women ages 60 to 80 at high risk for developing CVD. The risk was based on either a diagnosis of type 2 diabetes or the presence of ≥3 major risk factors, including smoking, hypertension, elevated LDL-C, low high-density lipoprotein cholesterol, overweight or obese, and a family history of early heart disease.

A Mediterranean diet, with increased intake of either olive oil or mixed nuts, is more protective against CVD than a recommended low-fat diet.Participants were randomly assigned to one of 3 dietary groups: One group was assigned to a Mediterranean diet supplemented with ≥4 tablespoons per day of extra virgin olive oil; a second group was put on a Mediterranean diet supplemented by 30 grams (about 1/3 cup) of mixed nuts daily; a third group (the controls) was advised to follow a low-fat diet. The majority of baseline characteristics and medications taken throughout the study were similar among all 3 groups.

Those in both Mediterranean diet groups were followed for a median of 4.8 years, during which time they received quarterly dietary classes and individual and group counseling. The controls received baseline training, plus a leaflet about low-fat diets annually. In year 3, however, the researchers began giving the control group the same level of counseling as those in the Mediterranean diet groups to avoid confounding results.

Adherence to the diets was determined by a self-reported 14-item dietary screening questionnaire, plus urinary hydroxytyrosol and serum alpha-linoleic acid levels to assess for olive oil and mixed nut compliance. Self-reporting5 and biometric data indicated good compliance with the Mediterranean diets, and there was no difference found in levels of exercise among the groups.

After 5 years, those in the Mediterranean diet groups had consumed significantly more olive oil, nuts, vegetables, fruits, wine, legumes, fish, seafood, and sofrito sauce (a popular tomato-based sauce) than the control group. Participants in the low-fat diet group had decreased their fat intake by 2%, while those in the Mediterranean groups had increased fat intake (by 2.03% for the olive oil group and 2.1% for the nut group). Overall, 37% of energy intake by those in the low-fat diet group came from fat (exceeding the <30% of calories derived from fat intake that defines a low-fat diet) vs 39% fat intake for those in both Mediterranean diet groups.

 

 

 

The primary outcome was a composite of myocardial infarction (MI), stroke, and death from cardiovascular causes, and there were clinically meaningful and statistically significant differences between the Mediterranean diet groups and the controls. The primary outcome rate for the supplemental olive oil group was 3.8%; 3.4% for the extra nuts group; and 4.4% for the controls. This represents a 30% reduction in risk for combined stroke, MI, and death due to cardiovascular causes for the Mediterranean diet groups (hazard ratio [HR]=0.7; 95% confidence interval [CI], 0.53-0.91; P=.009; number needed to treat [NNT]=148 for the olive oil group and HR=0.7; 95% CI, 0.53-0.94; P=.02; NNT=100 for the group consuming extra nuts). Similar benefits were found in the multivariable adjusted analyses. The results correspond to 3 fewer events (stroke, MI, or cardiovascular death) per 1000 person-years for this high-risk population.

The only individual outcome that showed a significant decrease was stroke, with an NNT of 125 in both Mediterranean diet groups. Outcomes for the controls were similar before and after they began receiving quarterly counseling.

WHAT'S NEW?: Mediterranean diet is better than a lower-fat regimen

Rates of CVD are higher in the United States than in Spain, so implementing a Mediterranean diet on a large scale in this country has the potential to produce a greater response than that seen in this study. This study indicates that a Mediterranean diet, with increased intake of either olive oil or mixed nuts, is more protective against CVD than a recommended low-fat diet. It also shows that advising patients at high risk to follow a Mediterranean diet, providing dietary counseling, and monitoring them for adherence, rather than simply recommending a low-fat diet, can significantly decrease the risk of stroke.

Rates of CVD are higher in the United States than in Spain, so implementing a Mediterranean diet on a large scale in this country has the potential to produce a greater response than that seen in this study.

CAVEATS: Would a true low-fat diet be a better comparison?

Although the control group’s diet was meant to be low fat, the participants did not achieve this, possibly due to the relatively low level of dietary education and personalized counseling at the start of the study. Their inability to reach the <30% fat target could also reflect the difficulty patients have, in general, in decreasing fat content in their diet, which may mean the diet they maintained was a more realistic comparison.

This study used one brand of olive oil and a particular mixture of nuts (walnuts, hazelnuts, and almonds); it is possible that variations on either of these could affect the benefits of the diet.

CHALLENGES TO IMPLEMENTATION: Fitting a Mediterranean diet into an American lifestyle

The typical US diet is significantly different from that of most Spaniards. Americans may find it difficult to add either ≥4 tablespoons of olive oil or 30 g (1/3 cup) of nuts daily, for example, due to both cost and availability. Limited access to both individual and group counseling could be a barrier, as well.

On the other hand, this practice changer has the potential to simplify dietary counseling by allowing clinicians to focus on just one type of diet, for which there are many resources available both online and in print. We believe it makes sense to recommend a Mediterranean diet, while continuing to recommend increased exercise, smoking cessation, and improved control of chronic disease to lower patients’ risk of poor outcomes from CVD.

Acknowledgement

The PURLs Surveillance System was supported in part by Grant Number UL 1RR 024999 from the National Center for Research Resources, a Clinical Translational Science Award to the University of Chicago. The content is solely the responsibility of the authors and does not necessarily represent the official views of the National Center for Research Resources or the National Institutes of Health.

Files
References

1. Estruch R, Ros F, Salas-Salvado J, et al. Primary prevention of cardiovascular disease with a Mediterranean Diet. N Engl J Med. 2013;368:1279-1290.

2. Go AS, Mozaffarian D, Roger VL, et al; American Heart Association Statistics Committee and Stroke Statistics Subcommittee. Heart disease and stroke statistics—2013 update: a report from the American Heart Association. Circulation. 2013;127:e6-e245.

3. Kushner RF. Barriers to providing nutrition counseling by physicians: a survey of primary care practitioners. Prev Med. 1995;24:546-552.

4. Behavioral Counseling to Promote a Healthful Diet and Physical Activity for Cardiovascular Disease Prevention in Adults, topic page. US Preventive Services Task Force Web site. Available at: http://www.uspreventiveservicestaskforce.org/uspstf/usp- sphys.htm. Accessed August 1, 2013.

5. Hu FB, Stampfer MJ, Manson JE, et al. Dietary fat intake and the risk of coronary heart disease in women. N Engl J Med. 1997;337:1491-1499.

6. Oomen C, Ocké MC, Feskens JM, et al. Association between trans fatty acid intake and 10-year risk of coronary heart disease in the Zutphen Elderly Study: a prospective population-based study. Lancet. 2001;357:746-751.

7. de Lorgeril M, Salen P, Martin JL, et al. Mediterranean diet, traditional risk factors and the rate of cardiovascular complications after myocardial infarction. Final report of the Lyon Diet Heart Study. Circulation. 1999;99:779-785.

8. Knoops KT, de Groot LC, Kromhout D, et al. Mediterranean diet, lifestyle factors, and 10-year mortality in elderly European men and women: the HALE project. JAMA. 2004;292:1433-1439.

9. Kris-Etherton P, Eckel RH, Howard BV, et al; Nutrition Committee Population Science Committee and Clinical Science Committee of the American Heart Association. Lyon Diet Heart Study. Benefits of a Mediterranean-style, National Education Program/AHA Step 1 Dietary Pattern on cardiovascular disease. Circulation. 2001;103:1823-1825.

10. Panagiotakos DB, Chrysohoou C, Pitsavos C, et al. The association of Mediterranean diet with lower risk of acute coronary syndromes, in hypertensive subjects. Int J Cardiol. 2002;82:141-147.

11. Panagiotakos DB, Pitsavos C, Chrysohoou C, et al. The role of traditional Mediterranean-type of diet and lifestyle, in the devel- opment of acute coronary syndromes: preliminary results from CARDIO 2000 study. Centr Eur J Public Health. 2002;10:11-15.

12. Esposito K, Marfella R, Ciotola M, et al. Effect of a Mediterranean-style diet on endothelial dysfunction and markers of vascular inflammation in the metabolic syndrome: a randomized trial. JAMA. 2004;292:1440-1446.

13. Vincent-Baudry S, Defoort C, Gerber M, et al. The Medi-RIVAGE study: reduction of cardiovascular disease risk factors after a 3-mo intervention with a Mediterranean-type diet or a low-fat diet. Am J Clin Nutr. 2005;82:964-971.

Article PDF
Author and Disclosure Information

Mark Gwynne, DO
Anne Mounsey, MD

Department of Family Medicine, University of North Carolina at Chapel Hill

PURLs EDITORS
Bernard Ewigman, MD, MSPH
James Stevermer, MD, MSPH

The University of Chicago (Dr. Ewigman); University of Missouri, Columbia (Dr. Stevermer)

Issue
The Journal of Family Practice - 62(12)
Publications
Topics
Page Number
745-746, 748
Legacy Keywords
Mark Gwynne; DO; Anne Mounsey; MD; diet; nutrition; mediterranean diet; cardiovascular disease; CVD
Sections
Files
Files
Author and Disclosure Information

Mark Gwynne, DO
Anne Mounsey, MD

Department of Family Medicine, University of North Carolina at Chapel Hill

PURLs EDITORS
Bernard Ewigman, MD, MSPH
James Stevermer, MD, MSPH

The University of Chicago (Dr. Ewigman); University of Missouri, Columbia (Dr. Stevermer)

Author and Disclosure Information

Mark Gwynne, DO
Anne Mounsey, MD

Department of Family Medicine, University of North Carolina at Chapel Hill

PURLs EDITORS
Bernard Ewigman, MD, MSPH
James Stevermer, MD, MSPH

The University of Chicago (Dr. Ewigman); University of Missouri, Columbia (Dr. Stevermer)

Article PDF
Article PDF
Practice changer

Counsel patients at high risk for cardiovascular disease and stroke to follow a Mediterranean diet, which is associated with a 30% risk reduction.1

Strength of recommendation

A: Based on one well-design randomized controlled trial (RCT).

Estruch R, Ros F, Salas-Salvado J, et al. Primary prevention of cardiovascular disease with a Mediterranean diet. N Engl J Med. 2013;368:1279-1290.

 

Illustrative case

A 62-year-old patient with diabetes, obesity, and a family history of early onset coronary artery disease is motivated to make significant lifestyle changes. You recommend moderate aerobic exercise for 30 minutes 5 times a week, but wonder whether a low-fat diet or a Mediterranean diet would be more effective in lowering her risk.

Cardiovascular disease (CVD), including heart disease and stroke, is the leading cause of mortality in the United States. CVD accounts for one in every 3 deaths,2 and stroke is a leading cause of long-term disability.2 The direct cost of treating CVD is estimated at $312.6 billion annually.2

Many modifiable risk factors contribute to CVD, including smoking, sedentary lifestyle, obesity, alcohol consumption, and poorly controlled chronic disease, as well as an unhealthy diet. A recent report from the American Heart Association suggests that 13% of deaths from CVD can be attributed to poor diet.2

Focus counseling on patients at risk
Primary care providers (PCPs) often struggle to effectively counsel patients on behavior change strategies, but face many barriers. Chief among them are the lack of time, training, and confidence in their counseling techniques, as well as a lack of patient motivation and readiness to change.3 In recognition of these barriers, the US Preventive Services Task Force recently recommended that PCPs focus behavioral counseling efforts on patients at high risk for heart disease.4

Large observational studies have found an association between trans fat and an increased risk of CVD, as well as a decreased risk of CVD in patients adhering to a Mediterranean diet.5-11 This type of diet typically includes a high intake of olive oil, fruit, nuts, vegetables, and cereals; moderate intake of fish and poultry; and low intake of dairy products, red meat, processed meats, and sweets. It also includes wine in moderation, consumed with meals.

Data on the physiologic properties of olive oil, including its antioxidant, vasodilating, and antiplatelet effects—as well as its effects on low-density lipoprotein cholesterol (LDL-C) that may inhibit atherogenesis—support the link between a Mediterranean diet and a decreased risk of CVD found in the observational studies.12,13 Until recently, however, no RCT had compared the effect of a Mediterranean diet with that of a low-fat diet for primary prevention of CVD.

STUDY SUMMARY: Mediterranean diet significantly lowers risk

Prevencion con Dieta Mediterranea (PREDIMED) was a large RCT (N=7447) comparing 2 variations of a Mediterranean diet with a low-fat diet for primary prevention of CVD. This Spanish study enrolled men 55 to 80 years of age and women ages 60 to 80 at high risk for developing CVD. The risk was based on either a diagnosis of type 2 diabetes or the presence of ≥3 major risk factors, including smoking, hypertension, elevated LDL-C, low high-density lipoprotein cholesterol, overweight or obese, and a family history of early heart disease.

A Mediterranean diet, with increased intake of either olive oil or mixed nuts, is more protective against CVD than a recommended low-fat diet.Participants were randomly assigned to one of 3 dietary groups: One group was assigned to a Mediterranean diet supplemented with ≥4 tablespoons per day of extra virgin olive oil; a second group was put on a Mediterranean diet supplemented by 30 grams (about 1/3 cup) of mixed nuts daily; a third group (the controls) was advised to follow a low-fat diet. The majority of baseline characteristics and medications taken throughout the study were similar among all 3 groups.

Those in both Mediterranean diet groups were followed for a median of 4.8 years, during which time they received quarterly dietary classes and individual and group counseling. The controls received baseline training, plus a leaflet about low-fat diets annually. In year 3, however, the researchers began giving the control group the same level of counseling as those in the Mediterranean diet groups to avoid confounding results.

Adherence to the diets was determined by a self-reported 14-item dietary screening questionnaire, plus urinary hydroxytyrosol and serum alpha-linoleic acid levels to assess for olive oil and mixed nut compliance. Self-reporting5 and biometric data indicated good compliance with the Mediterranean diets, and there was no difference found in levels of exercise among the groups.

After 5 years, those in the Mediterranean diet groups had consumed significantly more olive oil, nuts, vegetables, fruits, wine, legumes, fish, seafood, and sofrito sauce (a popular tomato-based sauce) than the control group. Participants in the low-fat diet group had decreased their fat intake by 2%, while those in the Mediterranean groups had increased fat intake (by 2.03% for the olive oil group and 2.1% for the nut group). Overall, 37% of energy intake by those in the low-fat diet group came from fat (exceeding the <30% of calories derived from fat intake that defines a low-fat diet) vs 39% fat intake for those in both Mediterranean diet groups.

 

 

 

The primary outcome was a composite of myocardial infarction (MI), stroke, and death from cardiovascular causes, and there were clinically meaningful and statistically significant differences between the Mediterranean diet groups and the controls. The primary outcome rate for the supplemental olive oil group was 3.8%; 3.4% for the extra nuts group; and 4.4% for the controls. This represents a 30% reduction in risk for combined stroke, MI, and death due to cardiovascular causes for the Mediterranean diet groups (hazard ratio [HR]=0.7; 95% confidence interval [CI], 0.53-0.91; P=.009; number needed to treat [NNT]=148 for the olive oil group and HR=0.7; 95% CI, 0.53-0.94; P=.02; NNT=100 for the group consuming extra nuts). Similar benefits were found in the multivariable adjusted analyses. The results correspond to 3 fewer events (stroke, MI, or cardiovascular death) per 1000 person-years for this high-risk population.

The only individual outcome that showed a significant decrease was stroke, with an NNT of 125 in both Mediterranean diet groups. Outcomes for the controls were similar before and after they began receiving quarterly counseling.

WHAT'S NEW?: Mediterranean diet is better than a lower-fat regimen

Rates of CVD are higher in the United States than in Spain, so implementing a Mediterranean diet on a large scale in this country has the potential to produce a greater response than that seen in this study. This study indicates that a Mediterranean diet, with increased intake of either olive oil or mixed nuts, is more protective against CVD than a recommended low-fat diet. It also shows that advising patients at high risk to follow a Mediterranean diet, providing dietary counseling, and monitoring them for adherence, rather than simply recommending a low-fat diet, can significantly decrease the risk of stroke.

Rates of CVD are higher in the United States than in Spain, so implementing a Mediterranean diet on a large scale in this country has the potential to produce a greater response than that seen in this study.

CAVEATS: Would a true low-fat diet be a better comparison?

Although the control group’s diet was meant to be low fat, the participants did not achieve this, possibly due to the relatively low level of dietary education and personalized counseling at the start of the study. Their inability to reach the <30% fat target could also reflect the difficulty patients have, in general, in decreasing fat content in their diet, which may mean the diet they maintained was a more realistic comparison.

This study used one brand of olive oil and a particular mixture of nuts (walnuts, hazelnuts, and almonds); it is possible that variations on either of these could affect the benefits of the diet.

CHALLENGES TO IMPLEMENTATION: Fitting a Mediterranean diet into an American lifestyle

The typical US diet is significantly different from that of most Spaniards. Americans may find it difficult to add either ≥4 tablespoons of olive oil or 30 g (1/3 cup) of nuts daily, for example, due to both cost and availability. Limited access to both individual and group counseling could be a barrier, as well.

On the other hand, this practice changer has the potential to simplify dietary counseling by allowing clinicians to focus on just one type of diet, for which there are many resources available both online and in print. We believe it makes sense to recommend a Mediterranean diet, while continuing to recommend increased exercise, smoking cessation, and improved control of chronic disease to lower patients’ risk of poor outcomes from CVD.

Acknowledgement

The PURLs Surveillance System was supported in part by Grant Number UL 1RR 024999 from the National Center for Research Resources, a Clinical Translational Science Award to the University of Chicago. The content is solely the responsibility of the authors and does not necessarily represent the official views of the National Center for Research Resources or the National Institutes of Health.

Practice changer

Counsel patients at high risk for cardiovascular disease and stroke to follow a Mediterranean diet, which is associated with a 30% risk reduction.1

Strength of recommendation

A: Based on one well-design randomized controlled trial (RCT).

Estruch R, Ros F, Salas-Salvado J, et al. Primary prevention of cardiovascular disease with a Mediterranean diet. N Engl J Med. 2013;368:1279-1290.

 

Illustrative case

A 62-year-old patient with diabetes, obesity, and a family history of early onset coronary artery disease is motivated to make significant lifestyle changes. You recommend moderate aerobic exercise for 30 minutes 5 times a week, but wonder whether a low-fat diet or a Mediterranean diet would be more effective in lowering her risk.

Cardiovascular disease (CVD), including heart disease and stroke, is the leading cause of mortality in the United States. CVD accounts for one in every 3 deaths,2 and stroke is a leading cause of long-term disability.2 The direct cost of treating CVD is estimated at $312.6 billion annually.2

Many modifiable risk factors contribute to CVD, including smoking, sedentary lifestyle, obesity, alcohol consumption, and poorly controlled chronic disease, as well as an unhealthy diet. A recent report from the American Heart Association suggests that 13% of deaths from CVD can be attributed to poor diet.2

Focus counseling on patients at risk
Primary care providers (PCPs) often struggle to effectively counsel patients on behavior change strategies, but face many barriers. Chief among them are the lack of time, training, and confidence in their counseling techniques, as well as a lack of patient motivation and readiness to change.3 In recognition of these barriers, the US Preventive Services Task Force recently recommended that PCPs focus behavioral counseling efforts on patients at high risk for heart disease.4

Large observational studies have found an association between trans fat and an increased risk of CVD, as well as a decreased risk of CVD in patients adhering to a Mediterranean diet.5-11 This type of diet typically includes a high intake of olive oil, fruit, nuts, vegetables, and cereals; moderate intake of fish and poultry; and low intake of dairy products, red meat, processed meats, and sweets. It also includes wine in moderation, consumed with meals.

Data on the physiologic properties of olive oil, including its antioxidant, vasodilating, and antiplatelet effects—as well as its effects on low-density lipoprotein cholesterol (LDL-C) that may inhibit atherogenesis—support the link between a Mediterranean diet and a decreased risk of CVD found in the observational studies.12,13 Until recently, however, no RCT had compared the effect of a Mediterranean diet with that of a low-fat diet for primary prevention of CVD.

STUDY SUMMARY: Mediterranean diet significantly lowers risk

Prevencion con Dieta Mediterranea (PREDIMED) was a large RCT (N=7447) comparing 2 variations of a Mediterranean diet with a low-fat diet for primary prevention of CVD. This Spanish study enrolled men 55 to 80 years of age and women ages 60 to 80 at high risk for developing CVD. The risk was based on either a diagnosis of type 2 diabetes or the presence of ≥3 major risk factors, including smoking, hypertension, elevated LDL-C, low high-density lipoprotein cholesterol, overweight or obese, and a family history of early heart disease.

A Mediterranean diet, with increased intake of either olive oil or mixed nuts, is more protective against CVD than a recommended low-fat diet.Participants were randomly assigned to one of 3 dietary groups: One group was assigned to a Mediterranean diet supplemented with ≥4 tablespoons per day of extra virgin olive oil; a second group was put on a Mediterranean diet supplemented by 30 grams (about 1/3 cup) of mixed nuts daily; a third group (the controls) was advised to follow a low-fat diet. The majority of baseline characteristics and medications taken throughout the study were similar among all 3 groups.

Those in both Mediterranean diet groups were followed for a median of 4.8 years, during which time they received quarterly dietary classes and individual and group counseling. The controls received baseline training, plus a leaflet about low-fat diets annually. In year 3, however, the researchers began giving the control group the same level of counseling as those in the Mediterranean diet groups to avoid confounding results.

Adherence to the diets was determined by a self-reported 14-item dietary screening questionnaire, plus urinary hydroxytyrosol and serum alpha-linoleic acid levels to assess for olive oil and mixed nut compliance. Self-reporting5 and biometric data indicated good compliance with the Mediterranean diets, and there was no difference found in levels of exercise among the groups.

After 5 years, those in the Mediterranean diet groups had consumed significantly more olive oil, nuts, vegetables, fruits, wine, legumes, fish, seafood, and sofrito sauce (a popular tomato-based sauce) than the control group. Participants in the low-fat diet group had decreased their fat intake by 2%, while those in the Mediterranean groups had increased fat intake (by 2.03% for the olive oil group and 2.1% for the nut group). Overall, 37% of energy intake by those in the low-fat diet group came from fat (exceeding the <30% of calories derived from fat intake that defines a low-fat diet) vs 39% fat intake for those in both Mediterranean diet groups.

 

 

 

The primary outcome was a composite of myocardial infarction (MI), stroke, and death from cardiovascular causes, and there were clinically meaningful and statistically significant differences between the Mediterranean diet groups and the controls. The primary outcome rate for the supplemental olive oil group was 3.8%; 3.4% for the extra nuts group; and 4.4% for the controls. This represents a 30% reduction in risk for combined stroke, MI, and death due to cardiovascular causes for the Mediterranean diet groups (hazard ratio [HR]=0.7; 95% confidence interval [CI], 0.53-0.91; P=.009; number needed to treat [NNT]=148 for the olive oil group and HR=0.7; 95% CI, 0.53-0.94; P=.02; NNT=100 for the group consuming extra nuts). Similar benefits were found in the multivariable adjusted analyses. The results correspond to 3 fewer events (stroke, MI, or cardiovascular death) per 1000 person-years for this high-risk population.

The only individual outcome that showed a significant decrease was stroke, with an NNT of 125 in both Mediterranean diet groups. Outcomes for the controls were similar before and after they began receiving quarterly counseling.

WHAT'S NEW?: Mediterranean diet is better than a lower-fat regimen

Rates of CVD are higher in the United States than in Spain, so implementing a Mediterranean diet on a large scale in this country has the potential to produce a greater response than that seen in this study. This study indicates that a Mediterranean diet, with increased intake of either olive oil or mixed nuts, is more protective against CVD than a recommended low-fat diet. It also shows that advising patients at high risk to follow a Mediterranean diet, providing dietary counseling, and monitoring them for adherence, rather than simply recommending a low-fat diet, can significantly decrease the risk of stroke.

Rates of CVD are higher in the United States than in Spain, so implementing a Mediterranean diet on a large scale in this country has the potential to produce a greater response than that seen in this study.

CAVEATS: Would a true low-fat diet be a better comparison?

Although the control group’s diet was meant to be low fat, the participants did not achieve this, possibly due to the relatively low level of dietary education and personalized counseling at the start of the study. Their inability to reach the <30% fat target could also reflect the difficulty patients have, in general, in decreasing fat content in their diet, which may mean the diet they maintained was a more realistic comparison.

This study used one brand of olive oil and a particular mixture of nuts (walnuts, hazelnuts, and almonds); it is possible that variations on either of these could affect the benefits of the diet.

CHALLENGES TO IMPLEMENTATION: Fitting a Mediterranean diet into an American lifestyle

The typical US diet is significantly different from that of most Spaniards. Americans may find it difficult to add either ≥4 tablespoons of olive oil or 30 g (1/3 cup) of nuts daily, for example, due to both cost and availability. Limited access to both individual and group counseling could be a barrier, as well.

On the other hand, this practice changer has the potential to simplify dietary counseling by allowing clinicians to focus on just one type of diet, for which there are many resources available both online and in print. We believe it makes sense to recommend a Mediterranean diet, while continuing to recommend increased exercise, smoking cessation, and improved control of chronic disease to lower patients’ risk of poor outcomes from CVD.

Acknowledgement

The PURLs Surveillance System was supported in part by Grant Number UL 1RR 024999 from the National Center for Research Resources, a Clinical Translational Science Award to the University of Chicago. The content is solely the responsibility of the authors and does not necessarily represent the official views of the National Center for Research Resources or the National Institutes of Health.

References

1. Estruch R, Ros F, Salas-Salvado J, et al. Primary prevention of cardiovascular disease with a Mediterranean Diet. N Engl J Med. 2013;368:1279-1290.

2. Go AS, Mozaffarian D, Roger VL, et al; American Heart Association Statistics Committee and Stroke Statistics Subcommittee. Heart disease and stroke statistics—2013 update: a report from the American Heart Association. Circulation. 2013;127:e6-e245.

3. Kushner RF. Barriers to providing nutrition counseling by physicians: a survey of primary care practitioners. Prev Med. 1995;24:546-552.

4. Behavioral Counseling to Promote a Healthful Diet and Physical Activity for Cardiovascular Disease Prevention in Adults, topic page. US Preventive Services Task Force Web site. Available at: http://www.uspreventiveservicestaskforce.org/uspstf/usp- sphys.htm. Accessed August 1, 2013.

5. Hu FB, Stampfer MJ, Manson JE, et al. Dietary fat intake and the risk of coronary heart disease in women. N Engl J Med. 1997;337:1491-1499.

6. Oomen C, Ocké MC, Feskens JM, et al. Association between trans fatty acid intake and 10-year risk of coronary heart disease in the Zutphen Elderly Study: a prospective population-based study. Lancet. 2001;357:746-751.

7. de Lorgeril M, Salen P, Martin JL, et al. Mediterranean diet, traditional risk factors and the rate of cardiovascular complications after myocardial infarction. Final report of the Lyon Diet Heart Study. Circulation. 1999;99:779-785.

8. Knoops KT, de Groot LC, Kromhout D, et al. Mediterranean diet, lifestyle factors, and 10-year mortality in elderly European men and women: the HALE project. JAMA. 2004;292:1433-1439.

9. Kris-Etherton P, Eckel RH, Howard BV, et al; Nutrition Committee Population Science Committee and Clinical Science Committee of the American Heart Association. Lyon Diet Heart Study. Benefits of a Mediterranean-style, National Education Program/AHA Step 1 Dietary Pattern on cardiovascular disease. Circulation. 2001;103:1823-1825.

10. Panagiotakos DB, Chrysohoou C, Pitsavos C, et al. The association of Mediterranean diet with lower risk of acute coronary syndromes, in hypertensive subjects. Int J Cardiol. 2002;82:141-147.

11. Panagiotakos DB, Pitsavos C, Chrysohoou C, et al. The role of traditional Mediterranean-type of diet and lifestyle, in the devel- opment of acute coronary syndromes: preliminary results from CARDIO 2000 study. Centr Eur J Public Health. 2002;10:11-15.

12. Esposito K, Marfella R, Ciotola M, et al. Effect of a Mediterranean-style diet on endothelial dysfunction and markers of vascular inflammation in the metabolic syndrome: a randomized trial. JAMA. 2004;292:1440-1446.

13. Vincent-Baudry S, Defoort C, Gerber M, et al. The Medi-RIVAGE study: reduction of cardiovascular disease risk factors after a 3-mo intervention with a Mediterranean-type diet or a low-fat diet. Am J Clin Nutr. 2005;82:964-971.

References

1. Estruch R, Ros F, Salas-Salvado J, et al. Primary prevention of cardiovascular disease with a Mediterranean Diet. N Engl J Med. 2013;368:1279-1290.

2. Go AS, Mozaffarian D, Roger VL, et al; American Heart Association Statistics Committee and Stroke Statistics Subcommittee. Heart disease and stroke statistics—2013 update: a report from the American Heart Association. Circulation. 2013;127:e6-e245.

3. Kushner RF. Barriers to providing nutrition counseling by physicians: a survey of primary care practitioners. Prev Med. 1995;24:546-552.

4. Behavioral Counseling to Promote a Healthful Diet and Physical Activity for Cardiovascular Disease Prevention in Adults, topic page. US Preventive Services Task Force Web site. Available at: http://www.uspreventiveservicestaskforce.org/uspstf/usp- sphys.htm. Accessed August 1, 2013.

5. Hu FB, Stampfer MJ, Manson JE, et al. Dietary fat intake and the risk of coronary heart disease in women. N Engl J Med. 1997;337:1491-1499.

6. Oomen C, Ocké MC, Feskens JM, et al. Association between trans fatty acid intake and 10-year risk of coronary heart disease in the Zutphen Elderly Study: a prospective population-based study. Lancet. 2001;357:746-751.

7. de Lorgeril M, Salen P, Martin JL, et al. Mediterranean diet, traditional risk factors and the rate of cardiovascular complications after myocardial infarction. Final report of the Lyon Diet Heart Study. Circulation. 1999;99:779-785.

8. Knoops KT, de Groot LC, Kromhout D, et al. Mediterranean diet, lifestyle factors, and 10-year mortality in elderly European men and women: the HALE project. JAMA. 2004;292:1433-1439.

9. Kris-Etherton P, Eckel RH, Howard BV, et al; Nutrition Committee Population Science Committee and Clinical Science Committee of the American Heart Association. Lyon Diet Heart Study. Benefits of a Mediterranean-style, National Education Program/AHA Step 1 Dietary Pattern on cardiovascular disease. Circulation. 2001;103:1823-1825.

10. Panagiotakos DB, Chrysohoou C, Pitsavos C, et al. The association of Mediterranean diet with lower risk of acute coronary syndromes, in hypertensive subjects. Int J Cardiol. 2002;82:141-147.

11. Panagiotakos DB, Pitsavos C, Chrysohoou C, et al. The role of traditional Mediterranean-type of diet and lifestyle, in the devel- opment of acute coronary syndromes: preliminary results from CARDIO 2000 study. Centr Eur J Public Health. 2002;10:11-15.

12. Esposito K, Marfella R, Ciotola M, et al. Effect of a Mediterranean-style diet on endothelial dysfunction and markers of vascular inflammation in the metabolic syndrome: a randomized trial. JAMA. 2004;292:1440-1446.

13. Vincent-Baudry S, Defoort C, Gerber M, et al. The Medi-RIVAGE study: reduction of cardiovascular disease risk factors after a 3-mo intervention with a Mediterranean-type diet or a low-fat diet. Am J Clin Nutr. 2005;82:964-971.

Issue
The Journal of Family Practice - 62(12)
Issue
The Journal of Family Practice - 62(12)
Page Number
745-746, 748
Page Number
745-746, 748
Publications
Publications
Topics
Article Type
Display Headline
Mediterranean diet: Higher fat but lower risk
Display Headline
Mediterranean diet: Higher fat but lower risk
Legacy Keywords
Mark Gwynne; DO; Anne Mounsey; MD; diet; nutrition; mediterranean diet; cardiovascular disease; CVD
Legacy Keywords
Mark Gwynne; DO; Anne Mounsey; MD; diet; nutrition; mediterranean diet; cardiovascular disease; CVD
Sections
PURLs Copyright

Copyright © 2013 Family Physicians Inquiries Network. All rights reserved.

Disallow All Ads
Alternative CME
Use ProPublica
Hide sidebar & use full width
render the right sidebar.
Conference Recap Checkbox
Not Conference Recap
Clinical Edge
Display the Slideshow in this Article
Article PDF Media
Media Files

Easing the Discomfort of a Speculum Exam

Article Type
Changed
Tue, 12/13/2016 - 12:08
Display Headline
Easing the Discomfort of a Speculum Exam
Applying a small amount of gel to the blades of a speculum before insertion won’t alter test results—but will reduce the patient’s discomfort.

PRACTICE CHANGER

Put lubricating gel, not water, on the speculum every time you do a pelvic exam.1

STRENGTH OF ­RECOMMENDATION

B: Based on one good-quality, randomized controlled trial.

Hill DA, Lamvu G. Effect of lubricating gel on patient comfort during vaginal speculum examination. Obstet Gynecol. 2012;119 (2 pt 1):
227-231.

ILLUSTRATIVE CASE

A 24-year-old woman presents for an annual exam, including Pap smear and testing for sexually transmitted diseases. She tells you how painful her previous speculum exam was and how worried she is about having another. To ease her discomfort, should you apply lubricating gel or water to the speculum before vaginal insertion?

Physiology teaches us that vaginal entry requires lubrication. But traditional teaching has held that lubricating gel on a speculum can interfere with the results of a Pap smear and chlamydia tesing.2 Pelvic exams performed without lubricating gel on the speculum can cause significant discomfort—possibly bad enough to prevent some women from undergoing the recommended screening tests.3

Until now, we’ve only ­evaluated gel’s impact on test results

Studies comparing lubricating gel and water have conclusively shown that a small amount of gel, used on the outside of the speculum blades, does not interfere with either Pap testing or detection of Chlamydia trachomatis.4,5 One liquid-based cytology manufacturer, however, discourages the use of lubricants with “carbomers” or “carbopol polymers,” but states that water-based lubricants have not been shown to interfere with Pap smear results.6 No studies have evaluated lubricants from a patient perspective—until now.

STUDY SUMMARY

Lubricating gel eases discomfort

The study by Hill and Lamvu1 was a six-month, single-blind, randomized trial of women ages 18 to 50 who sought care at an Orlando obstetrics and gynecology department for conditions requiring vaginal speculum examination.1 The study excluded women who might have an altered perception of pain during speculum insertion: those who were menopausal, pregnant, or within six weeks’ postpartum; had dyspareunia, vaginitis, vulvar pain, or vulvar lesions; were undergoing a procedure; or had never had vaginal intercourse. Women who were not fluent in English were excluded, as well.

The study included 120 women who underwent computer-generated randomization into two groups with no marked differences in demographics. A single examiner performed all the speculum exams, using a standard protocol with a medium-size Graves speculum. The examiner applied 0.3 mL water-based lubricating gel to the speculum before insertion for the women in one group, and used 3 mL water for the other.

Immediately after the speculum was inserted and opened—before the examiner attempted to visualize the cervix—patients were given a visual analog scale and told to indicate the level of pain with insertion, using a scale of 0 (no pain) to 10 (the worst pain imaginable). The gel group had lower pain scores for speculum insertion compared with the water group (1.41 ± 1.55 vs 2.15 ± 1.93), a statistically significant difference of 0.74. Twenty of the 59 patients in the gel group (33.9%) rated their pain as 0, compared with six of 60 (10%) in the water group.

Although pain, rather than sampling quality, was the primary outcome of the study, the authors also reported that all of the women who underwent Pap screening (73) had adequate cytology.

WHAT’S NEW?

It’s time for lubrication to become standard practice

This trial is the first to study speculum lubrication from this patient-oriented outcome and to show that women experience less pain when lubricating gel, rather than water, is applied to the speculum. This knowledge, combined with previous studies showing that a small amount of water-based lubricating gel does not interfere with liquid-based cytology or chlamydia test results, should make the use of lubricating gel standard practice when performing speculum examinations.

CAVEATS

We see no downside

The exclusion criteria of this study were meant to eliminate women who had an altered perception of pain that could skew study results. Yet those who met the exclusion criteria may also benefit from a pelvic exam with gel lubrication. We see no harm in trying a small amount of lubricant when examining them, as well.

In addition, the study did not compare various types and sizes of specula. However, we see no reason why the benefit of a gel lubricant would be limited to the type of speculum used by the examiner.

Studies in emergency departments that have used visual analog scales to measure interventions that decrease pain have used a mean difference of 0.9 as “clinically meaningful.” 7,8 The 0.74 difference observed in this study does not rise to that level. However, one in three patients in the gel group marked 0 on the pain scale, indicating that they had no pain, compared with only one in 10 in the water group.

 

 

We believe that the higher proportion of women experiencing no pain and the mean difference of 0.74 on the pain scale (both statistically significant), combined with the lack of risk associated with the use of a water-based lubricant, makes this a clinically useful practice changer.

CHALLENGES TO IMPLEMENTATION

There aren’t any

Other than clinical inertia, we see no challenges to the implementation of this recommendation.

References

1. Hill DA, Lamvu G. Effect of lubricating gel
on patient comfort during vaginal specu-
lum examination. Obstet Gynecol. 2012;119 (2 pt 1):227-231.

2. Harmanli O, Jones KA. Using lubricant for speculum insertion. Obstet Gynecol. 2010; 116:415-417.

3. Hoyo C, Yarnall KSH, Skinner CS, et al. Pain predicts non-adherence to pap smear screening among middle-aged African American women. Prev Med. 2005;41:439-445.

4. Amies AE, Miller L, Lee S, et al. The effect of vaginal speculum lubrication on the rate of unsatisfactory cervical cytology diagnosis. Obstet Gynecol. 2002;100:889-892.

5. Griffith WF, Stuart GS, Gluck KL, et al. Vaginal speculum lubrication and its effects on cervical cytology and microbiology. Contraception. 2005;72:60-64.

6. Evantash E. Lubricant use during Pap test collection. Bedford, Mass: Hologic, Inc; 2009. www.thinprep.com/hcp/specimen_collection/common_questions.html. Ac-
cessed October 9, 2013.

7. Bijur PE, Silver W, Gallagher EJ. Reliability of the visual analog scale for measurement of acute pain. Acad Emerg Med. 2001;8:1153-1157.

8. Kelly AM. Does the clinically significant difference in visual analog scale pain scores vary with gender, age, or cause of pain? Acad Emerg Med. 1998;5:1086-1090.

Acknowledgement

The PURLs Surveillance System is supported in part by Grant Number UL 1RR 024999 from the National Center for Research Resources, a Clinical Translational Science Award to the University of Chicago. The content is solely the responsibility of the authors and does not necessarily represent the official views of the National Center for Research Resources or the National Institutes of Health.

Copyright © 2013. The Family Physicians Inquiries Network. All rights reserved.

Reprinted with permission from the Family Physicians Inquiries Network and The Journal of Family Practice. 2013;61(9):E1-E3.

Author and Disclosure Information

Nicole VandenBerg, MD, Shailendra Prasad, MBBS, MPH

Issue
Clinician Reviews - 23(11)
Publications
Topics
Page Number
28
Legacy Keywords
Speculum, exam, comfort, gel, lubrication, pelvic exam, pap smear, annual,
Sections
Author and Disclosure Information

Nicole VandenBerg, MD, Shailendra Prasad, MBBS, MPH

Author and Disclosure Information

Nicole VandenBerg, MD, Shailendra Prasad, MBBS, MPH

Applying a small amount of gel to the blades of a speculum before insertion won’t alter test results—but will reduce the patient’s discomfort.
Applying a small amount of gel to the blades of a speculum before insertion won’t alter test results—but will reduce the patient’s discomfort.

PRACTICE CHANGER

Put lubricating gel, not water, on the speculum every time you do a pelvic exam.1

STRENGTH OF ­RECOMMENDATION

B: Based on one good-quality, randomized controlled trial.

Hill DA, Lamvu G. Effect of lubricating gel on patient comfort during vaginal speculum examination. Obstet Gynecol. 2012;119 (2 pt 1):
227-231.

ILLUSTRATIVE CASE

A 24-year-old woman presents for an annual exam, including Pap smear and testing for sexually transmitted diseases. She tells you how painful her previous speculum exam was and how worried she is about having another. To ease her discomfort, should you apply lubricating gel or water to the speculum before vaginal insertion?

Physiology teaches us that vaginal entry requires lubrication. But traditional teaching has held that lubricating gel on a speculum can interfere with the results of a Pap smear and chlamydia tesing.2 Pelvic exams performed without lubricating gel on the speculum can cause significant discomfort—possibly bad enough to prevent some women from undergoing the recommended screening tests.3

Until now, we’ve only ­evaluated gel’s impact on test results

Studies comparing lubricating gel and water have conclusively shown that a small amount of gel, used on the outside of the speculum blades, does not interfere with either Pap testing or detection of Chlamydia trachomatis.4,5 One liquid-based cytology manufacturer, however, discourages the use of lubricants with “carbomers” or “carbopol polymers,” but states that water-based lubricants have not been shown to interfere with Pap smear results.6 No studies have evaluated lubricants from a patient perspective—until now.

STUDY SUMMARY

Lubricating gel eases discomfort

The study by Hill and Lamvu1 was a six-month, single-blind, randomized trial of women ages 18 to 50 who sought care at an Orlando obstetrics and gynecology department for conditions requiring vaginal speculum examination.1 The study excluded women who might have an altered perception of pain during speculum insertion: those who were menopausal, pregnant, or within six weeks’ postpartum; had dyspareunia, vaginitis, vulvar pain, or vulvar lesions; were undergoing a procedure; or had never had vaginal intercourse. Women who were not fluent in English were excluded, as well.

The study included 120 women who underwent computer-generated randomization into two groups with no marked differences in demographics. A single examiner performed all the speculum exams, using a standard protocol with a medium-size Graves speculum. The examiner applied 0.3 mL water-based lubricating gel to the speculum before insertion for the women in one group, and used 3 mL water for the other.

Immediately after the speculum was inserted and opened—before the examiner attempted to visualize the cervix—patients were given a visual analog scale and told to indicate the level of pain with insertion, using a scale of 0 (no pain) to 10 (the worst pain imaginable). The gel group had lower pain scores for speculum insertion compared with the water group (1.41 ± 1.55 vs 2.15 ± 1.93), a statistically significant difference of 0.74. Twenty of the 59 patients in the gel group (33.9%) rated their pain as 0, compared with six of 60 (10%) in the water group.

Although pain, rather than sampling quality, was the primary outcome of the study, the authors also reported that all of the women who underwent Pap screening (73) had adequate cytology.

WHAT’S NEW?

It’s time for lubrication to become standard practice

This trial is the first to study speculum lubrication from this patient-oriented outcome and to show that women experience less pain when lubricating gel, rather than water, is applied to the speculum. This knowledge, combined with previous studies showing that a small amount of water-based lubricating gel does not interfere with liquid-based cytology or chlamydia test results, should make the use of lubricating gel standard practice when performing speculum examinations.

CAVEATS

We see no downside

The exclusion criteria of this study were meant to eliminate women who had an altered perception of pain that could skew study results. Yet those who met the exclusion criteria may also benefit from a pelvic exam with gel lubrication. We see no harm in trying a small amount of lubricant when examining them, as well.

In addition, the study did not compare various types and sizes of specula. However, we see no reason why the benefit of a gel lubricant would be limited to the type of speculum used by the examiner.

Studies in emergency departments that have used visual analog scales to measure interventions that decrease pain have used a mean difference of 0.9 as “clinically meaningful.” 7,8 The 0.74 difference observed in this study does not rise to that level. However, one in three patients in the gel group marked 0 on the pain scale, indicating that they had no pain, compared with only one in 10 in the water group.

 

 

We believe that the higher proportion of women experiencing no pain and the mean difference of 0.74 on the pain scale (both statistically significant), combined with the lack of risk associated with the use of a water-based lubricant, makes this a clinically useful practice changer.

CHALLENGES TO IMPLEMENTATION

There aren’t any

Other than clinical inertia, we see no challenges to the implementation of this recommendation.

References

1. Hill DA, Lamvu G. Effect of lubricating gel
on patient comfort during vaginal specu-
lum examination. Obstet Gynecol. 2012;119 (2 pt 1):227-231.

2. Harmanli O, Jones KA. Using lubricant for speculum insertion. Obstet Gynecol. 2010; 116:415-417.

3. Hoyo C, Yarnall KSH, Skinner CS, et al. Pain predicts non-adherence to pap smear screening among middle-aged African American women. Prev Med. 2005;41:439-445.

4. Amies AE, Miller L, Lee S, et al. The effect of vaginal speculum lubrication on the rate of unsatisfactory cervical cytology diagnosis. Obstet Gynecol. 2002;100:889-892.

5. Griffith WF, Stuart GS, Gluck KL, et al. Vaginal speculum lubrication and its effects on cervical cytology and microbiology. Contraception. 2005;72:60-64.

6. Evantash E. Lubricant use during Pap test collection. Bedford, Mass: Hologic, Inc; 2009. www.thinprep.com/hcp/specimen_collection/common_questions.html. Ac-
cessed October 9, 2013.

7. Bijur PE, Silver W, Gallagher EJ. Reliability of the visual analog scale for measurement of acute pain. Acad Emerg Med. 2001;8:1153-1157.

8. Kelly AM. Does the clinically significant difference in visual analog scale pain scores vary with gender, age, or cause of pain? Acad Emerg Med. 1998;5:1086-1090.

Acknowledgement

The PURLs Surveillance System is supported in part by Grant Number UL 1RR 024999 from the National Center for Research Resources, a Clinical Translational Science Award to the University of Chicago. The content is solely the responsibility of the authors and does not necessarily represent the official views of the National Center for Research Resources or the National Institutes of Health.

Copyright © 2013. The Family Physicians Inquiries Network. All rights reserved.

Reprinted with permission from the Family Physicians Inquiries Network and The Journal of Family Practice. 2013;61(9):E1-E3.

PRACTICE CHANGER

Put lubricating gel, not water, on the speculum every time you do a pelvic exam.1

STRENGTH OF ­RECOMMENDATION

B: Based on one good-quality, randomized controlled trial.

Hill DA, Lamvu G. Effect of lubricating gel on patient comfort during vaginal speculum examination. Obstet Gynecol. 2012;119 (2 pt 1):
227-231.

ILLUSTRATIVE CASE

A 24-year-old woman presents for an annual exam, including Pap smear and testing for sexually transmitted diseases. She tells you how painful her previous speculum exam was and how worried she is about having another. To ease her discomfort, should you apply lubricating gel or water to the speculum before vaginal insertion?

Physiology teaches us that vaginal entry requires lubrication. But traditional teaching has held that lubricating gel on a speculum can interfere with the results of a Pap smear and chlamydia tesing.2 Pelvic exams performed without lubricating gel on the speculum can cause significant discomfort—possibly bad enough to prevent some women from undergoing the recommended screening tests.3

Until now, we’ve only ­evaluated gel’s impact on test results

Studies comparing lubricating gel and water have conclusively shown that a small amount of gel, used on the outside of the speculum blades, does not interfere with either Pap testing or detection of Chlamydia trachomatis.4,5 One liquid-based cytology manufacturer, however, discourages the use of lubricants with “carbomers” or “carbopol polymers,” but states that water-based lubricants have not been shown to interfere with Pap smear results.6 No studies have evaluated lubricants from a patient perspective—until now.

STUDY SUMMARY

Lubricating gel eases discomfort

The study by Hill and Lamvu1 was a six-month, single-blind, randomized trial of women ages 18 to 50 who sought care at an Orlando obstetrics and gynecology department for conditions requiring vaginal speculum examination.1 The study excluded women who might have an altered perception of pain during speculum insertion: those who were menopausal, pregnant, or within six weeks’ postpartum; had dyspareunia, vaginitis, vulvar pain, or vulvar lesions; were undergoing a procedure; or had never had vaginal intercourse. Women who were not fluent in English were excluded, as well.

The study included 120 women who underwent computer-generated randomization into two groups with no marked differences in demographics. A single examiner performed all the speculum exams, using a standard protocol with a medium-size Graves speculum. The examiner applied 0.3 mL water-based lubricating gel to the speculum before insertion for the women in one group, and used 3 mL water for the other.

Immediately after the speculum was inserted and opened—before the examiner attempted to visualize the cervix—patients were given a visual analog scale and told to indicate the level of pain with insertion, using a scale of 0 (no pain) to 10 (the worst pain imaginable). The gel group had lower pain scores for speculum insertion compared with the water group (1.41 ± 1.55 vs 2.15 ± 1.93), a statistically significant difference of 0.74. Twenty of the 59 patients in the gel group (33.9%) rated their pain as 0, compared with six of 60 (10%) in the water group.

Although pain, rather than sampling quality, was the primary outcome of the study, the authors also reported that all of the women who underwent Pap screening (73) had adequate cytology.

WHAT’S NEW?

It’s time for lubrication to become standard practice

This trial is the first to study speculum lubrication from this patient-oriented outcome and to show that women experience less pain when lubricating gel, rather than water, is applied to the speculum. This knowledge, combined with previous studies showing that a small amount of water-based lubricating gel does not interfere with liquid-based cytology or chlamydia test results, should make the use of lubricating gel standard practice when performing speculum examinations.

CAVEATS

We see no downside

The exclusion criteria of this study were meant to eliminate women who had an altered perception of pain that could skew study results. Yet those who met the exclusion criteria may also benefit from a pelvic exam with gel lubrication. We see no harm in trying a small amount of lubricant when examining them, as well.

In addition, the study did not compare various types and sizes of specula. However, we see no reason why the benefit of a gel lubricant would be limited to the type of speculum used by the examiner.

Studies in emergency departments that have used visual analog scales to measure interventions that decrease pain have used a mean difference of 0.9 as “clinically meaningful.” 7,8 The 0.74 difference observed in this study does not rise to that level. However, one in three patients in the gel group marked 0 on the pain scale, indicating that they had no pain, compared with only one in 10 in the water group.

 

 

We believe that the higher proportion of women experiencing no pain and the mean difference of 0.74 on the pain scale (both statistically significant), combined with the lack of risk associated with the use of a water-based lubricant, makes this a clinically useful practice changer.

CHALLENGES TO IMPLEMENTATION

There aren’t any

Other than clinical inertia, we see no challenges to the implementation of this recommendation.

References

1. Hill DA, Lamvu G. Effect of lubricating gel
on patient comfort during vaginal specu-
lum examination. Obstet Gynecol. 2012;119 (2 pt 1):227-231.

2. Harmanli O, Jones KA. Using lubricant for speculum insertion. Obstet Gynecol. 2010; 116:415-417.

3. Hoyo C, Yarnall KSH, Skinner CS, et al. Pain predicts non-adherence to pap smear screening among middle-aged African American women. Prev Med. 2005;41:439-445.

4. Amies AE, Miller L, Lee S, et al. The effect of vaginal speculum lubrication on the rate of unsatisfactory cervical cytology diagnosis. Obstet Gynecol. 2002;100:889-892.

5. Griffith WF, Stuart GS, Gluck KL, et al. Vaginal speculum lubrication and its effects on cervical cytology and microbiology. Contraception. 2005;72:60-64.

6. Evantash E. Lubricant use during Pap test collection. Bedford, Mass: Hologic, Inc; 2009. www.thinprep.com/hcp/specimen_collection/common_questions.html. Ac-
cessed October 9, 2013.

7. Bijur PE, Silver W, Gallagher EJ. Reliability of the visual analog scale for measurement of acute pain. Acad Emerg Med. 2001;8:1153-1157.

8. Kelly AM. Does the clinically significant difference in visual analog scale pain scores vary with gender, age, or cause of pain? Acad Emerg Med. 1998;5:1086-1090.

Acknowledgement

The PURLs Surveillance System is supported in part by Grant Number UL 1RR 024999 from the National Center for Research Resources, a Clinical Translational Science Award to the University of Chicago. The content is solely the responsibility of the authors and does not necessarily represent the official views of the National Center for Research Resources or the National Institutes of Health.

Copyright © 2013. The Family Physicians Inquiries Network. All rights reserved.

Reprinted with permission from the Family Physicians Inquiries Network and The Journal of Family Practice. 2013;61(9):E1-E3.

Issue
Clinician Reviews - 23(11)
Issue
Clinician Reviews - 23(11)
Page Number
28
Page Number
28
Publications
Publications
Topics
Article Type
Display Headline
Easing the Discomfort of a Speculum Exam
Display Headline
Easing the Discomfort of a Speculum Exam
Legacy Keywords
Speculum, exam, comfort, gel, lubrication, pelvic exam, pap smear, annual,
Legacy Keywords
Speculum, exam, comfort, gel, lubrication, pelvic exam, pap smear, annual,
Sections
Article Source

PURLs Copyright

Inside the Article

Is self-swabbing for STIs a good idea?

Article Type
Changed
Mon, 01/14/2019 - 13:44
Display Headline
Is self-swabbing for STIs a good idea?
Practice changer

Ask women who are at risk for sexually transmitted infections (STIs) to self-swab for chlamydia and gonorrhea testing; self-collection of vulvovaginal swabs with nucleic acid amplification testing (NAAT) has excellent sensitivity in women with and without symptoms.1,2

Strength of recommendation

B: Based on prospective diagnostic cohort study.

Schoeman SA, Stewart CM, Booth RA, et al. Assessment of best single sample for finding chlamydia in women with and without symptoms: a diagnostic test study. BMJ. 2012;345:e8013.

Stewart CM, Schoeman SA, Booth RA, et al. Assessment of self taken swabs versus clinician taken swab cultures for diagnosing gonorrhoea in women: single centre, diagnostic accuracy study. BMJ. 2012;345:e8107.

 

Illustrative case

An 18-year-old woman comes to your office requesting testing for STIs. She has no symptoms. What is the best way to collect samples for chlamydia and gonorrhea testing?

Despite public health efforts, chlamydia and gonorrhea remain significant health problems, with more than 1.4 million cases of chlamydia and 321,849 cases of gonorrhea reported in the United States in 2011.3 Both can have devastating effects on reproduction, even in women who are asymptomatic.

Annual testing is recommended for women at risk
According to the Centers for Disease Control and Prevention (CDC), most reported cases of chlamydia (70%) and gonorrhea (62%) occur in men and women between the ages of 15 and 24 years.3 Both the CDC and the US Preventive Services Task Force recommend annual chlamydia screening for all sexually active women younger than 25, and for older women with risk factors, including having multiple sex partners and living in communities with a high burden of disease.4,5 Annual gonorrhea screening is recommended for sexually active women with risk factors, as well.4,5

How best to test? A number of unknowns
NAAT is the most sensitive test for detection of chlamydia and gonorrhea, but other questions about how best to screen for STIs remain.1,6 It has not been clear whether self-collected vulvovaginal swabs are equivalent to clinician-collected urethral or endocervical swabs for the detection of gonorrhea, or whether NAAT testing of the self-collected swabs or culture of the clinician-collected swabs is a more sensitive test for gonorrhea.

While some studies have found self-collected vulvovaginal samples to be as sensitive as clinician-collected endocervical samples for the diagnosis of chlamydia and gonorrhea, samples are still often collected by clinicians.7,8 Collecting endocervical swabs is uncomfortable for patients and time consuming for clinicians, and evidence suggests that patients prefer noninvasive sampling.9

STUDY SUMMARY: Self-collected samples are highly sensitive

This study was designed to compare the sensitivity and specificity of self-collected vulvovaginal swabs vs clinician-collected swabs for chlamydia and gonorrhea, both in asymptomatic women and women with symptoms of an STI. Test methods were also assessed for gonorrhea, comparing detection rates of self-swabs tested with NAAT vs the culture of clinician-collected urethral and endocervical samples.

The researchers evaluated a total of 3973 women, ages 16 to 59 years, who sought care at a single sexual health center in the United Kingdom. The average age was 25 years; 37% of the participants reported a prior STI, and 42% had at least one symptom suggestive of an STI. Exclusion criteria included having taken an antibiotic in the preceding 28 days and being unable or unwilling to take a vulvovaginal swab or undergo clinician examination and sample collection.

Self-collected vulvovaginal swabs are the sample of choice for both chlamydia and gonorrhea testing in women, regardless of whether patients have symptoms.The women performed vulvovaginal swabs for NAAT (Aptima Combo-2, Hologic GenProbe, San Diego, Calif) prior to a speculum exam; endocervical swab for both NAAT and culture and a urethral swab for culture were collected by the clinician. All the swabs sent for NAAT were tested for chlamydia and gonorrhea, and cultures were performed to detect gonorrhea.

Chlamydia: Vulvovaginal swabs have higher detection rates

Of the 3867 participants with complete results, 10.2% were infected with chlamydia. Self-collected vulvovaginal swabs were significantly more sensitive than endocervical swabs (97% vs 88%; P<.00001) and had equal specificity (99.9% vs 100%). In women with symptoms of an STI, the sensitivity was 97% vs 88% (P<.0008); in those with no symptoms, the sensitivity was 97% vs 89% (P<.002).

 

 

 

Gonorrhea: Self-collection, NAAT yield better results

Gonorrhea was found in 2.5% of the 3859 women with complete results for testing of this STI. Self-collected swabs and physician-collected swabs analyzed by NAAT both had excellent sensitivity (99% vs 96%; P=.375). But self-collected samples that underwent NAAT were significantly more sensitive than clinician-collected urethral and endocervical samples that were cultured (99% vs 81%; P<.001). The number needed to test by self-collection for NAAT (compared with clinician-collected culture) to detect one additional case of gonorrhea was 5.

In women with symptoms suggestive of infection, the NAAT assays—both physician- and self-collected—were equivalent and were more sensitive than gonorrhea culture (P=.004). In asymptomatic women, 1.8% of whom had gonorrhea, the vulvovaginal swab sent for NAAT was more sensitive than culture (98% vs 78%; P=0.008) and equivalent to the endocervical swab for NAAT (90%).

The bottom line: Self-collected vulvovaginal swabs are the sample of choice for both chlamydia and gonorrhea testing in women, regardless of whether they have symptoms. When a clinical examination is needed, either the clinician or the patient can collect a vulvovaginal swab.

WHAT'S NEW: Endocervical samples, cultures have lower detection rates

In this study, endocervical samples collected by the physician rather than self-collected vulvovaginal samples would have missed 9% (one in 11) of chlamydial infections in women with symptoms of an STI. Vulvovaginal swabs and endocervical swabs have equal sensitivity for the diagnosis of gonorrhea when NAAT is used, but culture would have missed one in 5 gonorrhea infections (in women with and without symptoms).

CAVEATS: NAAT is costly, and does not test for drug sensitivity

Although NAAT has replaced cell culture methodology as the gold standard for gonorrhea and chlamydia diagnosis, it is potentially costly if not readily available in your practice setting. What’s more, NAAT does not allow testing for antibiotic sensitivity, which is particularly relevant with increasing resistance of gonorrhea to multiple antibiotics. In addition, it’s unclear whether these results would apply to all NAAT assays or just the one used in this study.

These studies examine sensitivity and specificity of gonorrhea and chlamydia testing in a high-risk population—women who were seeking care in a sexual health center. Your patient population may be lower risk, which will lower the prevalence of STIs and lower the positive predictive value of NAAT. A positive NAAT test for an STI should be followed by a confirmation NAAT in low-risk populations.

CHALLENGES TO IMPLEMENTATION: Reconsidering the way we practice

Most family physicians are accustomed to performing a full examination on patients with a suspected STI, and changing the flow of the office visit may be difficult. And, to implement this practice changer properly, it would be necessary to provide patient instruction in self-collection technique.

Also, making this change could be costly if you do not have this particular NAAT available. Once implemented, however, self-collection with NAAT will likely save time and be more comfortable for your patients. It will also provide a higher sensitivity in detecting chlamydia infections and equal sensitivity in detecting gonorrhea compared with physician-collected NAAT testing.

ACKNOWLEDGEMENT
The PURLs surveillance system was supported in part by Grant Number UL1RR024999 from the National center for Research Resources, a clinical Translational science Award to the University of Chicago. The content is solely the responsibility of the authors and does not necessarily represent the official views of the National center for Research Resources or the National Institutes of Health.

Click here to view PURL METHODOLOGY 1

Click here to view PURL METHODOLOGY 2

References

1. Schoeman SA, Stewart CM, Booth RA, et al. Assessment of best single sample for finding chlamydia in women with and without symptoms: a diagnostic test study. BMJ. 2012;345:e8013.

2. Stewart CM, Schoeman SA, Booth RA, et al. Assessment of self taken swabs versus clinician taken swab for cultures for diagnosing gonorrhea in women: single centre, diagnostic accuracy study. BMJ. 2012;345:e8107.

3. Centers for Disease Control and Prevention. Fact sheet: STD trends in the United States, 2011 national data for chlamydia, gonorrhea, and syphilis. Atlanta, GA: US Department of Health and Human Services, Centers for Disease Control and Prevention, 2012. http://www.cdc.gov/std/stats11/trends-2011.pdf. Accessed July 17, 2013.

4. Centers for Disease Control and Prevention. Fact sheet: Incidence, prevalence, and cost of sexually transmitted infections in the United States. Atlanta, GA: US Department of Health and Human Services, Centers for Disease Control and Prevention, 2013. http://www.cdc.gov/std/stats/STI-Estimates-Fact-Sheet-Feb-2013.pdf. Accessed July 17, 2013.

5. USPSTF Recommendations for STI screening. United States Preventive
Services Task Force Web site. http://www.uspreventiveservicestaskforce.org/uspstf08/methods/stinfections.htm Accessed August 29, 2013.

6. 2010 STD Treatment Guidelines. Centers for Disease Control and Prevention Web site. http://www.cdc.gov/std/treatment/2010/default.htm. Accessed October 23, 2013.

7. Cook RL, Hutchison SL, Østergaard L, Braithwaite RS, Ness RB. Systematic review: Non-invasive testing for Chlamydia trachomatis and Neisseria gonorrhoeae. Ann Intern Med. 2005:142:914-925.

8. Moss S, Mallinson H. The contribution of APTIMA Combo 2 assay to the diagnosis of gonorrhoea in genitourinary medicine setting. Int J STD AIDS. 2007;18:551-554.

9. Chernesky MA, Hook EW 3rd, Martin DH, et al. Women find it easy and prefer to collect their own vaginal swabs to diagnose chlamydia trachomatis or Neisseria gonorrhaea infections. Sex Transm Dis. 2005;32:729-733.

Article PDF
Author and Disclosure Information

Cristy Page, MD, MPH
Anne Mounsey, MD
Kate Rowland, MD, MS

Department of Family Medicine, University of North Carolina at Chapel Hill (Drs. Page and Mounsey); Department of Family Medicine, University of Chicago (Dr. Rowland)

PURLs EDITOR
James Stevermer, MD, MPH
Department of Family Medicine, University of Missouri

Issue
The Journal of Family Practice - 62(11)
Publications
Topics
Page Number
651-653
Legacy Keywords
Cristy Page; MD; MPH; Anne Mounsey; MD; Kate Rowland; MD; MS; STIs; self-swabbing; gonorrhea; NAAT; nucleic acid amplification testing; sexually transmitted infections
Sections
Author and Disclosure Information

Cristy Page, MD, MPH
Anne Mounsey, MD
Kate Rowland, MD, MS

Department of Family Medicine, University of North Carolina at Chapel Hill (Drs. Page and Mounsey); Department of Family Medicine, University of Chicago (Dr. Rowland)

PURLs EDITOR
James Stevermer, MD, MPH
Department of Family Medicine, University of Missouri

Author and Disclosure Information

Cristy Page, MD, MPH
Anne Mounsey, MD
Kate Rowland, MD, MS

Department of Family Medicine, University of North Carolina at Chapel Hill (Drs. Page and Mounsey); Department of Family Medicine, University of Chicago (Dr. Rowland)

PURLs EDITOR
James Stevermer, MD, MPH
Department of Family Medicine, University of Missouri

Article PDF
Article PDF
Practice changer

Ask women who are at risk for sexually transmitted infections (STIs) to self-swab for chlamydia and gonorrhea testing; self-collection of vulvovaginal swabs with nucleic acid amplification testing (NAAT) has excellent sensitivity in women with and without symptoms.1,2

Strength of recommendation

B: Based on prospective diagnostic cohort study.

Schoeman SA, Stewart CM, Booth RA, et al. Assessment of best single sample for finding chlamydia in women with and without symptoms: a diagnostic test study. BMJ. 2012;345:e8013.

Stewart CM, Schoeman SA, Booth RA, et al. Assessment of self taken swabs versus clinician taken swab cultures for diagnosing gonorrhoea in women: single centre, diagnostic accuracy study. BMJ. 2012;345:e8107.

 

Illustrative case

An 18-year-old woman comes to your office requesting testing for STIs. She has no symptoms. What is the best way to collect samples for chlamydia and gonorrhea testing?

Despite public health efforts, chlamydia and gonorrhea remain significant health problems, with more than 1.4 million cases of chlamydia and 321,849 cases of gonorrhea reported in the United States in 2011.3 Both can have devastating effects on reproduction, even in women who are asymptomatic.

Annual testing is recommended for women at risk
According to the Centers for Disease Control and Prevention (CDC), most reported cases of chlamydia (70%) and gonorrhea (62%) occur in men and women between the ages of 15 and 24 years.3 Both the CDC and the US Preventive Services Task Force recommend annual chlamydia screening for all sexually active women younger than 25, and for older women with risk factors, including having multiple sex partners and living in communities with a high burden of disease.4,5 Annual gonorrhea screening is recommended for sexually active women with risk factors, as well.4,5

How best to test? A number of unknowns
NAAT is the most sensitive test for detection of chlamydia and gonorrhea, but other questions about how best to screen for STIs remain.1,6 It has not been clear whether self-collected vulvovaginal swabs are equivalent to clinician-collected urethral or endocervical swabs for the detection of gonorrhea, or whether NAAT testing of the self-collected swabs or culture of the clinician-collected swabs is a more sensitive test for gonorrhea.

While some studies have found self-collected vulvovaginal samples to be as sensitive as clinician-collected endocervical samples for the diagnosis of chlamydia and gonorrhea, samples are still often collected by clinicians.7,8 Collecting endocervical swabs is uncomfortable for patients and time consuming for clinicians, and evidence suggests that patients prefer noninvasive sampling.9

STUDY SUMMARY: Self-collected samples are highly sensitive

This study was designed to compare the sensitivity and specificity of self-collected vulvovaginal swabs vs clinician-collected swabs for chlamydia and gonorrhea, both in asymptomatic women and women with symptoms of an STI. Test methods were also assessed for gonorrhea, comparing detection rates of self-swabs tested with NAAT vs the culture of clinician-collected urethral and endocervical samples.

The researchers evaluated a total of 3973 women, ages 16 to 59 years, who sought care at a single sexual health center in the United Kingdom. The average age was 25 years; 37% of the participants reported a prior STI, and 42% had at least one symptom suggestive of an STI. Exclusion criteria included having taken an antibiotic in the preceding 28 days and being unable or unwilling to take a vulvovaginal swab or undergo clinician examination and sample collection.

Self-collected vulvovaginal swabs are the sample of choice for both chlamydia and gonorrhea testing in women, regardless of whether patients have symptoms.The women performed vulvovaginal swabs for NAAT (Aptima Combo-2, Hologic GenProbe, San Diego, Calif) prior to a speculum exam; endocervical swab for both NAAT and culture and a urethral swab for culture were collected by the clinician. All the swabs sent for NAAT were tested for chlamydia and gonorrhea, and cultures were performed to detect gonorrhea.

Chlamydia: Vulvovaginal swabs have higher detection rates

Of the 3867 participants with complete results, 10.2% were infected with chlamydia. Self-collected vulvovaginal swabs were significantly more sensitive than endocervical swabs (97% vs 88%; P<.00001) and had equal specificity (99.9% vs 100%). In women with symptoms of an STI, the sensitivity was 97% vs 88% (P<.0008); in those with no symptoms, the sensitivity was 97% vs 89% (P<.002).

 

 

 

Gonorrhea: Self-collection, NAAT yield better results

Gonorrhea was found in 2.5% of the 3859 women with complete results for testing of this STI. Self-collected swabs and physician-collected swabs analyzed by NAAT both had excellent sensitivity (99% vs 96%; P=.375). But self-collected samples that underwent NAAT were significantly more sensitive than clinician-collected urethral and endocervical samples that were cultured (99% vs 81%; P<.001). The number needed to test by self-collection for NAAT (compared with clinician-collected culture) to detect one additional case of gonorrhea was 5.

In women with symptoms suggestive of infection, the NAAT assays—both physician- and self-collected—were equivalent and were more sensitive than gonorrhea culture (P=.004). In asymptomatic women, 1.8% of whom had gonorrhea, the vulvovaginal swab sent for NAAT was more sensitive than culture (98% vs 78%; P=0.008) and equivalent to the endocervical swab for NAAT (90%).

The bottom line: Self-collected vulvovaginal swabs are the sample of choice for both chlamydia and gonorrhea testing in women, regardless of whether they have symptoms. When a clinical examination is needed, either the clinician or the patient can collect a vulvovaginal swab.

WHAT'S NEW: Endocervical samples, cultures have lower detection rates

In this study, endocervical samples collected by the physician rather than self-collected vulvovaginal samples would have missed 9% (one in 11) of chlamydial infections in women with symptoms of an STI. Vulvovaginal swabs and endocervical swabs have equal sensitivity for the diagnosis of gonorrhea when NAAT is used, but culture would have missed one in 5 gonorrhea infections (in women with and without symptoms).

CAVEATS: NAAT is costly, and does not test for drug sensitivity

Although NAAT has replaced cell culture methodology as the gold standard for gonorrhea and chlamydia diagnosis, it is potentially costly if not readily available in your practice setting. What’s more, NAAT does not allow testing for antibiotic sensitivity, which is particularly relevant with increasing resistance of gonorrhea to multiple antibiotics. In addition, it’s unclear whether these results would apply to all NAAT assays or just the one used in this study.

These studies examine sensitivity and specificity of gonorrhea and chlamydia testing in a high-risk population—women who were seeking care in a sexual health center. Your patient population may be lower risk, which will lower the prevalence of STIs and lower the positive predictive value of NAAT. A positive NAAT test for an STI should be followed by a confirmation NAAT in low-risk populations.

CHALLENGES TO IMPLEMENTATION: Reconsidering the way we practice

Most family physicians are accustomed to performing a full examination on patients with a suspected STI, and changing the flow of the office visit may be difficult. And, to implement this practice changer properly, it would be necessary to provide patient instruction in self-collection technique.

Also, making this change could be costly if you do not have this particular NAAT available. Once implemented, however, self-collection with NAAT will likely save time and be more comfortable for your patients. It will also provide a higher sensitivity in detecting chlamydia infections and equal sensitivity in detecting gonorrhea compared with physician-collected NAAT testing.

ACKNOWLEDGEMENT
The PURLs surveillance system was supported in part by Grant Number UL1RR024999 from the National center for Research Resources, a clinical Translational science Award to the University of Chicago. The content is solely the responsibility of the authors and does not necessarily represent the official views of the National center for Research Resources or the National Institutes of Health.

Click here to view PURL METHODOLOGY 1

Click here to view PURL METHODOLOGY 2

Practice changer

Ask women who are at risk for sexually transmitted infections (STIs) to self-swab for chlamydia and gonorrhea testing; self-collection of vulvovaginal swabs with nucleic acid amplification testing (NAAT) has excellent sensitivity in women with and without symptoms.1,2

Strength of recommendation

B: Based on prospective diagnostic cohort study.

Schoeman SA, Stewart CM, Booth RA, et al. Assessment of best single sample for finding chlamydia in women with and without symptoms: a diagnostic test study. BMJ. 2012;345:e8013.

Stewart CM, Schoeman SA, Booth RA, et al. Assessment of self taken swabs versus clinician taken swab cultures for diagnosing gonorrhoea in women: single centre, diagnostic accuracy study. BMJ. 2012;345:e8107.

 

Illustrative case

An 18-year-old woman comes to your office requesting testing for STIs. She has no symptoms. What is the best way to collect samples for chlamydia and gonorrhea testing?

Despite public health efforts, chlamydia and gonorrhea remain significant health problems, with more than 1.4 million cases of chlamydia and 321,849 cases of gonorrhea reported in the United States in 2011.3 Both can have devastating effects on reproduction, even in women who are asymptomatic.

Annual testing is recommended for women at risk
According to the Centers for Disease Control and Prevention (CDC), most reported cases of chlamydia (70%) and gonorrhea (62%) occur in men and women between the ages of 15 and 24 years.3 Both the CDC and the US Preventive Services Task Force recommend annual chlamydia screening for all sexually active women younger than 25, and for older women with risk factors, including having multiple sex partners and living in communities with a high burden of disease.4,5 Annual gonorrhea screening is recommended for sexually active women with risk factors, as well.4,5

How best to test? A number of unknowns
NAAT is the most sensitive test for detection of chlamydia and gonorrhea, but other questions about how best to screen for STIs remain.1,6 It has not been clear whether self-collected vulvovaginal swabs are equivalent to clinician-collected urethral or endocervical swabs for the detection of gonorrhea, or whether NAAT testing of the self-collected swabs or culture of the clinician-collected swabs is a more sensitive test for gonorrhea.

While some studies have found self-collected vulvovaginal samples to be as sensitive as clinician-collected endocervical samples for the diagnosis of chlamydia and gonorrhea, samples are still often collected by clinicians.7,8 Collecting endocervical swabs is uncomfortable for patients and time consuming for clinicians, and evidence suggests that patients prefer noninvasive sampling.9

STUDY SUMMARY: Self-collected samples are highly sensitive

This study was designed to compare the sensitivity and specificity of self-collected vulvovaginal swabs vs clinician-collected swabs for chlamydia and gonorrhea, both in asymptomatic women and women with symptoms of an STI. Test methods were also assessed for gonorrhea, comparing detection rates of self-swabs tested with NAAT vs the culture of clinician-collected urethral and endocervical samples.

The researchers evaluated a total of 3973 women, ages 16 to 59 years, who sought care at a single sexual health center in the United Kingdom. The average age was 25 years; 37% of the participants reported a prior STI, and 42% had at least one symptom suggestive of an STI. Exclusion criteria included having taken an antibiotic in the preceding 28 days and being unable or unwilling to take a vulvovaginal swab or undergo clinician examination and sample collection.

Self-collected vulvovaginal swabs are the sample of choice for both chlamydia and gonorrhea testing in women, regardless of whether patients have symptoms.The women performed vulvovaginal swabs for NAAT (Aptima Combo-2, Hologic GenProbe, San Diego, Calif) prior to a speculum exam; endocervical swab for both NAAT and culture and a urethral swab for culture were collected by the clinician. All the swabs sent for NAAT were tested for chlamydia and gonorrhea, and cultures were performed to detect gonorrhea.

Chlamydia: Vulvovaginal swabs have higher detection rates

Of the 3867 participants with complete results, 10.2% were infected with chlamydia. Self-collected vulvovaginal swabs were significantly more sensitive than endocervical swabs (97% vs 88%; P<.00001) and had equal specificity (99.9% vs 100%). In women with symptoms of an STI, the sensitivity was 97% vs 88% (P<.0008); in those with no symptoms, the sensitivity was 97% vs 89% (P<.002).

 

 

 

Gonorrhea: Self-collection, NAAT yield better results

Gonorrhea was found in 2.5% of the 3859 women with complete results for testing of this STI. Self-collected swabs and physician-collected swabs analyzed by NAAT both had excellent sensitivity (99% vs 96%; P=.375). But self-collected samples that underwent NAAT were significantly more sensitive than clinician-collected urethral and endocervical samples that were cultured (99% vs 81%; P<.001). The number needed to test by self-collection for NAAT (compared with clinician-collected culture) to detect one additional case of gonorrhea was 5.

In women with symptoms suggestive of infection, the NAAT assays—both physician- and self-collected—were equivalent and were more sensitive than gonorrhea culture (P=.004). In asymptomatic women, 1.8% of whom had gonorrhea, the vulvovaginal swab sent for NAAT was more sensitive than culture (98% vs 78%; P=0.008) and equivalent to the endocervical swab for NAAT (90%).

The bottom line: Self-collected vulvovaginal swabs are the sample of choice for both chlamydia and gonorrhea testing in women, regardless of whether they have symptoms. When a clinical examination is needed, either the clinician or the patient can collect a vulvovaginal swab.

WHAT'S NEW: Endocervical samples, cultures have lower detection rates

In this study, endocervical samples collected by the physician rather than self-collected vulvovaginal samples would have missed 9% (one in 11) of chlamydial infections in women with symptoms of an STI. Vulvovaginal swabs and endocervical swabs have equal sensitivity for the diagnosis of gonorrhea when NAAT is used, but culture would have missed one in 5 gonorrhea infections (in women with and without symptoms).

CAVEATS: NAAT is costly, and does not test for drug sensitivity

Although NAAT has replaced cell culture methodology as the gold standard for gonorrhea and chlamydia diagnosis, it is potentially costly if not readily available in your practice setting. What’s more, NAAT does not allow testing for antibiotic sensitivity, which is particularly relevant with increasing resistance of gonorrhea to multiple antibiotics. In addition, it’s unclear whether these results would apply to all NAAT assays or just the one used in this study.

These studies examine sensitivity and specificity of gonorrhea and chlamydia testing in a high-risk population—women who were seeking care in a sexual health center. Your patient population may be lower risk, which will lower the prevalence of STIs and lower the positive predictive value of NAAT. A positive NAAT test for an STI should be followed by a confirmation NAAT in low-risk populations.

CHALLENGES TO IMPLEMENTATION: Reconsidering the way we practice

Most family physicians are accustomed to performing a full examination on patients with a suspected STI, and changing the flow of the office visit may be difficult. And, to implement this practice changer properly, it would be necessary to provide patient instruction in self-collection technique.

Also, making this change could be costly if you do not have this particular NAAT available. Once implemented, however, self-collection with NAAT will likely save time and be more comfortable for your patients. It will also provide a higher sensitivity in detecting chlamydia infections and equal sensitivity in detecting gonorrhea compared with physician-collected NAAT testing.

ACKNOWLEDGEMENT
The PURLs surveillance system was supported in part by Grant Number UL1RR024999 from the National center for Research Resources, a clinical Translational science Award to the University of Chicago. The content is solely the responsibility of the authors and does not necessarily represent the official views of the National center for Research Resources or the National Institutes of Health.

Click here to view PURL METHODOLOGY 1

Click here to view PURL METHODOLOGY 2

References

1. Schoeman SA, Stewart CM, Booth RA, et al. Assessment of best single sample for finding chlamydia in women with and without symptoms: a diagnostic test study. BMJ. 2012;345:e8013.

2. Stewart CM, Schoeman SA, Booth RA, et al. Assessment of self taken swabs versus clinician taken swab for cultures for diagnosing gonorrhea in women: single centre, diagnostic accuracy study. BMJ. 2012;345:e8107.

3. Centers for Disease Control and Prevention. Fact sheet: STD trends in the United States, 2011 national data for chlamydia, gonorrhea, and syphilis. Atlanta, GA: US Department of Health and Human Services, Centers for Disease Control and Prevention, 2012. http://www.cdc.gov/std/stats11/trends-2011.pdf. Accessed July 17, 2013.

4. Centers for Disease Control and Prevention. Fact sheet: Incidence, prevalence, and cost of sexually transmitted infections in the United States. Atlanta, GA: US Department of Health and Human Services, Centers for Disease Control and Prevention, 2013. http://www.cdc.gov/std/stats/STI-Estimates-Fact-Sheet-Feb-2013.pdf. Accessed July 17, 2013.

5. USPSTF Recommendations for STI screening. United States Preventive
Services Task Force Web site. http://www.uspreventiveservicestaskforce.org/uspstf08/methods/stinfections.htm Accessed August 29, 2013.

6. 2010 STD Treatment Guidelines. Centers for Disease Control and Prevention Web site. http://www.cdc.gov/std/treatment/2010/default.htm. Accessed October 23, 2013.

7. Cook RL, Hutchison SL, Østergaard L, Braithwaite RS, Ness RB. Systematic review: Non-invasive testing for Chlamydia trachomatis and Neisseria gonorrhoeae. Ann Intern Med. 2005:142:914-925.

8. Moss S, Mallinson H. The contribution of APTIMA Combo 2 assay to the diagnosis of gonorrhoea in genitourinary medicine setting. Int J STD AIDS. 2007;18:551-554.

9. Chernesky MA, Hook EW 3rd, Martin DH, et al. Women find it easy and prefer to collect their own vaginal swabs to diagnose chlamydia trachomatis or Neisseria gonorrhaea infections. Sex Transm Dis. 2005;32:729-733.

References

1. Schoeman SA, Stewart CM, Booth RA, et al. Assessment of best single sample for finding chlamydia in women with and without symptoms: a diagnostic test study. BMJ. 2012;345:e8013.

2. Stewart CM, Schoeman SA, Booth RA, et al. Assessment of self taken swabs versus clinician taken swab for cultures for diagnosing gonorrhea in women: single centre, diagnostic accuracy study. BMJ. 2012;345:e8107.

3. Centers for Disease Control and Prevention. Fact sheet: STD trends in the United States, 2011 national data for chlamydia, gonorrhea, and syphilis. Atlanta, GA: US Department of Health and Human Services, Centers for Disease Control and Prevention, 2012. http://www.cdc.gov/std/stats11/trends-2011.pdf. Accessed July 17, 2013.

4. Centers for Disease Control and Prevention. Fact sheet: Incidence, prevalence, and cost of sexually transmitted infections in the United States. Atlanta, GA: US Department of Health and Human Services, Centers for Disease Control and Prevention, 2013. http://www.cdc.gov/std/stats/STI-Estimates-Fact-Sheet-Feb-2013.pdf. Accessed July 17, 2013.

5. USPSTF Recommendations for STI screening. United States Preventive
Services Task Force Web site. http://www.uspreventiveservicestaskforce.org/uspstf08/methods/stinfections.htm Accessed August 29, 2013.

6. 2010 STD Treatment Guidelines. Centers for Disease Control and Prevention Web site. http://www.cdc.gov/std/treatment/2010/default.htm. Accessed October 23, 2013.

7. Cook RL, Hutchison SL, Østergaard L, Braithwaite RS, Ness RB. Systematic review: Non-invasive testing for Chlamydia trachomatis and Neisseria gonorrhoeae. Ann Intern Med. 2005:142:914-925.

8. Moss S, Mallinson H. The contribution of APTIMA Combo 2 assay to the diagnosis of gonorrhoea in genitourinary medicine setting. Int J STD AIDS. 2007;18:551-554.

9. Chernesky MA, Hook EW 3rd, Martin DH, et al. Women find it easy and prefer to collect their own vaginal swabs to diagnose chlamydia trachomatis or Neisseria gonorrhaea infections. Sex Transm Dis. 2005;32:729-733.

Issue
The Journal of Family Practice - 62(11)
Issue
The Journal of Family Practice - 62(11)
Page Number
651-653
Page Number
651-653
Publications
Publications
Topics
Article Type
Display Headline
Is self-swabbing for STIs a good idea?
Display Headline
Is self-swabbing for STIs a good idea?
Legacy Keywords
Cristy Page; MD; MPH; Anne Mounsey; MD; Kate Rowland; MD; MS; STIs; self-swabbing; gonorrhea; NAAT; nucleic acid amplification testing; sexually transmitted infections
Legacy Keywords
Cristy Page; MD; MPH; Anne Mounsey; MD; Kate Rowland; MD; MS; STIs; self-swabbing; gonorrhea; NAAT; nucleic acid amplification testing; sexually transmitted infections
Sections
PURLs Copyright

Copyright © 2013 Family Physicians Inquiries Network. All rights reserved.

Disallow All Ads
Alternative CME
Article PDF Media

This Asthma Treatment Has a Lasting Side Effect in Children

Article Type
Changed
Tue, 12/13/2016 - 12:08
Display Headline
This Asthma Treatment Has a Lasting Side Effect in Children
A new study finds that when children with asthma use inhaled corticosteroids, the effect on growth may not be temporary, as once thought.

Practice changer

Before prescribing inhaled corticosteroids (ICS) for a child with asthma, tell the patient—and parents—that their use could lead to a small but permanent effect on adult height.1

STRENGTH OF ­RECOMMENDATIONS

B: Based on one prospective study.

Illustrative case

A 10-year-old boy is brought in by his father for asthma follow-up. The child uses an albuterol inhaler but has had increased coughing and wheezing recently. You are ready to step up his asthma therapy to include ICS. But the patient’s father questions this, noting that he recently read that steroids may reduce a child’s growth. How should you respond?

Inhaled corticosteroids are a mainstay in the treatment of asthma ranging from mild persistent to severe. Standards of care for asthma treatment involve a stepwise approach, with ICS added if symptoms are not controlled with short-acting beta antagonists alone.2 In addition, monotherapy with ICS is more effective for controlling symptoms than leukotriene inhibitors or other controller medications, while also decreasing hospitalizations and nocturnal awakenings and improving quality of life—with few side effects.3

What we know about ICS
and children’s growth

One adverse effect of ICS, however, is that of “decreased linear growth velocity”4—ie, slowing the rate at which children grow. Until recently, children were thought to “catch up” later in life, either by growing for a longer period of time than they would have had they not taken ICS or by growing at an increased velocity after ICS medications are discontinued.4-6

Study summary
The effect on growth is small, but long lasting

Kelly et al conducted a prospective observational cohort study that followed 943 participants in the Childhood Asthma Management Program (CAMP) in the years after the randomized controlled trial (RCT) ended.

A double-blind, placebo-controlled RCT, CAMP studied the linear growth of 1,041 children with mild to moderate persistent asthma who were divided into three treatment groups: One group received 200 g inhaled budesonide twice daily; a second group received 8 mg inhaled nedocromil twice daily; and a third group received placebo. Albuterol was used symptomatically by all three groups.7 The children ranged in age from 5 to 13 years at the start of the study; 98 patients—split evenly among the three treatment arms—were lost to follow-up.

During the four to six years of the CAMP trial, the budesonide group received a mean total of 636 mg ICS, whereas the nedocromil and placebo groups received an average of 88.5 and 109.4 mg ICS, respectively. After the RCT ended, all participants had standardized asthma treatment, receiving mean adjusted total doses of ICS of 381 mg for the budesonide group, 347.9 mg for the nedocromil group, and 355 mg for the placebo group.

Patients’ height was measured every six months for the next 4.5 years, and once or twice a year thereafter until they reached adult height (at a mean age of 24.9 ± 2.7 years).

ICS users were a half-inch shorter

Long-term ICS use was linked to a lower adult height. The adjusted mean height was 171.1 cm for the budesonide group versus 172.3 cm for those on placebo, a difference of 1.2 cm, or 0.47 inch; the mean adult height in the nedocromil group (172.1 cm) was similar to that of the placebo group (−0.2 cm).

The lower adult height in the ICS group did not vary significantly based on sex, age at trial entry, race, or duration of asthma prior to trial entry; however, dose was a key factor. A larger daily dose of budesonide—particularly in the first two years of the RCT—was associated with a lower adult height (about −0.1 cm for each g/kg in that two-year timeframe). This was consistent with results from studies that examined other types of ICS (beclomethasone, fluticasone, and mometasone).8-11

The study also showed that growth velocity was reduced in the first two years of assigned treatment with budesonide, and this was primarily among prepubertal participants. After the initial two-year slowing in growth rate, the children resumed growing at normal speeds.

What’s new?

Now we know:
Children don’t “catch up”

Retrospective studies have reported that children taking ICS for mild persistent to moderate asthma would have an initial slowing in growth velocity but then “catch up” by growing for a longer period of time.3-5 This is the first prospective study with good follow-up to show that ICS use affects long-term growth and adult height. While the effect is not large, some children and their families might be concerned about it.

Caveats
ICS use was atypical

The randomized controlled portion of the study used a prescribed dose of budesonide without regard to symptoms. This is not the typical pattern of ICS use. In addition, compliance with ICS varies significantly.12 Because the effect on adult height appears to be dose dependent, however, we think the results of this study are valid.

In addition, there was a placebo control group (and big differences in exposure to ICS) only for the duration of the RCT. During the subsequent study, all patients received equivalent doses of ICS. This means that the variation in mean adult height achieved can be primarily ascribed to participants’ use of ICS during the 4- to 6-year CAMP trial. Of note, the effect of ICS was greatest in prepubertal participants, so there may be a diminished effect as teens approach their final height.

 

 

The study did not look at the effect of ICS use in patients with severe asthma—the group most likely to use ICS. However, the benefits of ICS for those with severe asthma likely outweigh any negative effects on adult height.

Challenges to implementation
What to tell patients

The message we convey to patients (and parents) about ICS use is a nuanced one. We can stress that ICS remain very important in the treatment of asthma and, while it appears that their use causes a slight decrease in adult height, most children with persistent asthma benefit from ICS.

References

1. Kelly HW, Sternberg AL, Lescher R, et al; CAMP Research Group. Effect of inhaled glucocorticoids in childhood on adult height. 
N Engl J Med. 2012;367:904-912.

2. National Institutes of Health National Heart, Lung and Blood Institute. Expert Panel Report 3: Guidelines for the Diagnosis and Management of Asthma. National Asthma Education and Prevention Program, 2007. www.nhlbi.nih.gov/guidelines/asthma/asthgdln.pdf. Accessed August 15, 2013.

3. Chauhan BF, Ducharme FM. Anti-leukotriene agents compared to inhaled corticosteroids in the management of recurrent and/ or chronic asthma in adults and children. Cochrane Database Syst Rev. 2012;(5): CD002314.

4. Agertoft L, Pedersen S. Effect of long-term treatment with budesonide on adult height in children with asthma. N Engl J Med. 2000; 343:1064-1069.

5. Van Bever HP, Desager KN, Lijssens N, et al. Does treatment of asthmatic children with inhaled corticosteroids affect their adult height? Pediatr Pulmonol. 1999;27:369-375.

6. Silverstein MD, Yunginger JW, Reed CE, et al. Attained adult height after childhood asthma: effect of glucocorticoid therapy. J Allergy Clin Immunol. 1997;99:466-474.

7. The Childhood Asthma Management Program Research Group. Long-term effects of budesonide or nedocromil in children with asthma. N Engl J Med. 2000;343:1054-1063.

8. Tinkelman DG, Reed CE, Nelson HS, et al. Aerosol beclomethasone dipropionate compared with theophylline as primary treatment of chronic, mild to moderately severe asthma in children. Pediatrics. 1993;92:64-77.

9. Verberne AA, Frost C, Roorda RJ, et al. One year treatment with salmeterol compared with beclomethasone in children with asthma. Am J Respir Crit Care Med. 1997;156:688-695.

10. Allen DB, Bronsky EA, LaForce CF, et al. Growth in asthmatic children treated with fluticasone propionate. J Pediatr. 1998;132: 472-477.

11. Skoner DP, Meltzer EO, Milgrom H, et al. Effects of inhaled mometasone furoate on growth velocity and adrenal function: a placebo-controlled trial in children 4-9 years old with mild persistent asthma. J Asthma. 2011;48:848-859.

12. Cochrane MG, Bala MV, Downs KE, et al. Inhaled corticosteroids for asthma therapy: patient compliance, devices, and inhalation technique. Chest. 2000;117:542-550.

ACKNOWLEDGEMENT

The PURLs Surveillance System is supported in part by Grant Number UL 1RR 024999 from the National Center for Research Resources, a Clinical Translational Science Award to the University of Chicago. The content is solely the responsibility of the authors and does not necessarily represent the official views of the National Center for Research Resources or the National Institutes of Health.

Copyright © 2013. The Family Physicians Inquiries Network. All rights reserved.

Reprinted with permission from the Family Physicians Inquiries Network and The Journal of Family Practice. 2013;62(9):500-502.

Author and Disclosure Information

Tanner Nissly, DO, Shailendra Prasad, MBBS, MPH

Issue
Clinician Reviews - 23(10)
Publications
Topics
Page Number
19-20
Legacy Keywords
Asthma, treatment, side-effects, inhaled corticosteroids, ICS, height, pediatrics, children
Sections
Author and Disclosure Information

Tanner Nissly, DO, Shailendra Prasad, MBBS, MPH

Author and Disclosure Information

Tanner Nissly, DO, Shailendra Prasad, MBBS, MPH

A new study finds that when children with asthma use inhaled corticosteroids, the effect on growth may not be temporary, as once thought.
A new study finds that when children with asthma use inhaled corticosteroids, the effect on growth may not be temporary, as once thought.

Practice changer

Before prescribing inhaled corticosteroids (ICS) for a child with asthma, tell the patient—and parents—that their use could lead to a small but permanent effect on adult height.1

STRENGTH OF ­RECOMMENDATIONS

B: Based on one prospective study.

Illustrative case

A 10-year-old boy is brought in by his father for asthma follow-up. The child uses an albuterol inhaler but has had increased coughing and wheezing recently. You are ready to step up his asthma therapy to include ICS. But the patient’s father questions this, noting that he recently read that steroids may reduce a child’s growth. How should you respond?

Inhaled corticosteroids are a mainstay in the treatment of asthma ranging from mild persistent to severe. Standards of care for asthma treatment involve a stepwise approach, with ICS added if symptoms are not controlled with short-acting beta antagonists alone.2 In addition, monotherapy with ICS is more effective for controlling symptoms than leukotriene inhibitors or other controller medications, while also decreasing hospitalizations and nocturnal awakenings and improving quality of life—with few side effects.3

What we know about ICS
and children’s growth

One adverse effect of ICS, however, is that of “decreased linear growth velocity”4—ie, slowing the rate at which children grow. Until recently, children were thought to “catch up” later in life, either by growing for a longer period of time than they would have had they not taken ICS or by growing at an increased velocity after ICS medications are discontinued.4-6

Study summary
The effect on growth is small, but long lasting

Kelly et al conducted a prospective observational cohort study that followed 943 participants in the Childhood Asthma Management Program (CAMP) in the years after the randomized controlled trial (RCT) ended.

A double-blind, placebo-controlled RCT, CAMP studied the linear growth of 1,041 children with mild to moderate persistent asthma who were divided into three treatment groups: One group received 200 g inhaled budesonide twice daily; a second group received 8 mg inhaled nedocromil twice daily; and a third group received placebo. Albuterol was used symptomatically by all three groups.7 The children ranged in age from 5 to 13 years at the start of the study; 98 patients—split evenly among the three treatment arms—were lost to follow-up.

During the four to six years of the CAMP trial, the budesonide group received a mean total of 636 mg ICS, whereas the nedocromil and placebo groups received an average of 88.5 and 109.4 mg ICS, respectively. After the RCT ended, all participants had standardized asthma treatment, receiving mean adjusted total doses of ICS of 381 mg for the budesonide group, 347.9 mg for the nedocromil group, and 355 mg for the placebo group.

Patients’ height was measured every six months for the next 4.5 years, and once or twice a year thereafter until they reached adult height (at a mean age of 24.9 ± 2.7 years).

ICS users were a half-inch shorter

Long-term ICS use was linked to a lower adult height. The adjusted mean height was 171.1 cm for the budesonide group versus 172.3 cm for those on placebo, a difference of 1.2 cm, or 0.47 inch; the mean adult height in the nedocromil group (172.1 cm) was similar to that of the placebo group (−0.2 cm).

The lower adult height in the ICS group did not vary significantly based on sex, age at trial entry, race, or duration of asthma prior to trial entry; however, dose was a key factor. A larger daily dose of budesonide—particularly in the first two years of the RCT—was associated with a lower adult height (about −0.1 cm for each g/kg in that two-year timeframe). This was consistent with results from studies that examined other types of ICS (beclomethasone, fluticasone, and mometasone).8-11

The study also showed that growth velocity was reduced in the first two years of assigned treatment with budesonide, and this was primarily among prepubertal participants. After the initial two-year slowing in growth rate, the children resumed growing at normal speeds.

What’s new?

Now we know:
Children don’t “catch up”

Retrospective studies have reported that children taking ICS for mild persistent to moderate asthma would have an initial slowing in growth velocity but then “catch up” by growing for a longer period of time.3-5 This is the first prospective study with good follow-up to show that ICS use affects long-term growth and adult height. While the effect is not large, some children and their families might be concerned about it.

Caveats
ICS use was atypical

The randomized controlled portion of the study used a prescribed dose of budesonide without regard to symptoms. This is not the typical pattern of ICS use. In addition, compliance with ICS varies significantly.12 Because the effect on adult height appears to be dose dependent, however, we think the results of this study are valid.

In addition, there was a placebo control group (and big differences in exposure to ICS) only for the duration of the RCT. During the subsequent study, all patients received equivalent doses of ICS. This means that the variation in mean adult height achieved can be primarily ascribed to participants’ use of ICS during the 4- to 6-year CAMP trial. Of note, the effect of ICS was greatest in prepubertal participants, so there may be a diminished effect as teens approach their final height.

 

 

The study did not look at the effect of ICS use in patients with severe asthma—the group most likely to use ICS. However, the benefits of ICS for those with severe asthma likely outweigh any negative effects on adult height.

Challenges to implementation
What to tell patients

The message we convey to patients (and parents) about ICS use is a nuanced one. We can stress that ICS remain very important in the treatment of asthma and, while it appears that their use causes a slight decrease in adult height, most children with persistent asthma benefit from ICS.

References

1. Kelly HW, Sternberg AL, Lescher R, et al; CAMP Research Group. Effect of inhaled glucocorticoids in childhood on adult height. 
N Engl J Med. 2012;367:904-912.

2. National Institutes of Health National Heart, Lung and Blood Institute. Expert Panel Report 3: Guidelines for the Diagnosis and Management of Asthma. National Asthma Education and Prevention Program, 2007. www.nhlbi.nih.gov/guidelines/asthma/asthgdln.pdf. Accessed August 15, 2013.

3. Chauhan BF, Ducharme FM. Anti-leukotriene agents compared to inhaled corticosteroids in the management of recurrent and/ or chronic asthma in adults and children. Cochrane Database Syst Rev. 2012;(5): CD002314.

4. Agertoft L, Pedersen S. Effect of long-term treatment with budesonide on adult height in children with asthma. N Engl J Med. 2000; 343:1064-1069.

5. Van Bever HP, Desager KN, Lijssens N, et al. Does treatment of asthmatic children with inhaled corticosteroids affect their adult height? Pediatr Pulmonol. 1999;27:369-375.

6. Silverstein MD, Yunginger JW, Reed CE, et al. Attained adult height after childhood asthma: effect of glucocorticoid therapy. J Allergy Clin Immunol. 1997;99:466-474.

7. The Childhood Asthma Management Program Research Group. Long-term effects of budesonide or nedocromil in children with asthma. N Engl J Med. 2000;343:1054-1063.

8. Tinkelman DG, Reed CE, Nelson HS, et al. Aerosol beclomethasone dipropionate compared with theophylline as primary treatment of chronic, mild to moderately severe asthma in children. Pediatrics. 1993;92:64-77.

9. Verberne AA, Frost C, Roorda RJ, et al. One year treatment with salmeterol compared with beclomethasone in children with asthma. Am J Respir Crit Care Med. 1997;156:688-695.

10. Allen DB, Bronsky EA, LaForce CF, et al. Growth in asthmatic children treated with fluticasone propionate. J Pediatr. 1998;132: 472-477.

11. Skoner DP, Meltzer EO, Milgrom H, et al. Effects of inhaled mometasone furoate on growth velocity and adrenal function: a placebo-controlled trial in children 4-9 years old with mild persistent asthma. J Asthma. 2011;48:848-859.

12. Cochrane MG, Bala MV, Downs KE, et al. Inhaled corticosteroids for asthma therapy: patient compliance, devices, and inhalation technique. Chest. 2000;117:542-550.

ACKNOWLEDGEMENT

The PURLs Surveillance System is supported in part by Grant Number UL 1RR 024999 from the National Center for Research Resources, a Clinical Translational Science Award to the University of Chicago. The content is solely the responsibility of the authors and does not necessarily represent the official views of the National Center for Research Resources or the National Institutes of Health.

Copyright © 2013. The Family Physicians Inquiries Network. All rights reserved.

Reprinted with permission from the Family Physicians Inquiries Network and The Journal of Family Practice. 2013;62(9):500-502.

Practice changer

Before prescribing inhaled corticosteroids (ICS) for a child with asthma, tell the patient—and parents—that their use could lead to a small but permanent effect on adult height.1

STRENGTH OF ­RECOMMENDATIONS

B: Based on one prospective study.

Illustrative case

A 10-year-old boy is brought in by his father for asthma follow-up. The child uses an albuterol inhaler but has had increased coughing and wheezing recently. You are ready to step up his asthma therapy to include ICS. But the patient’s father questions this, noting that he recently read that steroids may reduce a child’s growth. How should you respond?

Inhaled corticosteroids are a mainstay in the treatment of asthma ranging from mild persistent to severe. Standards of care for asthma treatment involve a stepwise approach, with ICS added if symptoms are not controlled with short-acting beta antagonists alone.2 In addition, monotherapy with ICS is more effective for controlling symptoms than leukotriene inhibitors or other controller medications, while also decreasing hospitalizations and nocturnal awakenings and improving quality of life—with few side effects.3

What we know about ICS
and children’s growth

One adverse effect of ICS, however, is that of “decreased linear growth velocity”4—ie, slowing the rate at which children grow. Until recently, children were thought to “catch up” later in life, either by growing for a longer period of time than they would have had they not taken ICS or by growing at an increased velocity after ICS medications are discontinued.4-6

Study summary
The effect on growth is small, but long lasting

Kelly et al conducted a prospective observational cohort study that followed 943 participants in the Childhood Asthma Management Program (CAMP) in the years after the randomized controlled trial (RCT) ended.

A double-blind, placebo-controlled RCT, CAMP studied the linear growth of 1,041 children with mild to moderate persistent asthma who were divided into three treatment groups: One group received 200 g inhaled budesonide twice daily; a second group received 8 mg inhaled nedocromil twice daily; and a third group received placebo. Albuterol was used symptomatically by all three groups.7 The children ranged in age from 5 to 13 years at the start of the study; 98 patients—split evenly among the three treatment arms—were lost to follow-up.

During the four to six years of the CAMP trial, the budesonide group received a mean total of 636 mg ICS, whereas the nedocromil and placebo groups received an average of 88.5 and 109.4 mg ICS, respectively. After the RCT ended, all participants had standardized asthma treatment, receiving mean adjusted total doses of ICS of 381 mg for the budesonide group, 347.9 mg for the nedocromil group, and 355 mg for the placebo group.

Patients’ height was measured every six months for the next 4.5 years, and once or twice a year thereafter until they reached adult height (at a mean age of 24.9 ± 2.7 years).

ICS users were a half-inch shorter

Long-term ICS use was linked to a lower adult height. The adjusted mean height was 171.1 cm for the budesonide group versus 172.3 cm for those on placebo, a difference of 1.2 cm, or 0.47 inch; the mean adult height in the nedocromil group (172.1 cm) was similar to that of the placebo group (−0.2 cm).

The lower adult height in the ICS group did not vary significantly based on sex, age at trial entry, race, or duration of asthma prior to trial entry; however, dose was a key factor. A larger daily dose of budesonide—particularly in the first two years of the RCT—was associated with a lower adult height (about −0.1 cm for each g/kg in that two-year timeframe). This was consistent with results from studies that examined other types of ICS (beclomethasone, fluticasone, and mometasone).8-11

The study also showed that growth velocity was reduced in the first two years of assigned treatment with budesonide, and this was primarily among prepubertal participants. After the initial two-year slowing in growth rate, the children resumed growing at normal speeds.

What’s new?

Now we know:
Children don’t “catch up”

Retrospective studies have reported that children taking ICS for mild persistent to moderate asthma would have an initial slowing in growth velocity but then “catch up” by growing for a longer period of time.3-5 This is the first prospective study with good follow-up to show that ICS use affects long-term growth and adult height. While the effect is not large, some children and their families might be concerned about it.

Caveats
ICS use was atypical

The randomized controlled portion of the study used a prescribed dose of budesonide without regard to symptoms. This is not the typical pattern of ICS use. In addition, compliance with ICS varies significantly.12 Because the effect on adult height appears to be dose dependent, however, we think the results of this study are valid.

In addition, there was a placebo control group (and big differences in exposure to ICS) only for the duration of the RCT. During the subsequent study, all patients received equivalent doses of ICS. This means that the variation in mean adult height achieved can be primarily ascribed to participants’ use of ICS during the 4- to 6-year CAMP trial. Of note, the effect of ICS was greatest in prepubertal participants, so there may be a diminished effect as teens approach their final height.

 

 

The study did not look at the effect of ICS use in patients with severe asthma—the group most likely to use ICS. However, the benefits of ICS for those with severe asthma likely outweigh any negative effects on adult height.

Challenges to implementation
What to tell patients

The message we convey to patients (and parents) about ICS use is a nuanced one. We can stress that ICS remain very important in the treatment of asthma and, while it appears that their use causes a slight decrease in adult height, most children with persistent asthma benefit from ICS.

References

1. Kelly HW, Sternberg AL, Lescher R, et al; CAMP Research Group. Effect of inhaled glucocorticoids in childhood on adult height. 
N Engl J Med. 2012;367:904-912.

2. National Institutes of Health National Heart, Lung and Blood Institute. Expert Panel Report 3: Guidelines for the Diagnosis and Management of Asthma. National Asthma Education and Prevention Program, 2007. www.nhlbi.nih.gov/guidelines/asthma/asthgdln.pdf. Accessed August 15, 2013.

3. Chauhan BF, Ducharme FM. Anti-leukotriene agents compared to inhaled corticosteroids in the management of recurrent and/ or chronic asthma in adults and children. Cochrane Database Syst Rev. 2012;(5): CD002314.

4. Agertoft L, Pedersen S. Effect of long-term treatment with budesonide on adult height in children with asthma. N Engl J Med. 2000; 343:1064-1069.

5. Van Bever HP, Desager KN, Lijssens N, et al. Does treatment of asthmatic children with inhaled corticosteroids affect their adult height? Pediatr Pulmonol. 1999;27:369-375.

6. Silverstein MD, Yunginger JW, Reed CE, et al. Attained adult height after childhood asthma: effect of glucocorticoid therapy. J Allergy Clin Immunol. 1997;99:466-474.

7. The Childhood Asthma Management Program Research Group. Long-term effects of budesonide or nedocromil in children with asthma. N Engl J Med. 2000;343:1054-1063.

8. Tinkelman DG, Reed CE, Nelson HS, et al. Aerosol beclomethasone dipropionate compared with theophylline as primary treatment of chronic, mild to moderately severe asthma in children. Pediatrics. 1993;92:64-77.

9. Verberne AA, Frost C, Roorda RJ, et al. One year treatment with salmeterol compared with beclomethasone in children with asthma. Am J Respir Crit Care Med. 1997;156:688-695.

10. Allen DB, Bronsky EA, LaForce CF, et al. Growth in asthmatic children treated with fluticasone propionate. J Pediatr. 1998;132: 472-477.

11. Skoner DP, Meltzer EO, Milgrom H, et al. Effects of inhaled mometasone furoate on growth velocity and adrenal function: a placebo-controlled trial in children 4-9 years old with mild persistent asthma. J Asthma. 2011;48:848-859.

12. Cochrane MG, Bala MV, Downs KE, et al. Inhaled corticosteroids for asthma therapy: patient compliance, devices, and inhalation technique. Chest. 2000;117:542-550.

ACKNOWLEDGEMENT

The PURLs Surveillance System is supported in part by Grant Number UL 1RR 024999 from the National Center for Research Resources, a Clinical Translational Science Award to the University of Chicago. The content is solely the responsibility of the authors and does not necessarily represent the official views of the National Center for Research Resources or the National Institutes of Health.

Copyright © 2013. The Family Physicians Inquiries Network. All rights reserved.

Reprinted with permission from the Family Physicians Inquiries Network and The Journal of Family Practice. 2013;62(9):500-502.

Issue
Clinician Reviews - 23(10)
Issue
Clinician Reviews - 23(10)
Page Number
19-20
Page Number
19-20
Publications
Publications
Topics
Article Type
Display Headline
This Asthma Treatment Has a Lasting Side Effect in Children
Display Headline
This Asthma Treatment Has a Lasting Side Effect in Children
Legacy Keywords
Asthma, treatment, side-effects, inhaled corticosteroids, ICS, height, pediatrics, children
Legacy Keywords
Asthma, treatment, side-effects, inhaled corticosteroids, ICS, height, pediatrics, children
Sections
Article Source

PURLs Copyright

Inside the Article

Ramipril for claudication?

Article Type
Changed
Mon, 01/14/2019 - 13:43
Display Headline
Ramipril for claudication?

Practice changer

Consider prescribing ramipril for patients who have intermittent claudication.1

Strength of recommendation

A: Based on a high-quality placebo-controlled randomized controlled trial (RCT) consistent with prior RCTs.

Ahimastos AA, Walker PJ, Askew C, et al. Effect of ramipril on walking times and quality of life among patients with peripheral artery disease and intermittent claudication: a randomized controlled trial. JAMA. 2013;309:453-460.

Illustrative case

A 63-year-old man presents with pain in both legs, which starts with activity and resolves with rest. He has a resting blood pressure of 135/77 mm Hg consistent with past measurements, and an ankle-brachial index (ABI) <0.90, which is consistent with peripheral artery disease (PAD). His daily medications are 81 mg aspirin, 25 mg hydrochlorothiazide, and 40 mg simvastatin. What additional agent could be added for his symptoms?

PAD, defined as an ABI <0.9, affects approximately 5% of Americans older than 40 years. About two-thirds of those with PAD are asymptomatic; the remaining third suffer from intermittent claudication (IC).2

Exercise and smoking cessation are effective at reducing IC symptoms, as well as the long-term cardiovascular event risk associated with PAD.3 But even with these lifestyle changes, patients with PAD are often troubled by persistent symptoms.

Few evidence-based treatments for IC

Compared with placebo, the antiplatelet agents indobufen and picotamide have been shown to improve pain-free walking distance (PFWD).4 So have cilostazol5 and naftidrofuryl,6 as well as lipid-lowering agents.7

In a pilot study of 40 patients, 10 mg ramipril was shown to improve pain-free walking time (PFWT) at 24 weeks by 227 seconds (95% confidence interval [CI]=175-278; P<.001). That represents a 164% increase from baseline, vs no change in PFWT at 24 weeks for the placebo group.8 A recent small (N=33), double-blinded RCT found similar improvements in maximum treadmill walking distance, PFWD, and patient-reported walking distance at 24 weeks with ramipril compared with placebo.9

In the HOPE study, a subsection of patients who were older than 55 years and had PAD were treated with a daily target dose of 10 mg ramipril for a mean of 4.5 years. Compared with placebo, ramipril reduced the primary outcome—cardiovascular mortality, myocardial infarction (MI), or stroke—by 25% (risk ratio=0.75; 95% CI, 0.61-0.92).10

In the study reported on here, Ahimastos et al took a closer look at ramipril.

STUDY SUMMARY: Patients on ramipril can walk longer pain free

The authors conducted a double-blind, randomized placebo-controlled trial evaluating the effectiveness of 10 mg/d ramipril for the improvement of maximum walk time (MWT) and PFWT in patients with PAD.1 Eligible patients had an ABI <0.9 in at least one leg and a history of IC in at least one leg, with stable claudication symptoms and a stable medical regimen for 6 months or more. Exclusion criteria included a resting blood pressure >160/100 mm Hg; use of ACE inhibitors, angiotensin II receptor blockers, potassium sparing diuretics, or potassium supplements in the past 6 months; serum creatinine >2.3 mg/dL; renal artery stenosis; previous coronary or lower extremity revascularization procedure; MI in the past 3 months; major surgery planned for the following year; critical limb ischemia; or any condition other than PAD limiting walking ability.

In total, 212 patients underwent randomization, and either took 10 mg/d ramipril or placebo for 24 weeks. The participants had similar baseline characteristics. Most were male (83.5%), with a mean age of 65.5 years; 33.5% were current smokers; 50% had hypertension; and 24.1% had type 2 diabetes.

Ramipril not only reduces cardiovascular mortality, MI, and stroke in patients with PAD, but its effective in improving patient-oriented outcomes such as duration of walking without developing intermittent claudication. Primary outcomes—PFWT and MWT—improved in the ramipril group. Compared with the placebo group, those in the ramipril group had a mean PFWT increase of 75 seconds (95% CI, 60-89; P<.001) and a 255-second increase in MWT (95% CI, 215-295; P<.001), a 52% and 107% increase from baseline, respectively. Most secondary measures (including the Walking Impairment Questionnaire median distance score, the speed score, and the stair-climbing score) also improved significantly, relative to the placebo group. However, ABI did not change significantly in either group.

WHAT'S NEW: Evidence that ramipril improves patient-oriented outcomes

Ramipril not only reduces cardiovascular mortality, MI, and stroke in patients with PAD,10 but is effective in improving patient-oriented outcomes such as duration of walking without developing IC.

CAVEATS: Would ramipril help less stable patients?

Inclusion criteria used by Ahimastos et al limit the generalizability of this study to patients with stable symptoms for 6 months or more. Similarly, because the study lasted for 24 weeks, it is not known whether ramipril’s benefits for patients with claudication would continue indefinitely. Also of note: The ABI did not improve in the treatment cohort at the end of this 24-week period, and the authors did not report objective outcomes such as revascularization or mortality.

 

 

CHALLENGES TO IMPLEMENTATION: Monitoring, adverse effects may present problems

Use of ACE inhibitors requires monitoring of renal function and serum potassium. In addition, ACE inhibitors can induce a chronic cough that often limits their use in those affected; 6.6% of the treatment group withdrew from this study due to persistent cough.1

ACKNOWLEDGEMENT
The PURLs surveillance system was supported in part by Grant Number UL1RR024999 from the National center for Research Resources, a clinical Translational science Award to the University of Chicago. The content is solely the responsibility of the authors and does not necessarily represent the official views of the National center for Research Resources or the National Institutes of Health.

References

1. Ahimastos AA, Walker PJ, Askew C, et al. Effect of ramipril on walking times and quality of life among patients with peripheral artery disease and intermittent claudication: a randomized controlled trial. JAMA. 2013;309:453-460.

2. Centers for Disease Control and Prevention. Lower extremity disease among persons aged ≥40 years with and without diabe- tes—United States, 1999-2002. MMWR Morb Mortal Wkly Rep. 2005;54:1158-1160.

3. Watson L, Ellis B, Leng GC. Exercise for intermittent claudication. Cochrane Database Syst Rev. 2008;(4):CD000990.

4. Wong PF, Chong LY, Mikhailidis DP, et al. Antiplatelet agents for intermittent claudication. Cochrane Database Syst Rev. 2011;(11):CD001272.

5. Robless P, Mikhailidis DP, Stansby GP. Cilostazol for peripheral arterial disease. Cochrane Database Syst Rev. 2008;(1):CD003748.

6. de Backer TL, Vander Stichele R, Lehert P, et al. Naftidrofuryl for intermittent claudication. Cochrane Database Syst Rev. 2012;(12):CD001368.

7. Aung PP, Maxwell HG, Jepson RG, et al. Lipid-lowering for pe- ripheral arterial disease of the lower limb. Cochrane Database Syst Rev. 2007;(4):CD000123.

8. Ahimastos AA, Lawler A, Reid CM, et al. Brief communication: ramipril markedly improves walking ability in patients with pe- ripheral arterial disease: a randomized trial. Ann Intern Med. 2006;144:660-664.

9. Shahin Y, Cockcroft JR, Chetter IC. Randomized clinical trial of angiotensin-converting enzyme inhibitor, ramipril, in patients with intermittent claudication. Br J Surg. 2013;100:1154-1163.

10. Ostergren J, Sleight P, Dagenais G, et al. Impact of ramipril in pa- tients with evidence of clinical or subclinical peripheral arterial disease. Eur Heart J. 2004;25:17-24.

Article PDF
Author and Disclosure Information

Luke A. Stephens, MD, MSPH; Nina Rogers, MD; James J. Stevermer, MD, MSPH
Department of Family and Community Medicine, University of Missouri, Columbia (Drs. Stephens and Stevermer); Department of Family Medicine, University of Chicago (Dr. Rogers)

PURLs EDITOR
Anne Mounsey, MD
Department of Family Medicine, University of North Carolina at Chapel Hill

Issue
The Journal of Family Practice - 62(10)
Publications
Topics
Page Number
579-580
Legacy Keywords
Luke A. Stephens; MD; MSPH; Nina Rogers; MD; James J. Stevermer; MD; MSPH; PURLs; ramipril; claudication; ACE inhibitor; ankle-brachial index; ABI; peripheral artery disease; PAD; pain-free walking distance; PFWD
Sections
Author and Disclosure Information

Luke A. Stephens, MD, MSPH; Nina Rogers, MD; James J. Stevermer, MD, MSPH
Department of Family and Community Medicine, University of Missouri, Columbia (Drs. Stephens and Stevermer); Department of Family Medicine, University of Chicago (Dr. Rogers)

PURLs EDITOR
Anne Mounsey, MD
Department of Family Medicine, University of North Carolina at Chapel Hill

Author and Disclosure Information

Luke A. Stephens, MD, MSPH; Nina Rogers, MD; James J. Stevermer, MD, MSPH
Department of Family and Community Medicine, University of Missouri, Columbia (Drs. Stephens and Stevermer); Department of Family Medicine, University of Chicago (Dr. Rogers)

PURLs EDITOR
Anne Mounsey, MD
Department of Family Medicine, University of North Carolina at Chapel Hill

Article PDF
Article PDF

Practice changer

Consider prescribing ramipril for patients who have intermittent claudication.1

Strength of recommendation

A: Based on a high-quality placebo-controlled randomized controlled trial (RCT) consistent with prior RCTs.

Ahimastos AA, Walker PJ, Askew C, et al. Effect of ramipril on walking times and quality of life among patients with peripheral artery disease and intermittent claudication: a randomized controlled trial. JAMA. 2013;309:453-460.

Illustrative case

A 63-year-old man presents with pain in both legs, which starts with activity and resolves with rest. He has a resting blood pressure of 135/77 mm Hg consistent with past measurements, and an ankle-brachial index (ABI) <0.90, which is consistent with peripheral artery disease (PAD). His daily medications are 81 mg aspirin, 25 mg hydrochlorothiazide, and 40 mg simvastatin. What additional agent could be added for his symptoms?

PAD, defined as an ABI <0.9, affects approximately 5% of Americans older than 40 years. About two-thirds of those with PAD are asymptomatic; the remaining third suffer from intermittent claudication (IC).2

Exercise and smoking cessation are effective at reducing IC symptoms, as well as the long-term cardiovascular event risk associated with PAD.3 But even with these lifestyle changes, patients with PAD are often troubled by persistent symptoms.

Few evidence-based treatments for IC

Compared with placebo, the antiplatelet agents indobufen and picotamide have been shown to improve pain-free walking distance (PFWD).4 So have cilostazol5 and naftidrofuryl,6 as well as lipid-lowering agents.7

In a pilot study of 40 patients, 10 mg ramipril was shown to improve pain-free walking time (PFWT) at 24 weeks by 227 seconds (95% confidence interval [CI]=175-278; P<.001). That represents a 164% increase from baseline, vs no change in PFWT at 24 weeks for the placebo group.8 A recent small (N=33), double-blinded RCT found similar improvements in maximum treadmill walking distance, PFWD, and patient-reported walking distance at 24 weeks with ramipril compared with placebo.9

In the HOPE study, a subsection of patients who were older than 55 years and had PAD were treated with a daily target dose of 10 mg ramipril for a mean of 4.5 years. Compared with placebo, ramipril reduced the primary outcome—cardiovascular mortality, myocardial infarction (MI), or stroke—by 25% (risk ratio=0.75; 95% CI, 0.61-0.92).10

In the study reported on here, Ahimastos et al took a closer look at ramipril.

STUDY SUMMARY: Patients on ramipril can walk longer pain free

The authors conducted a double-blind, randomized placebo-controlled trial evaluating the effectiveness of 10 mg/d ramipril for the improvement of maximum walk time (MWT) and PFWT in patients with PAD.1 Eligible patients had an ABI <0.9 in at least one leg and a history of IC in at least one leg, with stable claudication symptoms and a stable medical regimen for 6 months or more. Exclusion criteria included a resting blood pressure >160/100 mm Hg; use of ACE inhibitors, angiotensin II receptor blockers, potassium sparing diuretics, or potassium supplements in the past 6 months; serum creatinine >2.3 mg/dL; renal artery stenosis; previous coronary or lower extremity revascularization procedure; MI in the past 3 months; major surgery planned for the following year; critical limb ischemia; or any condition other than PAD limiting walking ability.

In total, 212 patients underwent randomization, and either took 10 mg/d ramipril or placebo for 24 weeks. The participants had similar baseline characteristics. Most were male (83.5%), with a mean age of 65.5 years; 33.5% were current smokers; 50% had hypertension; and 24.1% had type 2 diabetes.

Ramipril not only reduces cardiovascular mortality, MI, and stroke in patients with PAD, but its effective in improving patient-oriented outcomes such as duration of walking without developing intermittent claudication. Primary outcomes—PFWT and MWT—improved in the ramipril group. Compared with the placebo group, those in the ramipril group had a mean PFWT increase of 75 seconds (95% CI, 60-89; P<.001) and a 255-second increase in MWT (95% CI, 215-295; P<.001), a 52% and 107% increase from baseline, respectively. Most secondary measures (including the Walking Impairment Questionnaire median distance score, the speed score, and the stair-climbing score) also improved significantly, relative to the placebo group. However, ABI did not change significantly in either group.

WHAT'S NEW: Evidence that ramipril improves patient-oriented outcomes

Ramipril not only reduces cardiovascular mortality, MI, and stroke in patients with PAD,10 but is effective in improving patient-oriented outcomes such as duration of walking without developing IC.

CAVEATS: Would ramipril help less stable patients?

Inclusion criteria used by Ahimastos et al limit the generalizability of this study to patients with stable symptoms for 6 months or more. Similarly, because the study lasted for 24 weeks, it is not known whether ramipril’s benefits for patients with claudication would continue indefinitely. Also of note: The ABI did not improve in the treatment cohort at the end of this 24-week period, and the authors did not report objective outcomes such as revascularization or mortality.

 

 

CHALLENGES TO IMPLEMENTATION: Monitoring, adverse effects may present problems

Use of ACE inhibitors requires monitoring of renal function and serum potassium. In addition, ACE inhibitors can induce a chronic cough that often limits their use in those affected; 6.6% of the treatment group withdrew from this study due to persistent cough.1

ACKNOWLEDGEMENT
The PURLs surveillance system was supported in part by Grant Number UL1RR024999 from the National center for Research Resources, a clinical Translational science Award to the University of Chicago. The content is solely the responsibility of the authors and does not necessarily represent the official views of the National center for Research Resources or the National Institutes of Health.

Practice changer

Consider prescribing ramipril for patients who have intermittent claudication.1

Strength of recommendation

A: Based on a high-quality placebo-controlled randomized controlled trial (RCT) consistent with prior RCTs.

Ahimastos AA, Walker PJ, Askew C, et al. Effect of ramipril on walking times and quality of life among patients with peripheral artery disease and intermittent claudication: a randomized controlled trial. JAMA. 2013;309:453-460.

Illustrative case

A 63-year-old man presents with pain in both legs, which starts with activity and resolves with rest. He has a resting blood pressure of 135/77 mm Hg consistent with past measurements, and an ankle-brachial index (ABI) <0.90, which is consistent with peripheral artery disease (PAD). His daily medications are 81 mg aspirin, 25 mg hydrochlorothiazide, and 40 mg simvastatin. What additional agent could be added for his symptoms?

PAD, defined as an ABI <0.9, affects approximately 5% of Americans older than 40 years. About two-thirds of those with PAD are asymptomatic; the remaining third suffer from intermittent claudication (IC).2

Exercise and smoking cessation are effective at reducing IC symptoms, as well as the long-term cardiovascular event risk associated with PAD.3 But even with these lifestyle changes, patients with PAD are often troubled by persistent symptoms.

Few evidence-based treatments for IC

Compared with placebo, the antiplatelet agents indobufen and picotamide have been shown to improve pain-free walking distance (PFWD).4 So have cilostazol5 and naftidrofuryl,6 as well as lipid-lowering agents.7

In a pilot study of 40 patients, 10 mg ramipril was shown to improve pain-free walking time (PFWT) at 24 weeks by 227 seconds (95% confidence interval [CI]=175-278; P<.001). That represents a 164% increase from baseline, vs no change in PFWT at 24 weeks for the placebo group.8 A recent small (N=33), double-blinded RCT found similar improvements in maximum treadmill walking distance, PFWD, and patient-reported walking distance at 24 weeks with ramipril compared with placebo.9

In the HOPE study, a subsection of patients who were older than 55 years and had PAD were treated with a daily target dose of 10 mg ramipril for a mean of 4.5 years. Compared with placebo, ramipril reduced the primary outcome—cardiovascular mortality, myocardial infarction (MI), or stroke—by 25% (risk ratio=0.75; 95% CI, 0.61-0.92).10

In the study reported on here, Ahimastos et al took a closer look at ramipril.

STUDY SUMMARY: Patients on ramipril can walk longer pain free

The authors conducted a double-blind, randomized placebo-controlled trial evaluating the effectiveness of 10 mg/d ramipril for the improvement of maximum walk time (MWT) and PFWT in patients with PAD.1 Eligible patients had an ABI <0.9 in at least one leg and a history of IC in at least one leg, with stable claudication symptoms and a stable medical regimen for 6 months or more. Exclusion criteria included a resting blood pressure >160/100 mm Hg; use of ACE inhibitors, angiotensin II receptor blockers, potassium sparing diuretics, or potassium supplements in the past 6 months; serum creatinine >2.3 mg/dL; renal artery stenosis; previous coronary or lower extremity revascularization procedure; MI in the past 3 months; major surgery planned for the following year; critical limb ischemia; or any condition other than PAD limiting walking ability.

In total, 212 patients underwent randomization, and either took 10 mg/d ramipril or placebo for 24 weeks. The participants had similar baseline characteristics. Most were male (83.5%), with a mean age of 65.5 years; 33.5% were current smokers; 50% had hypertension; and 24.1% had type 2 diabetes.

Ramipril not only reduces cardiovascular mortality, MI, and stroke in patients with PAD, but its effective in improving patient-oriented outcomes such as duration of walking without developing intermittent claudication. Primary outcomes—PFWT and MWT—improved in the ramipril group. Compared with the placebo group, those in the ramipril group had a mean PFWT increase of 75 seconds (95% CI, 60-89; P<.001) and a 255-second increase in MWT (95% CI, 215-295; P<.001), a 52% and 107% increase from baseline, respectively. Most secondary measures (including the Walking Impairment Questionnaire median distance score, the speed score, and the stair-climbing score) also improved significantly, relative to the placebo group. However, ABI did not change significantly in either group.

WHAT'S NEW: Evidence that ramipril improves patient-oriented outcomes

Ramipril not only reduces cardiovascular mortality, MI, and stroke in patients with PAD,10 but is effective in improving patient-oriented outcomes such as duration of walking without developing IC.

CAVEATS: Would ramipril help less stable patients?

Inclusion criteria used by Ahimastos et al limit the generalizability of this study to patients with stable symptoms for 6 months or more. Similarly, because the study lasted for 24 weeks, it is not known whether ramipril’s benefits for patients with claudication would continue indefinitely. Also of note: The ABI did not improve in the treatment cohort at the end of this 24-week period, and the authors did not report objective outcomes such as revascularization or mortality.

 

 

CHALLENGES TO IMPLEMENTATION: Monitoring, adverse effects may present problems

Use of ACE inhibitors requires monitoring of renal function and serum potassium. In addition, ACE inhibitors can induce a chronic cough that often limits their use in those affected; 6.6% of the treatment group withdrew from this study due to persistent cough.1

ACKNOWLEDGEMENT
The PURLs surveillance system was supported in part by Grant Number UL1RR024999 from the National center for Research Resources, a clinical Translational science Award to the University of Chicago. The content is solely the responsibility of the authors and does not necessarily represent the official views of the National center for Research Resources or the National Institutes of Health.

References

1. Ahimastos AA, Walker PJ, Askew C, et al. Effect of ramipril on walking times and quality of life among patients with peripheral artery disease and intermittent claudication: a randomized controlled trial. JAMA. 2013;309:453-460.

2. Centers for Disease Control and Prevention. Lower extremity disease among persons aged ≥40 years with and without diabe- tes—United States, 1999-2002. MMWR Morb Mortal Wkly Rep. 2005;54:1158-1160.

3. Watson L, Ellis B, Leng GC. Exercise for intermittent claudication. Cochrane Database Syst Rev. 2008;(4):CD000990.

4. Wong PF, Chong LY, Mikhailidis DP, et al. Antiplatelet agents for intermittent claudication. Cochrane Database Syst Rev. 2011;(11):CD001272.

5. Robless P, Mikhailidis DP, Stansby GP. Cilostazol for peripheral arterial disease. Cochrane Database Syst Rev. 2008;(1):CD003748.

6. de Backer TL, Vander Stichele R, Lehert P, et al. Naftidrofuryl for intermittent claudication. Cochrane Database Syst Rev. 2012;(12):CD001368.

7. Aung PP, Maxwell HG, Jepson RG, et al. Lipid-lowering for pe- ripheral arterial disease of the lower limb. Cochrane Database Syst Rev. 2007;(4):CD000123.

8. Ahimastos AA, Lawler A, Reid CM, et al. Brief communication: ramipril markedly improves walking ability in patients with pe- ripheral arterial disease: a randomized trial. Ann Intern Med. 2006;144:660-664.

9. Shahin Y, Cockcroft JR, Chetter IC. Randomized clinical trial of angiotensin-converting enzyme inhibitor, ramipril, in patients with intermittent claudication. Br J Surg. 2013;100:1154-1163.

10. Ostergren J, Sleight P, Dagenais G, et al. Impact of ramipril in pa- tients with evidence of clinical or subclinical peripheral arterial disease. Eur Heart J. 2004;25:17-24.

References

1. Ahimastos AA, Walker PJ, Askew C, et al. Effect of ramipril on walking times and quality of life among patients with peripheral artery disease and intermittent claudication: a randomized controlled trial. JAMA. 2013;309:453-460.

2. Centers for Disease Control and Prevention. Lower extremity disease among persons aged ≥40 years with and without diabe- tes—United States, 1999-2002. MMWR Morb Mortal Wkly Rep. 2005;54:1158-1160.

3. Watson L, Ellis B, Leng GC. Exercise for intermittent claudication. Cochrane Database Syst Rev. 2008;(4):CD000990.

4. Wong PF, Chong LY, Mikhailidis DP, et al. Antiplatelet agents for intermittent claudication. Cochrane Database Syst Rev. 2011;(11):CD001272.

5. Robless P, Mikhailidis DP, Stansby GP. Cilostazol for peripheral arterial disease. Cochrane Database Syst Rev. 2008;(1):CD003748.

6. de Backer TL, Vander Stichele R, Lehert P, et al. Naftidrofuryl for intermittent claudication. Cochrane Database Syst Rev. 2012;(12):CD001368.

7. Aung PP, Maxwell HG, Jepson RG, et al. Lipid-lowering for pe- ripheral arterial disease of the lower limb. Cochrane Database Syst Rev. 2007;(4):CD000123.

8. Ahimastos AA, Lawler A, Reid CM, et al. Brief communication: ramipril markedly improves walking ability in patients with pe- ripheral arterial disease: a randomized trial. Ann Intern Med. 2006;144:660-664.

9. Shahin Y, Cockcroft JR, Chetter IC. Randomized clinical trial of angiotensin-converting enzyme inhibitor, ramipril, in patients with intermittent claudication. Br J Surg. 2013;100:1154-1163.

10. Ostergren J, Sleight P, Dagenais G, et al. Impact of ramipril in pa- tients with evidence of clinical or subclinical peripheral arterial disease. Eur Heart J. 2004;25:17-24.

Issue
The Journal of Family Practice - 62(10)
Issue
The Journal of Family Practice - 62(10)
Page Number
579-580
Page Number
579-580
Publications
Publications
Topics
Article Type
Display Headline
Ramipril for claudication?
Display Headline
Ramipril for claudication?
Legacy Keywords
Luke A. Stephens; MD; MSPH; Nina Rogers; MD; James J. Stevermer; MD; MSPH; PURLs; ramipril; claudication; ACE inhibitor; ankle-brachial index; ABI; peripheral artery disease; PAD; pain-free walking distance; PFWD
Legacy Keywords
Luke A. Stephens; MD; MSPH; Nina Rogers; MD; James J. Stevermer; MD; MSPH; PURLs; ramipril; claudication; ACE inhibitor; ankle-brachial index; ABI; peripheral artery disease; PAD; pain-free walking distance; PFWD
Sections
Article Source

PURLs Copyright

Copyright © 2013 Family Physicians Inquiries Network. All rights reserved.

Inside the Article

Article PDF Media

Should You Still Recommend Omega-3 Supplements?

Article Type
Changed
Tue, 12/13/2016 - 12:08
Display Headline
Should You Still Recommend Omega-3 Supplements?
Probably not, a new meta-analysis suggests. Read the details here.

PRACTICE CHANGER
Stop recommending omega-3 fatty acid supplements for cardiovascular protection. They have no significant impact on all-cause mortality, acute myocardial infarction (MI), sudden death, or stroke.1

STRENGTH OF RECOMMENDATION
A: Based on a meta-analysis of randomized controlled trials (RCTs).

ILLUSTRATIVE CASE
A 59-year-old patient who had an MI three years ago is taking an ACE inhibitor, a statin, and a -blocker. He asks you whether he should also take omega-3 fatty acid supplements to further decrease his risk for heart disease. What should you tell him?

Coronary artery disease (CAD) kills more than 500,000 Americans every year,2 and medical and dietary therapies for primary and secondary cardiovascular protection are paramount. Omega-3 polyunsaturated fatty acid (PUFA) supplementation is one such therapy.

Omega-3 PUFAs are precursors to certain prostaglandins that decrease the proinflammatory state in patients with CAD. They also lower triglyceride levels and produce an anti-arrhythmic effect by promoting electrical stability.

But do PUFA supplements ­provide cardioprotection?

The American Heart Association’s Nutrition Committee recommends either omega-3 PUFA supplementation with 250 to 500 mg of eicosapentaenoic acid (EPA) and docosahexaenoic acid (DHA) per day or two servings of oily fish per week for both primary and secondary prevention of CAD.3 The European Society of Cardiology also encourages increased consumption of oily fish.4

These recommendations are based on primary and secondary prevention studies performed between 1989 and 2007, which found a 15% to 29% decrease in all-cause mortality and nonfatal cardiovascular events associated with regular intake of omega-3 fatty acids.5-7 The systematic review and meta-analysis detailed below revisited the effect of omega-3 supplementation on major cardiovascular outcomes.1

STUDY SUMMARY
Omega-3 supplements don’t lower cardiovascular risk

This meta-analysis included 20 RCTs with a total of 68,680 patients. The median age was 68, with a range of 49 to 70.  Thirteen of the studies evaluated omega-3 PUFAs for secondary prevention of cardiovascular outcomes, four assessed both primary and secondary prevention, and three looked at outcomes in patients with implantable cardioverter defibrillators. All lasted longer than one year, and most were high quality, with a low risk for bias.

The median treatment duration was two years, with a maximum of 6.2 years. The mean omega-3 PUFA dose evaluated in the studies was 1.5 g/d, with the exception of two studies in which patients received omega-3 PUFAs through dietary sources. Twelve studies used a dose of 1 g or more per day. Half of the included trials were performed during the period when statins were routinely prescribed for cardiovascular risk modification (1998 or later).

Outcomes included all-cause mortality (17 studies), cardiac death (13 studies), sudden death (seven studies), MI (13 studies), and stroke (nine studies).

This meta-analysis found trends toward a decrease in all-cause mortality, cardiac death, sudden death, and MI in patients taking omega-3 PUFAs, but no statistically significant association between any of the outcomes and omega-3 PUFA supplementation. The relative risk for all-cause mortality was 0.96. Prespecified subgroup analysis found no association between treatment effect and omega-3 fatty acid dose.

Are dietary sources of omega-3s more effective?

In the two trials that involved dietary supplementation with omega-3 PUFAs, the results for all-cause mortality and cardiac death were conflicting, with one showing an increase in all-cause mortality and cardiac death and the other showing a decrease in both outcomes compared with the control group.

No harmful effects of omega-3 PUFAs were found in either the supplement- or diet-based studies.

WHAT’S NEW
More evidence of little benefit

The meta-analysis by Rizos et al is the most up-to-date, comprehensive look at the value of omega-3 fatty acids for primary and secondary prevention of cardiovascular events. It differs from previous reviews in that most of the included studies were well-done RCTs.

In addition, the studies were performed in both primary and secondary cardiovascular disease prevention settings and involved different forms of omega-3 PUFA supplementation, including dietary sources and supplements. The trials were predominantly larger than those included in previous systematic reviews, as well. The baseline risk for cardiovascular disease in the newer studies (seven of the 20 RCTs were completed after 2007) may be different from that of previous studies because of increased use of certain medications, such as statins.

In recent years, other studies of omega-3 PUFAs have had similar results. A meta-analysis of 14 RCTs found that omega-3 PUFA supplementation offered no benefit for the secondary prevention of cardiovascular disease.8 The FORWARD trial—published earlier this year—showed that omega-3 PUFAs did not decrease the recurrence of atrial fibrillation in patients with a history of confirmed paroxysmal atrial fibrillation.9 And an earlier (2006) analysis of RCTs and cohort studies found no benefit from omega-3 fatty acids for primary prevention of cardiovascular disease or cancer.10

 

 

CAVEATS
No significant help, and no harm

While this meta-analysis found no statistically significant benefits from omega-3 PUFAs, there is no evidence of harm from PUFA intake, whether from dietary sources or supplements. There is no need to tell patients who wish to take omega-3 supplements not to do so. But we should not promote their use for the sole purpose of cardiovascular disease prevention.

CHALLENGES TO IMPLEMENTATION
Changing minds won’t be easy

Despite recent findings indicating that omega-3 PUFAs provide little primary or secondary protection against cardiovascular events, advertising from supplement manufacturers may make it hard to change patients’ minds. Because diets and supplements containing these fatty acids do not cause apparent harm, patients and clinicians may decide that a small potential benefit is worth the expense.

References

1. Rizos E, Ntzani E, Bika E, et al. Association between omega-3 fatty acid supplementation and risk of major cardiovascular disease events: a systemic review and meta-analysis. JAMA. 2012; 308:1024-1033.

2. CDC. FastStats. Deaths and mortality. 2010 [updated April 5, 2013]. www.cdc.gov/nchs/fastats/deaths.htm. Accessed July 21, 2013.

3. Kris-Etherton PM, Harris WS, Appel LJ, et al. Fish consumption, fish oil, omega-3 fatty acids and cardiovascular disease. Circulation. 2002;106:2747-2757.

4. Van de Werf F, Bax J, Betriu A, et al. Management of acute myocardial infarction in patients presenting with persistent ST-segment elevation: the Task Force on the Management of ST-segment Elevation Acute Myocardial Infarction of the European Society of Cardiology. Eur Heart J. 2008;29:2909-2945.

5. Artham SM, Lavie CJ, Milani RV, et al. Fish oil in primary and secondary cardiovascular prevention. Ochsner J. 2008;8: 49-60.

6. Gruppo Italiano per lo Studio della Sopravvivenza nell’Infarto miocardico. Dietary supplementation with n-3 polyunsaturated fatty acids and vitamin E after myocardial infarction: results of GISSI-Prevenzione trial. Lancet. 1999; 354:447-455.

7. Yokoyama M, Origasa H, Matsuzaki M, et al. Effects of eicosapentaenoic acid on major coronary events in hypercholesteroleamic patients (JELIS): a randomized open-label, blinded endpoint analysis. Lancet. 2007;369:1090-1098.

8. Kwak SM, Myung SK, Lee YJ, et al. Efficacy of omega-3 fatty acid supplements (eicosapentaenoic acid and docosahexaenoic acid) in the secondary prevention of cardiovascular disease: a meta-analysis of randomized, double-blind, placebo-controlled trials. Arch Intern Med. 2012;172: 686-694.

9. Macchia A, Grancelli H, Varini S, et al. Omega-3 fatty acids for the prevention of recurrent symptomatic atrial fibrillation: results of the FORWARD (Randomized Trial to Assess Efficacy of PUFA for the Maintenance of Sinus Rhythm in Persistent Atrial Fibrillation) trial. J Am Coll Cardiol. 2013;61:463-468.

10. Hooper L, Thompson RL, Harrison RA, et al. Risks and benefits of omega 3 fats for mortality, cardiovascular disease, and cancer: systemic review. BMJ. 2006;332:752-760.

Acknowledgement

The PURLs Surveillance System was supported in part by Grant Number UL1RR024999 from the National Center for Research Resources, a Clinical Translational Science Award to the University of Chicago. The content is solely the responsibility of the authors and does not necessarily represent the official views of the National Center for Research Resources or the National Institutes of Health.

Copyright © 2013. The Family Physicians Inquiries Network. All rights reserved.

Reprinted with permission from the Family Physicians Inquiries Network and The Journal of Family Practice. 2013;62(7):372-374.

Author and Disclosure Information

Julie Monaco, MD, Anne Mounsey, MD, Jennifer Bello Kottenstette, MD

Issue
Clinician Reviews - 23(9)
Publications
Topics
Page Number
20
Legacy Keywords
purls, omega-3, supplements, cardiovascular, protection, practice change, fatty acid
Sections
Author and Disclosure Information

Julie Monaco, MD, Anne Mounsey, MD, Jennifer Bello Kottenstette, MD

Author and Disclosure Information

Julie Monaco, MD, Anne Mounsey, MD, Jennifer Bello Kottenstette, MD

Probably not, a new meta-analysis suggests. Read the details here.
Probably not, a new meta-analysis suggests. Read the details here.

PRACTICE CHANGER
Stop recommending omega-3 fatty acid supplements for cardiovascular protection. They have no significant impact on all-cause mortality, acute myocardial infarction (MI), sudden death, or stroke.1

STRENGTH OF RECOMMENDATION
A: Based on a meta-analysis of randomized controlled trials (RCTs).

ILLUSTRATIVE CASE
A 59-year-old patient who had an MI three years ago is taking an ACE inhibitor, a statin, and a -blocker. He asks you whether he should also take omega-3 fatty acid supplements to further decrease his risk for heart disease. What should you tell him?

Coronary artery disease (CAD) kills more than 500,000 Americans every year,2 and medical and dietary therapies for primary and secondary cardiovascular protection are paramount. Omega-3 polyunsaturated fatty acid (PUFA) supplementation is one such therapy.

Omega-3 PUFAs are precursors to certain prostaglandins that decrease the proinflammatory state in patients with CAD. They also lower triglyceride levels and produce an anti-arrhythmic effect by promoting electrical stability.

But do PUFA supplements ­provide cardioprotection?

The American Heart Association’s Nutrition Committee recommends either omega-3 PUFA supplementation with 250 to 500 mg of eicosapentaenoic acid (EPA) and docosahexaenoic acid (DHA) per day or two servings of oily fish per week for both primary and secondary prevention of CAD.3 The European Society of Cardiology also encourages increased consumption of oily fish.4

These recommendations are based on primary and secondary prevention studies performed between 1989 and 2007, which found a 15% to 29% decrease in all-cause mortality and nonfatal cardiovascular events associated with regular intake of omega-3 fatty acids.5-7 The systematic review and meta-analysis detailed below revisited the effect of omega-3 supplementation on major cardiovascular outcomes.1

STUDY SUMMARY
Omega-3 supplements don’t lower cardiovascular risk

This meta-analysis included 20 RCTs with a total of 68,680 patients. The median age was 68, with a range of 49 to 70.  Thirteen of the studies evaluated omega-3 PUFAs for secondary prevention of cardiovascular outcomes, four assessed both primary and secondary prevention, and three looked at outcomes in patients with implantable cardioverter defibrillators. All lasted longer than one year, and most were high quality, with a low risk for bias.

The median treatment duration was two years, with a maximum of 6.2 years. The mean omega-3 PUFA dose evaluated in the studies was 1.5 g/d, with the exception of two studies in which patients received omega-3 PUFAs through dietary sources. Twelve studies used a dose of 1 g or more per day. Half of the included trials were performed during the period when statins were routinely prescribed for cardiovascular risk modification (1998 or later).

Outcomes included all-cause mortality (17 studies), cardiac death (13 studies), sudden death (seven studies), MI (13 studies), and stroke (nine studies).

This meta-analysis found trends toward a decrease in all-cause mortality, cardiac death, sudden death, and MI in patients taking omega-3 PUFAs, but no statistically significant association between any of the outcomes and omega-3 PUFA supplementation. The relative risk for all-cause mortality was 0.96. Prespecified subgroup analysis found no association between treatment effect and omega-3 fatty acid dose.

Are dietary sources of omega-3s more effective?

In the two trials that involved dietary supplementation with omega-3 PUFAs, the results for all-cause mortality and cardiac death were conflicting, with one showing an increase in all-cause mortality and cardiac death and the other showing a decrease in both outcomes compared with the control group.

No harmful effects of omega-3 PUFAs were found in either the supplement- or diet-based studies.

WHAT’S NEW
More evidence of little benefit

The meta-analysis by Rizos et al is the most up-to-date, comprehensive look at the value of omega-3 fatty acids for primary and secondary prevention of cardiovascular events. It differs from previous reviews in that most of the included studies were well-done RCTs.

In addition, the studies were performed in both primary and secondary cardiovascular disease prevention settings and involved different forms of omega-3 PUFA supplementation, including dietary sources and supplements. The trials were predominantly larger than those included in previous systematic reviews, as well. The baseline risk for cardiovascular disease in the newer studies (seven of the 20 RCTs were completed after 2007) may be different from that of previous studies because of increased use of certain medications, such as statins.

In recent years, other studies of omega-3 PUFAs have had similar results. A meta-analysis of 14 RCTs found that omega-3 PUFA supplementation offered no benefit for the secondary prevention of cardiovascular disease.8 The FORWARD trial—published earlier this year—showed that omega-3 PUFAs did not decrease the recurrence of atrial fibrillation in patients with a history of confirmed paroxysmal atrial fibrillation.9 And an earlier (2006) analysis of RCTs and cohort studies found no benefit from omega-3 fatty acids for primary prevention of cardiovascular disease or cancer.10

 

 

CAVEATS
No significant help, and no harm

While this meta-analysis found no statistically significant benefits from omega-3 PUFAs, there is no evidence of harm from PUFA intake, whether from dietary sources or supplements. There is no need to tell patients who wish to take omega-3 supplements not to do so. But we should not promote their use for the sole purpose of cardiovascular disease prevention.

CHALLENGES TO IMPLEMENTATION
Changing minds won’t be easy

Despite recent findings indicating that omega-3 PUFAs provide little primary or secondary protection against cardiovascular events, advertising from supplement manufacturers may make it hard to change patients’ minds. Because diets and supplements containing these fatty acids do not cause apparent harm, patients and clinicians may decide that a small potential benefit is worth the expense.

References

1. Rizos E, Ntzani E, Bika E, et al. Association between omega-3 fatty acid supplementation and risk of major cardiovascular disease events: a systemic review and meta-analysis. JAMA. 2012; 308:1024-1033.

2. CDC. FastStats. Deaths and mortality. 2010 [updated April 5, 2013]. www.cdc.gov/nchs/fastats/deaths.htm. Accessed July 21, 2013.

3. Kris-Etherton PM, Harris WS, Appel LJ, et al. Fish consumption, fish oil, omega-3 fatty acids and cardiovascular disease. Circulation. 2002;106:2747-2757.

4. Van de Werf F, Bax J, Betriu A, et al. Management of acute myocardial infarction in patients presenting with persistent ST-segment elevation: the Task Force on the Management of ST-segment Elevation Acute Myocardial Infarction of the European Society of Cardiology. Eur Heart J. 2008;29:2909-2945.

5. Artham SM, Lavie CJ, Milani RV, et al. Fish oil in primary and secondary cardiovascular prevention. Ochsner J. 2008;8: 49-60.

6. Gruppo Italiano per lo Studio della Sopravvivenza nell’Infarto miocardico. Dietary supplementation with n-3 polyunsaturated fatty acids and vitamin E after myocardial infarction: results of GISSI-Prevenzione trial. Lancet. 1999; 354:447-455.

7. Yokoyama M, Origasa H, Matsuzaki M, et al. Effects of eicosapentaenoic acid on major coronary events in hypercholesteroleamic patients (JELIS): a randomized open-label, blinded endpoint analysis. Lancet. 2007;369:1090-1098.

8. Kwak SM, Myung SK, Lee YJ, et al. Efficacy of omega-3 fatty acid supplements (eicosapentaenoic acid and docosahexaenoic acid) in the secondary prevention of cardiovascular disease: a meta-analysis of randomized, double-blind, placebo-controlled trials. Arch Intern Med. 2012;172: 686-694.

9. Macchia A, Grancelli H, Varini S, et al. Omega-3 fatty acids for the prevention of recurrent symptomatic atrial fibrillation: results of the FORWARD (Randomized Trial to Assess Efficacy of PUFA for the Maintenance of Sinus Rhythm in Persistent Atrial Fibrillation) trial. J Am Coll Cardiol. 2013;61:463-468.

10. Hooper L, Thompson RL, Harrison RA, et al. Risks and benefits of omega 3 fats for mortality, cardiovascular disease, and cancer: systemic review. BMJ. 2006;332:752-760.

Acknowledgement

The PURLs Surveillance System was supported in part by Grant Number UL1RR024999 from the National Center for Research Resources, a Clinical Translational Science Award to the University of Chicago. The content is solely the responsibility of the authors and does not necessarily represent the official views of the National Center for Research Resources or the National Institutes of Health.

Copyright © 2013. The Family Physicians Inquiries Network. All rights reserved.

Reprinted with permission from the Family Physicians Inquiries Network and The Journal of Family Practice. 2013;62(7):372-374.

PRACTICE CHANGER
Stop recommending omega-3 fatty acid supplements for cardiovascular protection. They have no significant impact on all-cause mortality, acute myocardial infarction (MI), sudden death, or stroke.1

STRENGTH OF RECOMMENDATION
A: Based on a meta-analysis of randomized controlled trials (RCTs).

ILLUSTRATIVE CASE
A 59-year-old patient who had an MI three years ago is taking an ACE inhibitor, a statin, and a -blocker. He asks you whether he should also take omega-3 fatty acid supplements to further decrease his risk for heart disease. What should you tell him?

Coronary artery disease (CAD) kills more than 500,000 Americans every year,2 and medical and dietary therapies for primary and secondary cardiovascular protection are paramount. Omega-3 polyunsaturated fatty acid (PUFA) supplementation is one such therapy.

Omega-3 PUFAs are precursors to certain prostaglandins that decrease the proinflammatory state in patients with CAD. They also lower triglyceride levels and produce an anti-arrhythmic effect by promoting electrical stability.

But do PUFA supplements ­provide cardioprotection?

The American Heart Association’s Nutrition Committee recommends either omega-3 PUFA supplementation with 250 to 500 mg of eicosapentaenoic acid (EPA) and docosahexaenoic acid (DHA) per day or two servings of oily fish per week for both primary and secondary prevention of CAD.3 The European Society of Cardiology also encourages increased consumption of oily fish.4

These recommendations are based on primary and secondary prevention studies performed between 1989 and 2007, which found a 15% to 29% decrease in all-cause mortality and nonfatal cardiovascular events associated with regular intake of omega-3 fatty acids.5-7 The systematic review and meta-analysis detailed below revisited the effect of omega-3 supplementation on major cardiovascular outcomes.1

STUDY SUMMARY
Omega-3 supplements don’t lower cardiovascular risk

This meta-analysis included 20 RCTs with a total of 68,680 patients. The median age was 68, with a range of 49 to 70.  Thirteen of the studies evaluated omega-3 PUFAs for secondary prevention of cardiovascular outcomes, four assessed both primary and secondary prevention, and three looked at outcomes in patients with implantable cardioverter defibrillators. All lasted longer than one year, and most were high quality, with a low risk for bias.

The median treatment duration was two years, with a maximum of 6.2 years. The mean omega-3 PUFA dose evaluated in the studies was 1.5 g/d, with the exception of two studies in which patients received omega-3 PUFAs through dietary sources. Twelve studies used a dose of 1 g or more per day. Half of the included trials were performed during the period when statins were routinely prescribed for cardiovascular risk modification (1998 or later).

Outcomes included all-cause mortality (17 studies), cardiac death (13 studies), sudden death (seven studies), MI (13 studies), and stroke (nine studies).

This meta-analysis found trends toward a decrease in all-cause mortality, cardiac death, sudden death, and MI in patients taking omega-3 PUFAs, but no statistically significant association between any of the outcomes and omega-3 PUFA supplementation. The relative risk for all-cause mortality was 0.96. Prespecified subgroup analysis found no association between treatment effect and omega-3 fatty acid dose.

Are dietary sources of omega-3s more effective?

In the two trials that involved dietary supplementation with omega-3 PUFAs, the results for all-cause mortality and cardiac death were conflicting, with one showing an increase in all-cause mortality and cardiac death and the other showing a decrease in both outcomes compared with the control group.

No harmful effects of omega-3 PUFAs were found in either the supplement- or diet-based studies.

WHAT’S NEW
More evidence of little benefit

The meta-analysis by Rizos et al is the most up-to-date, comprehensive look at the value of omega-3 fatty acids for primary and secondary prevention of cardiovascular events. It differs from previous reviews in that most of the included studies were well-done RCTs.

In addition, the studies were performed in both primary and secondary cardiovascular disease prevention settings and involved different forms of omega-3 PUFA supplementation, including dietary sources and supplements. The trials were predominantly larger than those included in previous systematic reviews, as well. The baseline risk for cardiovascular disease in the newer studies (seven of the 20 RCTs were completed after 2007) may be different from that of previous studies because of increased use of certain medications, such as statins.

In recent years, other studies of omega-3 PUFAs have had similar results. A meta-analysis of 14 RCTs found that omega-3 PUFA supplementation offered no benefit for the secondary prevention of cardiovascular disease.8 The FORWARD trial—published earlier this year—showed that omega-3 PUFAs did not decrease the recurrence of atrial fibrillation in patients with a history of confirmed paroxysmal atrial fibrillation.9 And an earlier (2006) analysis of RCTs and cohort studies found no benefit from omega-3 fatty acids for primary prevention of cardiovascular disease or cancer.10

 

 

CAVEATS
No significant help, and no harm

While this meta-analysis found no statistically significant benefits from omega-3 PUFAs, there is no evidence of harm from PUFA intake, whether from dietary sources or supplements. There is no need to tell patients who wish to take omega-3 supplements not to do so. But we should not promote their use for the sole purpose of cardiovascular disease prevention.

CHALLENGES TO IMPLEMENTATION
Changing minds won’t be easy

Despite recent findings indicating that omega-3 PUFAs provide little primary or secondary protection against cardiovascular events, advertising from supplement manufacturers may make it hard to change patients’ minds. Because diets and supplements containing these fatty acids do not cause apparent harm, patients and clinicians may decide that a small potential benefit is worth the expense.

References

1. Rizos E, Ntzani E, Bika E, et al. Association between omega-3 fatty acid supplementation and risk of major cardiovascular disease events: a systemic review and meta-analysis. JAMA. 2012; 308:1024-1033.

2. CDC. FastStats. Deaths and mortality. 2010 [updated April 5, 2013]. www.cdc.gov/nchs/fastats/deaths.htm. Accessed July 21, 2013.

3. Kris-Etherton PM, Harris WS, Appel LJ, et al. Fish consumption, fish oil, omega-3 fatty acids and cardiovascular disease. Circulation. 2002;106:2747-2757.

4. Van de Werf F, Bax J, Betriu A, et al. Management of acute myocardial infarction in patients presenting with persistent ST-segment elevation: the Task Force on the Management of ST-segment Elevation Acute Myocardial Infarction of the European Society of Cardiology. Eur Heart J. 2008;29:2909-2945.

5. Artham SM, Lavie CJ, Milani RV, et al. Fish oil in primary and secondary cardiovascular prevention. Ochsner J. 2008;8: 49-60.

6. Gruppo Italiano per lo Studio della Sopravvivenza nell’Infarto miocardico. Dietary supplementation with n-3 polyunsaturated fatty acids and vitamin E after myocardial infarction: results of GISSI-Prevenzione trial. Lancet. 1999; 354:447-455.

7. Yokoyama M, Origasa H, Matsuzaki M, et al. Effects of eicosapentaenoic acid on major coronary events in hypercholesteroleamic patients (JELIS): a randomized open-label, blinded endpoint analysis. Lancet. 2007;369:1090-1098.

8. Kwak SM, Myung SK, Lee YJ, et al. Efficacy of omega-3 fatty acid supplements (eicosapentaenoic acid and docosahexaenoic acid) in the secondary prevention of cardiovascular disease: a meta-analysis of randomized, double-blind, placebo-controlled trials. Arch Intern Med. 2012;172: 686-694.

9. Macchia A, Grancelli H, Varini S, et al. Omega-3 fatty acids for the prevention of recurrent symptomatic atrial fibrillation: results of the FORWARD (Randomized Trial to Assess Efficacy of PUFA for the Maintenance of Sinus Rhythm in Persistent Atrial Fibrillation) trial. J Am Coll Cardiol. 2013;61:463-468.

10. Hooper L, Thompson RL, Harrison RA, et al. Risks and benefits of omega 3 fats for mortality, cardiovascular disease, and cancer: systemic review. BMJ. 2006;332:752-760.

Acknowledgement

The PURLs Surveillance System was supported in part by Grant Number UL1RR024999 from the National Center for Research Resources, a Clinical Translational Science Award to the University of Chicago. The content is solely the responsibility of the authors and does not necessarily represent the official views of the National Center for Research Resources or the National Institutes of Health.

Copyright © 2013. The Family Physicians Inquiries Network. All rights reserved.

Reprinted with permission from the Family Physicians Inquiries Network and The Journal of Family Practice. 2013;62(7):372-374.

Issue
Clinician Reviews - 23(9)
Issue
Clinician Reviews - 23(9)
Page Number
20
Page Number
20
Publications
Publications
Topics
Article Type
Display Headline
Should You Still Recommend Omega-3 Supplements?
Display Headline
Should You Still Recommend Omega-3 Supplements?
Legacy Keywords
purls, omega-3, supplements, cardiovascular, protection, practice change, fatty acid
Legacy Keywords
purls, omega-3, supplements, cardiovascular, protection, practice change, fatty acid
Sections
Article Source

PURLs Copyright

Inside the Article

Consider this strategy for upper GI bleeds

Article Type
Changed
Tue, 07/14/2020 - 15:18
Display Headline
Consider this strategy for upper GI bleeds
PRACTICE CHANGER

Do not order transfusions of red blood cells for patients with acute upper gastrointestinal bleeding unless their hemoglobin level <7 g/dL.

Villanueva C, Colomo A, Bosch A, et al. Transfusion strategies for acute upper gastrointestinal bleeding. N Engl J Med. 2013;368:11-21.1

A: Based on a single randomized controlled trial (RCT) consistent with other RCTs on recommendations for transfusion.

 

ILLUSTRATED CASE

An 82-year-old patient presents to the emergency department with several episodes of melena over the past week and one episode of hematemesis this morning. He denies any shortness of breath, dizziness, lightheadedness, or fatigue. He is tachycardic but normotensive. Lab results note a hemoglobin level of 8.3 g/dL. Should you order a transfusion of red blood cells?

Acute upper gastrointestinal bleeding (UGIB) commonly requires hospital admission, with approximately 61 cases per 100,000 population in the United States in 2009.2 Gastroduodenal peptic ulcer disease accounts for the majority of these cases.3 Although trends indicate an overall decrease in cases requiring hospitalization, UGIB remains a condition associated with a mortality rate of 2.5% and inpatient costs of $2 billion annually.2,3

Studies have been inconclusive—until now
An RCT published in 1999 showed a restrictive transfusion strategy (hemoglobin threshold of 7 g/dL) to be at least as effective as—and possibly superior to—a liberal strategy (threshold of 10 g/dL) in critically ill patients.4 In 2010, an RCT demonstrated that a liberal transfusion strategy (also defined as a transfusion threshold of 10 g/dL) did not reduce the rates of death or in-hospital morbidity in elderly patients after hip surgery.5 A recent Cochrane review of transfusion strategies for UGIB included only 3 small studies (N=93), so its authors could not draw any firm conclusions.6 The results of a new RCT, detailed below, are more conclusive.

STUDY SUMMARY: Restrictive transfusion policy lowers mortality risk

Villanueva et al conducted a nonblinded RCT comparing outcomes in patients admitted to the hospital with moderate-risk acute UGIB transfused on a liberal vs a restrictive strategy.1 The restrictive group used a transfusion hemoglobin threshold of 7 g/dL and a posttransfusion target of 7 to 9 g/dL; the liberal group used a threshold of 9 g/dL, with a posttransfusion target of 9 to 11 g/dL. Patients received one unit of red blood cells at a time until their hemoglobin was above the predetermined threshold.

Patients were excluded if they declined blood transfusion; had massive exsanguinating bleeding, acute coronary syndrome, symptomatic peripheral vasculopathy, stroke, lower GI bleeding, or a transient ischemic attack; had received a transfusion within the previous 90 days; or had a recent history of surgery or trauma. Patients at low risk of rebleeding (as defined by the Rockall risk scoring system) were also excluded. Randomization was stratified by the presence or absence of cirrhosis of the liver.

Participants (N=921) had confirmed hematemesis and/or melena on admission. All underwent emergency gastroscopy within 6 hours of admission, with subsequent interventions based on endoscopic findings. In addition to established hemoglobin levels, patients received a transfusion anytime they developed signs or symptoms related to anemia, massive bleeding, or the need for surgery. Staff monitored hemoglobin levels every 8 hours during the first 48 hours, then daily thereafter.

Both groups had similar baseline characteristics, including hemoglobin on admission and source of bleeding. The authors used intention-to-treat analysis to identify the primary outcome: death from any cause at 45 days. Secondary outcomes were further bleeding and in-hospital complications.

During hospitalization, 49% of patients in the restrictive group and 86% of those in the liberal group received a blood transfusion (P<.001). Thirty-two patients (17 from the restrictive group and 15 from the liberal group) withdrew from the study, leaving 889 patients for overall analysis.

At 45 days, overall mortality from any cause was 5% in the restrictive group and 9% in the liberal group (P=.02; number needed to treat [NNT]=25). Sub-group analysis revealed a lower risk of death in patients with cirrhosis and Child-Pugh class A or B disease assigned to the restrictive transfusion group vs the liberal group. The results showed a trend toward a lower risk of death in patients with bleeding from varices or peptic ulcers for the restrictive group, as well.

In addition, the restrictive transfusion group had a significantly lower rate of adverse events (40% vs 48% for the liberal transfusion group; P=.02, NNT=13), with a significant reduction in transfusion reactions (3% vs 9%; P=.001, NNT=17) and cardiac complications (11% vs 16%; P=.04, NNT=20). The restrictive group had a lower rate of further bleeding (10% vs 16% for the liberal transfusion group; P=.01, NNT=17), as well.

 

 

 

WHAT'S NEW: Many reasons to limit transfusions for acute upper GI bleed

This RCT provides evidence that patients with acute UGIB have improved survival rates and fewer adverse events when a restrictive transfusion strategy is used. In addition to improving patient outcomes, a restrictive strategy will likely reduce costs and overall use of blood products. Thus, the study, along with other recent evaluations, adds evidence to support more restrictive transfusion thresholds.

The AABB (formerly named the American Association of Blood Banks) recently
released guidelines calling for restrictive transfusion thresholds (7-8 g/dL) in stable hospitalized patients.7 In 2012, the American College of Gastroenterology published a practice guideline with a recommended target hemoglobin level of ≥7 g/dL in the management of patients who have ulcer bleeding but no signs of intravascular depletion or comorbidities such as coronary artery disease.8

CAVEATS: Results might differ when endoscopy is delayed

The patients in the study detailed here underwent emergency gastroscopy within 6 hours of admission, and both groups received the same therapies based on endoscopic findings. It remains unclear whether the benefits of a restrictive transfusion strategy would persist in patients who do not undergo endoscopy within that timeframe. And, because the reported baseline characteristics of the patients did not include the prevalence of cardiac disease, caution should be exercised before extrapolating these results to patients with underlying (active or historical) cardiac disease.

CHALLENGES TO IMPLEMENTATION: Changing long-held policies may be difficult

Although RCTs as well as clinical guidelines suggest that restrictive transfusion policies are safe and effective, changing long-held clinical practices is never easy.

ACKNOWLEDGEMENT

The PURLs Surveillance System was supported in part by Grant Number UL1RR024999 from the National Center For Research Resources, a Clinical Translational Science Award to the University of Chicago. The content is solely the responsibility of the authors and does not necessarily represent the official views of the National Center for Research Resources or the National Institutes of Health.

Files
References

1. Villanueva C, Colomo A, Bosch A, et al. Transfusion strategies for acute upper gastrointestinal bleeding. N Engl J Med.2013;368:11-21.

2. Laine L, Yang H, Chang SC,et al. Trends for incidence of hospitalization and death due to GI complications in the United States from 2001 to 2009. Am J Gastroenterol 2012; 107:1190-1195.

3. Gralnek IM, Barkun AN, Bardou M. Management of acute bleeding from a peptic ulcer. N Engl J Med 2008; 359:928-937.

4. Hebert PC, Wells G, Blajchman MA, et al. A multicenter, randomized, controlled clinical trial of transfusion requirements in critical care. Transfusion Requirements in Critical Care Investigators, Canadian Critical Care Trials Group. N Engl J Med 1999; 340:409-417.

5. Carson JL, Terrin ML, Noveck H, et al. Liberal or restrictive transfusion in high-risk patients after hip surgery. N Engl J Med 2011; 365:2453-2462.

6. Jairath V, Hearnshaw S, Brunskill SJ, et al. Red cell transfusion for the management of upper gastrointestinal haemorrhage. Cochrane Database of Systematic Reviews 2010;CD006613.

7. Carson JL, Grossman BJ, Kleinman S, et al. Red blood cell transfusion: a clinical practice guideline from the AABB. Ann Intern Med 2012; 157:49-58.

8. Laine L, Jensen DM. Management of patients with ulcer bleeding. Am J Gastroenterol 2012; 107:345-360.

Article PDF
Author and Disclosure Information

Kevin Frazer, MD;
Kate Kirley, MD;
James Stevermer, MD

University of Missouri-Columbia  (Drs. Frazer and Stevermer); The University of Chicago (Dr. Kirley)

PURLs Editor
Anne Mounsey, MD
University of North Carolina at Chapel Hill

Issue
The Journal of Family Practice - 62(9)
Publications
Topics
Page Number
E6-E8
Legacy Keywords
Kevin Frazer; MD; Kate Kirley; MD; James Stevermer; MD; upper gastrointestinal bleeding; UGIB; GI bleed; PURLs; endoscopy; American College of Gastroenterology
Sections
Files
Files
Author and Disclosure Information

Kevin Frazer, MD;
Kate Kirley, MD;
James Stevermer, MD

University of Missouri-Columbia  (Drs. Frazer and Stevermer); The University of Chicago (Dr. Kirley)

PURLs Editor
Anne Mounsey, MD
University of North Carolina at Chapel Hill

Author and Disclosure Information

Kevin Frazer, MD;
Kate Kirley, MD;
James Stevermer, MD

University of Missouri-Columbia  (Drs. Frazer and Stevermer); The University of Chicago (Dr. Kirley)

PURLs Editor
Anne Mounsey, MD
University of North Carolina at Chapel Hill

Article PDF
Article PDF
PRACTICE CHANGER

Do not order transfusions of red blood cells for patients with acute upper gastrointestinal bleeding unless their hemoglobin level <7 g/dL.

Villanueva C, Colomo A, Bosch A, et al. Transfusion strategies for acute upper gastrointestinal bleeding. N Engl J Med. 2013;368:11-21.1

A: Based on a single randomized controlled trial (RCT) consistent with other RCTs on recommendations for transfusion.

 

ILLUSTRATED CASE

An 82-year-old patient presents to the emergency department with several episodes of melena over the past week and one episode of hematemesis this morning. He denies any shortness of breath, dizziness, lightheadedness, or fatigue. He is tachycardic but normotensive. Lab results note a hemoglobin level of 8.3 g/dL. Should you order a transfusion of red blood cells?

Acute upper gastrointestinal bleeding (UGIB) commonly requires hospital admission, with approximately 61 cases per 100,000 population in the United States in 2009.2 Gastroduodenal peptic ulcer disease accounts for the majority of these cases.3 Although trends indicate an overall decrease in cases requiring hospitalization, UGIB remains a condition associated with a mortality rate of 2.5% and inpatient costs of $2 billion annually.2,3

Studies have been inconclusive—until now
An RCT published in 1999 showed a restrictive transfusion strategy (hemoglobin threshold of 7 g/dL) to be at least as effective as—and possibly superior to—a liberal strategy (threshold of 10 g/dL) in critically ill patients.4 In 2010, an RCT demonstrated that a liberal transfusion strategy (also defined as a transfusion threshold of 10 g/dL) did not reduce the rates of death or in-hospital morbidity in elderly patients after hip surgery.5 A recent Cochrane review of transfusion strategies for UGIB included only 3 small studies (N=93), so its authors could not draw any firm conclusions.6 The results of a new RCT, detailed below, are more conclusive.

STUDY SUMMARY: Restrictive transfusion policy lowers mortality risk

Villanueva et al conducted a nonblinded RCT comparing outcomes in patients admitted to the hospital with moderate-risk acute UGIB transfused on a liberal vs a restrictive strategy.1 The restrictive group used a transfusion hemoglobin threshold of 7 g/dL and a posttransfusion target of 7 to 9 g/dL; the liberal group used a threshold of 9 g/dL, with a posttransfusion target of 9 to 11 g/dL. Patients received one unit of red blood cells at a time until their hemoglobin was above the predetermined threshold.

Patients were excluded if they declined blood transfusion; had massive exsanguinating bleeding, acute coronary syndrome, symptomatic peripheral vasculopathy, stroke, lower GI bleeding, or a transient ischemic attack; had received a transfusion within the previous 90 days; or had a recent history of surgery or trauma. Patients at low risk of rebleeding (as defined by the Rockall risk scoring system) were also excluded. Randomization was stratified by the presence or absence of cirrhosis of the liver.

Participants (N=921) had confirmed hematemesis and/or melena on admission. All underwent emergency gastroscopy within 6 hours of admission, with subsequent interventions based on endoscopic findings. In addition to established hemoglobin levels, patients received a transfusion anytime they developed signs or symptoms related to anemia, massive bleeding, or the need for surgery. Staff monitored hemoglobin levels every 8 hours during the first 48 hours, then daily thereafter.

Both groups had similar baseline characteristics, including hemoglobin on admission and source of bleeding. The authors used intention-to-treat analysis to identify the primary outcome: death from any cause at 45 days. Secondary outcomes were further bleeding and in-hospital complications.

During hospitalization, 49% of patients in the restrictive group and 86% of those in the liberal group received a blood transfusion (P<.001). Thirty-two patients (17 from the restrictive group and 15 from the liberal group) withdrew from the study, leaving 889 patients for overall analysis.

At 45 days, overall mortality from any cause was 5% in the restrictive group and 9% in the liberal group (P=.02; number needed to treat [NNT]=25). Sub-group analysis revealed a lower risk of death in patients with cirrhosis and Child-Pugh class A or B disease assigned to the restrictive transfusion group vs the liberal group. The results showed a trend toward a lower risk of death in patients with bleeding from varices or peptic ulcers for the restrictive group, as well.

In addition, the restrictive transfusion group had a significantly lower rate of adverse events (40% vs 48% for the liberal transfusion group; P=.02, NNT=13), with a significant reduction in transfusion reactions (3% vs 9%; P=.001, NNT=17) and cardiac complications (11% vs 16%; P=.04, NNT=20). The restrictive group had a lower rate of further bleeding (10% vs 16% for the liberal transfusion group; P=.01, NNT=17), as well.

 

 

 

WHAT'S NEW: Many reasons to limit transfusions for acute upper GI bleed

This RCT provides evidence that patients with acute UGIB have improved survival rates and fewer adverse events when a restrictive transfusion strategy is used. In addition to improving patient outcomes, a restrictive strategy will likely reduce costs and overall use of blood products. Thus, the study, along with other recent evaluations, adds evidence to support more restrictive transfusion thresholds.

The AABB (formerly named the American Association of Blood Banks) recently
released guidelines calling for restrictive transfusion thresholds (7-8 g/dL) in stable hospitalized patients.7 In 2012, the American College of Gastroenterology published a practice guideline with a recommended target hemoglobin level of ≥7 g/dL in the management of patients who have ulcer bleeding but no signs of intravascular depletion or comorbidities such as coronary artery disease.8

CAVEATS: Results might differ when endoscopy is delayed

The patients in the study detailed here underwent emergency gastroscopy within 6 hours of admission, and both groups received the same therapies based on endoscopic findings. It remains unclear whether the benefits of a restrictive transfusion strategy would persist in patients who do not undergo endoscopy within that timeframe. And, because the reported baseline characteristics of the patients did not include the prevalence of cardiac disease, caution should be exercised before extrapolating these results to patients with underlying (active or historical) cardiac disease.

CHALLENGES TO IMPLEMENTATION: Changing long-held policies may be difficult

Although RCTs as well as clinical guidelines suggest that restrictive transfusion policies are safe and effective, changing long-held clinical practices is never easy.

ACKNOWLEDGEMENT

The PURLs Surveillance System was supported in part by Grant Number UL1RR024999 from the National Center For Research Resources, a Clinical Translational Science Award to the University of Chicago. The content is solely the responsibility of the authors and does not necessarily represent the official views of the National Center for Research Resources or the National Institutes of Health.

PRACTICE CHANGER

Do not order transfusions of red blood cells for patients with acute upper gastrointestinal bleeding unless their hemoglobin level <7 g/dL.

Villanueva C, Colomo A, Bosch A, et al. Transfusion strategies for acute upper gastrointestinal bleeding. N Engl J Med. 2013;368:11-21.1

A: Based on a single randomized controlled trial (RCT) consistent with other RCTs on recommendations for transfusion.

 

ILLUSTRATED CASE

An 82-year-old patient presents to the emergency department with several episodes of melena over the past week and one episode of hematemesis this morning. He denies any shortness of breath, dizziness, lightheadedness, or fatigue. He is tachycardic but normotensive. Lab results note a hemoglobin level of 8.3 g/dL. Should you order a transfusion of red blood cells?

Acute upper gastrointestinal bleeding (UGIB) commonly requires hospital admission, with approximately 61 cases per 100,000 population in the United States in 2009.2 Gastroduodenal peptic ulcer disease accounts for the majority of these cases.3 Although trends indicate an overall decrease in cases requiring hospitalization, UGIB remains a condition associated with a mortality rate of 2.5% and inpatient costs of $2 billion annually.2,3

Studies have been inconclusive—until now
An RCT published in 1999 showed a restrictive transfusion strategy (hemoglobin threshold of 7 g/dL) to be at least as effective as—and possibly superior to—a liberal strategy (threshold of 10 g/dL) in critically ill patients.4 In 2010, an RCT demonstrated that a liberal transfusion strategy (also defined as a transfusion threshold of 10 g/dL) did not reduce the rates of death or in-hospital morbidity in elderly patients after hip surgery.5 A recent Cochrane review of transfusion strategies for UGIB included only 3 small studies (N=93), so its authors could not draw any firm conclusions.6 The results of a new RCT, detailed below, are more conclusive.

STUDY SUMMARY: Restrictive transfusion policy lowers mortality risk

Villanueva et al conducted a nonblinded RCT comparing outcomes in patients admitted to the hospital with moderate-risk acute UGIB transfused on a liberal vs a restrictive strategy.1 The restrictive group used a transfusion hemoglobin threshold of 7 g/dL and a posttransfusion target of 7 to 9 g/dL; the liberal group used a threshold of 9 g/dL, with a posttransfusion target of 9 to 11 g/dL. Patients received one unit of red blood cells at a time until their hemoglobin was above the predetermined threshold.

Patients were excluded if they declined blood transfusion; had massive exsanguinating bleeding, acute coronary syndrome, symptomatic peripheral vasculopathy, stroke, lower GI bleeding, or a transient ischemic attack; had received a transfusion within the previous 90 days; or had a recent history of surgery or trauma. Patients at low risk of rebleeding (as defined by the Rockall risk scoring system) were also excluded. Randomization was stratified by the presence or absence of cirrhosis of the liver.

Participants (N=921) had confirmed hematemesis and/or melena on admission. All underwent emergency gastroscopy within 6 hours of admission, with subsequent interventions based on endoscopic findings. In addition to established hemoglobin levels, patients received a transfusion anytime they developed signs or symptoms related to anemia, massive bleeding, or the need for surgery. Staff monitored hemoglobin levels every 8 hours during the first 48 hours, then daily thereafter.

Both groups had similar baseline characteristics, including hemoglobin on admission and source of bleeding. The authors used intention-to-treat analysis to identify the primary outcome: death from any cause at 45 days. Secondary outcomes were further bleeding and in-hospital complications.

During hospitalization, 49% of patients in the restrictive group and 86% of those in the liberal group received a blood transfusion (P<.001). Thirty-two patients (17 from the restrictive group and 15 from the liberal group) withdrew from the study, leaving 889 patients for overall analysis.

At 45 days, overall mortality from any cause was 5% in the restrictive group and 9% in the liberal group (P=.02; number needed to treat [NNT]=25). Sub-group analysis revealed a lower risk of death in patients with cirrhosis and Child-Pugh class A or B disease assigned to the restrictive transfusion group vs the liberal group. The results showed a trend toward a lower risk of death in patients with bleeding from varices or peptic ulcers for the restrictive group, as well.

In addition, the restrictive transfusion group had a significantly lower rate of adverse events (40% vs 48% for the liberal transfusion group; P=.02, NNT=13), with a significant reduction in transfusion reactions (3% vs 9%; P=.001, NNT=17) and cardiac complications (11% vs 16%; P=.04, NNT=20). The restrictive group had a lower rate of further bleeding (10% vs 16% for the liberal transfusion group; P=.01, NNT=17), as well.

 

 

 

WHAT'S NEW: Many reasons to limit transfusions for acute upper GI bleed

This RCT provides evidence that patients with acute UGIB have improved survival rates and fewer adverse events when a restrictive transfusion strategy is used. In addition to improving patient outcomes, a restrictive strategy will likely reduce costs and overall use of blood products. Thus, the study, along with other recent evaluations, adds evidence to support more restrictive transfusion thresholds.

The AABB (formerly named the American Association of Blood Banks) recently
released guidelines calling for restrictive transfusion thresholds (7-8 g/dL) in stable hospitalized patients.7 In 2012, the American College of Gastroenterology published a practice guideline with a recommended target hemoglobin level of ≥7 g/dL in the management of patients who have ulcer bleeding but no signs of intravascular depletion or comorbidities such as coronary artery disease.8

CAVEATS: Results might differ when endoscopy is delayed

The patients in the study detailed here underwent emergency gastroscopy within 6 hours of admission, and both groups received the same therapies based on endoscopic findings. It remains unclear whether the benefits of a restrictive transfusion strategy would persist in patients who do not undergo endoscopy within that timeframe. And, because the reported baseline characteristics of the patients did not include the prevalence of cardiac disease, caution should be exercised before extrapolating these results to patients with underlying (active or historical) cardiac disease.

CHALLENGES TO IMPLEMENTATION: Changing long-held policies may be difficult

Although RCTs as well as clinical guidelines suggest that restrictive transfusion policies are safe and effective, changing long-held clinical practices is never easy.

ACKNOWLEDGEMENT

The PURLs Surveillance System was supported in part by Grant Number UL1RR024999 from the National Center For Research Resources, a Clinical Translational Science Award to the University of Chicago. The content is solely the responsibility of the authors and does not necessarily represent the official views of the National Center for Research Resources or the National Institutes of Health.

References

1. Villanueva C, Colomo A, Bosch A, et al. Transfusion strategies for acute upper gastrointestinal bleeding. N Engl J Med.2013;368:11-21.

2. Laine L, Yang H, Chang SC,et al. Trends for incidence of hospitalization and death due to GI complications in the United States from 2001 to 2009. Am J Gastroenterol 2012; 107:1190-1195.

3. Gralnek IM, Barkun AN, Bardou M. Management of acute bleeding from a peptic ulcer. N Engl J Med 2008; 359:928-937.

4. Hebert PC, Wells G, Blajchman MA, et al. A multicenter, randomized, controlled clinical trial of transfusion requirements in critical care. Transfusion Requirements in Critical Care Investigators, Canadian Critical Care Trials Group. N Engl J Med 1999; 340:409-417.

5. Carson JL, Terrin ML, Noveck H, et al. Liberal or restrictive transfusion in high-risk patients after hip surgery. N Engl J Med 2011; 365:2453-2462.

6. Jairath V, Hearnshaw S, Brunskill SJ, et al. Red cell transfusion for the management of upper gastrointestinal haemorrhage. Cochrane Database of Systematic Reviews 2010;CD006613.

7. Carson JL, Grossman BJ, Kleinman S, et al. Red blood cell transfusion: a clinical practice guideline from the AABB. Ann Intern Med 2012; 157:49-58.

8. Laine L, Jensen DM. Management of patients with ulcer bleeding. Am J Gastroenterol 2012; 107:345-360.

References

1. Villanueva C, Colomo A, Bosch A, et al. Transfusion strategies for acute upper gastrointestinal bleeding. N Engl J Med.2013;368:11-21.

2. Laine L, Yang H, Chang SC,et al. Trends for incidence of hospitalization and death due to GI complications in the United States from 2001 to 2009. Am J Gastroenterol 2012; 107:1190-1195.

3. Gralnek IM, Barkun AN, Bardou M. Management of acute bleeding from a peptic ulcer. N Engl J Med 2008; 359:928-937.

4. Hebert PC, Wells G, Blajchman MA, et al. A multicenter, randomized, controlled clinical trial of transfusion requirements in critical care. Transfusion Requirements in Critical Care Investigators, Canadian Critical Care Trials Group. N Engl J Med 1999; 340:409-417.

5. Carson JL, Terrin ML, Noveck H, et al. Liberal or restrictive transfusion in high-risk patients after hip surgery. N Engl J Med 2011; 365:2453-2462.

6. Jairath V, Hearnshaw S, Brunskill SJ, et al. Red cell transfusion for the management of upper gastrointestinal haemorrhage. Cochrane Database of Systematic Reviews 2010;CD006613.

7. Carson JL, Grossman BJ, Kleinman S, et al. Red blood cell transfusion: a clinical practice guideline from the AABB. Ann Intern Med 2012; 157:49-58.

8. Laine L, Jensen DM. Management of patients with ulcer bleeding. Am J Gastroenterol 2012; 107:345-360.

Issue
The Journal of Family Practice - 62(9)
Issue
The Journal of Family Practice - 62(9)
Page Number
E6-E8
Page Number
E6-E8
Publications
Publications
Topics
Article Type
Display Headline
Consider this strategy for upper GI bleeds
Display Headline
Consider this strategy for upper GI bleeds
Legacy Keywords
Kevin Frazer; MD; Kate Kirley; MD; James Stevermer; MD; upper gastrointestinal bleeding; UGIB; GI bleed; PURLs; endoscopy; American College of Gastroenterology
Legacy Keywords
Kevin Frazer; MD; Kate Kirley; MD; James Stevermer; MD; upper gastrointestinal bleeding; UGIB; GI bleed; PURLs; endoscopy; American College of Gastroenterology
Sections
PURLs Copyright

Copyright 2013. The Family Physicians Inquiries Network. All rights reserved.

Disallow All Ads
Alternative CME
Use ProPublica
Hide sidebar & use full width
render the right sidebar.
Conference Recap Checkbox
Not Conference Recap
Clinical Edge
Display the Slideshow in this Article
Article PDF Media
Media Files

This asthma treatment has a lasting side effect in children

Article Type
Changed
Tue, 07/14/2020 - 15:12
Display Headline
This asthma treatment has a lasting side effect in children
Practice changer

Before prescribing inhaled corticosteroids (ICS) for a child with asthma, tell the patient—and parents—that their use could lead to a small but permanent effect on adult height.1

STRENGTH OF RECOMMENDATIONS

B: Based on one prospective study.

Kelly HW, Sternberg AL, Lescher R, et al; CAMP Research Group. Effect of inhaled glucocorticoids in childhood on adult height. N Engl J Med. 2012;367:904-912.

 

Illustrative case

A 10-year-old boy is brought in by his father for asthma follow-up. The child uses an albuterol inhaler, but has had increased coughing and wheezing recently. You are ready to step up his asthma therapy to include ICS. But the patient’s father questions this, noting that he recently read that steroids may reduce a child’s growth. How should you respond?

Inhaled corticosteroids (ICS) are a mainstay in the treatment of asthma ranging from mild persistent to severe. Standards of care for asthma treatment involve a stepwise approach, with ICS added if symptoms are not controlled with short-acting beta antagonists alone.2 In addition, monotherapy with ICS is more effective for controlling symptoms than leukotriene inhibitors or other controller medications, while also decreasing hospitalizations and nocturnal awakenings and improving quality of life—with few side effects.3

What we know about ICS and children’s growth
One adverse effect of ICS, however, is that of “decreased linear growth velocity”4—ie, slowing the rate at which children grow. Until recently, children were thought to “catch up” later in life, either by growing for a longer period of time than they would had they not taken ICS or by growing at an increased velocity after ICS medications are discontinued.4-6

Study summary: The effect on growth is small, but long-lasting

Kelly et al conducted a prospective observational cohort study that followed 943 (90.7%) participants in the Childhood Asthma Management Program (CAMP) in the years after the randomized controlled trial (RCT) ended.

A double-blind, placebo-controlled RCT, CAMP studied the linear growth of 1041 children with mild-to-moderate persistent asthma who were divided into 3 treatment groups: One group received 200 mcg inhaled budesonide twice daily; a second group received 8 mg inhaled nedocromil twice daily; and a third group received placebo. Albuterol was used symptomatically by all 3 groups.7 The children ranged in age from 5 to 13 years at the start of the study; 98 patients—split evenly among the 3 treatment arms—were lost to follow-up.

During the 4 to 6 years of the CAMP trial, the budesonide group received a mean total of 636 mg ICS, whereas the nedocromil and placebo groups received an average of 88.5 and 109.4 mg ICS, respectively. After the RCT ended, all participants had standardized asthma treatment, receiving mean adjusted total doses of ICS of 381 mg for the budesonide group, 347.9 mg for the nedocromil group, and 355 mg for the placebo group.

Patients’ height was measured every 6 months for the next 4.5 years, and once or twice a year thereafter until they reached adult height (at a mean age of 24.9±2.7 years).

ICS users were a half inch shorter
Long-term ICS use was linked to a lower adult height. The adjusted mean height was 171.1 cm for the budesonide group vs 172.3 cm for those on placebo, a difference of 1.2 cm, or 0.47 inch (95% confidence interval [CI], −1.9 to −0.5; P=.001); the mean adult height in the nedocromil group (172.1 cm) was similar to that of the placebo group (−0.2 cm; 95% CI, −0.9 to 0.5; P=.61).

The lower adult height in the ICS group did not vary significantly based on sex, age at trial entry, race, or duration of asthma prior to trial entry; however, dose was a key factor. A larger daily dose of budesonide—particularly in the first 2 years of the RCT—was associated with a lower adult height (about −0.1 cm for each mcg/kg in that 2-year time frame). This was consistent with results from studies that looked at other types of ICS (beclomethasone, fluticasone, and mometasone).8-11

The study also showed that growth velocity was reduced in the first 2 years of assigned treatment with budesonide, and this was primarily among prepubertal participants. After the initial 2-year slowing in growth rate, the children resumed growing at normal speeds.

 

 

 

What’s new: Now we know: Children don’t “catch up"

Retrospective studies have reported that children on ICS for mild persistent to moderate asthma would have an initial slowing in growth velocity but then “catch up” by growing for a longer period of time.3-5 This is the first prospective study with good follow-up to show that ICS use affects long-term growth and adult height. While the effect is not large, some children and their families might be concerned about it.

Caveats: ICS use was atypical

The randomized controlled portion of the study used a prescribed dose of budesonide without regard to symptoms. This is not the typical pattern of ICS use. In addition, compliance with ICS varies significantly.12 Because the effect on adult height appears to be dose dependent, however, we think the results of this study are valid.

In addition, there was a placebo control group (and big differences in exposure to ICS) only for the duration of the RCT. During the subsequent study, all patients received equivalent doses of ICS. This means that the variation in mean adult height achieved can be primarily ascribed to participants’ use of ICS during the 4- to 6-year CAMP trial. Of note, the effect of ICS was greatest in prepubertal participants, so there may be a diminished effect as teens approach their final height.

The study did not look at the effect of ICS use in patients with severe asthma—the group most likely to use ICS. However, the benefits of ICS for those with severe asthma likely outweigh any negative effects on adult height.

Challenges to implementation: What to tell patients

The message we convey to patients (and parents) about ICS use is a nuanced one. We can stress that ICS remain very important in the treatment of asthma and, while it appears that their use causes a slight decrease in adult height, most children with persistent asthma benefit from ICS.

Acknowledgement

The PURLs Surveillance System is supported in part by Grant Number UL1RR024999 from the National Center for Research Resources, a Clinical Translational Science Award to the University of Chicago. The content is solely the responsibility of the authors and does not necessarily represent the official views of the National Center for Research Resources or the National Institutes of Health.

Files
References

1. Kelly HW, Sternberg AL, Lescher R, et al; CAMP Research Group. Effect of inhaled glucocorticoids in childhood on adult height. N Engl J Med. 2012;367:904-912.

2. Expert Panel Report 3: Guidelines for the Diagnosis and Management of Asthma. National Institutes of Health National Heart, Lung and Blood Institute: National Asthma Education and Prevention Program, 2007. Available at: http://www.nhlbi. nih.gov/guidelines/asthma/asthgdln.pdf. Accessed August 15, 2013.

3. Chauhan BF, Ducharme FM. Anti-leukotriene agents compared to inhaled corticosteroids in the management of recurrent and/ or chronic asthma in adults and children. Cochrane Database Syst Rev. 2012;(5):CD002314.

4. Agertoft L, Pedersen S. Effect of long- term treatment with budesonide on adult height in children with asthma. N Engl J Med. 2000;343:1064-1069.

5. Van Bever HP, Desager KN, Lijssens N, et al. Does treatment of asthmatic children with inhaled corticosteroids affect their adult height? Pediatr Pulmonol. 1999;27:369-375.

6. Silverstein MD, Yunginger JW, Reed CE, et al. Attained adult height after childhood asthma: effect of glucocorticoid therapy. J Allergy Clin Immunol. 1997;99:466-474.

7. The Childhood Asthma Management Program Research Group. Long-term effects of budesonide or nedocromil in children with asthma. N Engl J Med. 2000;343:1054-1063.

8. Tinkelman DG, Reed CE, Nelson HS, et al. Aerosol beclomethasone dipropionate compared with theophylline as primary treatment of chronic, mild to moderately severe asthma in children. Pediatrics. 1993;92:64-77.

9. Verberne AA, Frost C, Roorda RJ, et al. One year treatment with salmeterol compared with beclomethasone in children with asthma. Am J Respir Crit Care Med. 1997;156:688-695.

10. Allen DB, Bronsky EA, LaForce CF, et al. Growth in asthmatic children treated with fluticasone propionate. J Pediatr 1998;132: 472-477.

11. Skoner DP, Meltzer EO, Milgrom H, et al. Effects of inhaled mometasone furoate on growth velocity and adrenal function: a placebo-controlled trial in children 4-9 years old with mild persistent asthma. J Asthma. 2011;48:848-859.

12. Cochrane MG, Bala MV, Downs KE, et al. Inhaled corticosteroids for asthma therapy: patient compliance, devices, and inhalation technique. Chest. 2000;117:542-550.

Article PDF
Author and Disclosure Information

Tanner Nissly, DO;
Shailendra Prasad, MBBS, MPH

Department of Family Medicine and Community Health, University of Minnesota, Minneapolis

PURLs Editor
James Stevermer, MD, MSPH
Department of Family Medicine, University of Missouri at Columbia

Issue
The Journal of Family Practice - 62(9)
Publications
Topics
Page Number
500-502
Legacy Keywords
Tanner Nissly; DO; Shailendra Prasad; MBBS; MPH; asthma; PURLs; inhaled corticosteroids; ICS; Childhood Asthma Management Program; CAMP;
Sections
Files
Files
Author and Disclosure Information

Tanner Nissly, DO;
Shailendra Prasad, MBBS, MPH

Department of Family Medicine and Community Health, University of Minnesota, Minneapolis

PURLs Editor
James Stevermer, MD, MSPH
Department of Family Medicine, University of Missouri at Columbia

Author and Disclosure Information

Tanner Nissly, DO;
Shailendra Prasad, MBBS, MPH

Department of Family Medicine and Community Health, University of Minnesota, Minneapolis

PURLs Editor
James Stevermer, MD, MSPH
Department of Family Medicine, University of Missouri at Columbia

Article PDF
Article PDF
Practice changer

Before prescribing inhaled corticosteroids (ICS) for a child with asthma, tell the patient—and parents—that their use could lead to a small but permanent effect on adult height.1

STRENGTH OF RECOMMENDATIONS

B: Based on one prospective study.

Kelly HW, Sternberg AL, Lescher R, et al; CAMP Research Group. Effect of inhaled glucocorticoids in childhood on adult height. N Engl J Med. 2012;367:904-912.

 

Illustrative case

A 10-year-old boy is brought in by his father for asthma follow-up. The child uses an albuterol inhaler, but has had increased coughing and wheezing recently. You are ready to step up his asthma therapy to include ICS. But the patient’s father questions this, noting that he recently read that steroids may reduce a child’s growth. How should you respond?

Inhaled corticosteroids (ICS) are a mainstay in the treatment of asthma ranging from mild persistent to severe. Standards of care for asthma treatment involve a stepwise approach, with ICS added if symptoms are not controlled with short-acting beta antagonists alone.2 In addition, monotherapy with ICS is more effective for controlling symptoms than leukotriene inhibitors or other controller medications, while also decreasing hospitalizations and nocturnal awakenings and improving quality of life—with few side effects.3

What we know about ICS and children’s growth
One adverse effect of ICS, however, is that of “decreased linear growth velocity”4—ie, slowing the rate at which children grow. Until recently, children were thought to “catch up” later in life, either by growing for a longer period of time than they would had they not taken ICS or by growing at an increased velocity after ICS medications are discontinued.4-6

Study summary: The effect on growth is small, but long-lasting

Kelly et al conducted a prospective observational cohort study that followed 943 (90.7%) participants in the Childhood Asthma Management Program (CAMP) in the years after the randomized controlled trial (RCT) ended.

A double-blind, placebo-controlled RCT, CAMP studied the linear growth of 1041 children with mild-to-moderate persistent asthma who were divided into 3 treatment groups: One group received 200 mcg inhaled budesonide twice daily; a second group received 8 mg inhaled nedocromil twice daily; and a third group received placebo. Albuterol was used symptomatically by all 3 groups.7 The children ranged in age from 5 to 13 years at the start of the study; 98 patients—split evenly among the 3 treatment arms—were lost to follow-up.

During the 4 to 6 years of the CAMP trial, the budesonide group received a mean total of 636 mg ICS, whereas the nedocromil and placebo groups received an average of 88.5 and 109.4 mg ICS, respectively. After the RCT ended, all participants had standardized asthma treatment, receiving mean adjusted total doses of ICS of 381 mg for the budesonide group, 347.9 mg for the nedocromil group, and 355 mg for the placebo group.

Patients’ height was measured every 6 months for the next 4.5 years, and once or twice a year thereafter until they reached adult height (at a mean age of 24.9±2.7 years).

ICS users were a half inch shorter
Long-term ICS use was linked to a lower adult height. The adjusted mean height was 171.1 cm for the budesonide group vs 172.3 cm for those on placebo, a difference of 1.2 cm, or 0.47 inch (95% confidence interval [CI], −1.9 to −0.5; P=.001); the mean adult height in the nedocromil group (172.1 cm) was similar to that of the placebo group (−0.2 cm; 95% CI, −0.9 to 0.5; P=.61).

The lower adult height in the ICS group did not vary significantly based on sex, age at trial entry, race, or duration of asthma prior to trial entry; however, dose was a key factor. A larger daily dose of budesonide—particularly in the first 2 years of the RCT—was associated with a lower adult height (about −0.1 cm for each mcg/kg in that 2-year time frame). This was consistent with results from studies that looked at other types of ICS (beclomethasone, fluticasone, and mometasone).8-11

The study also showed that growth velocity was reduced in the first 2 years of assigned treatment with budesonide, and this was primarily among prepubertal participants. After the initial 2-year slowing in growth rate, the children resumed growing at normal speeds.

 

 

 

What’s new: Now we know: Children don’t “catch up"

Retrospective studies have reported that children on ICS for mild persistent to moderate asthma would have an initial slowing in growth velocity but then “catch up” by growing for a longer period of time.3-5 This is the first prospective study with good follow-up to show that ICS use affects long-term growth and adult height. While the effect is not large, some children and their families might be concerned about it.

Caveats: ICS use was atypical

The randomized controlled portion of the study used a prescribed dose of budesonide without regard to symptoms. This is not the typical pattern of ICS use. In addition, compliance with ICS varies significantly.12 Because the effect on adult height appears to be dose dependent, however, we think the results of this study are valid.

In addition, there was a placebo control group (and big differences in exposure to ICS) only for the duration of the RCT. During the subsequent study, all patients received equivalent doses of ICS. This means that the variation in mean adult height achieved can be primarily ascribed to participants’ use of ICS during the 4- to 6-year CAMP trial. Of note, the effect of ICS was greatest in prepubertal participants, so there may be a diminished effect as teens approach their final height.

The study did not look at the effect of ICS use in patients with severe asthma—the group most likely to use ICS. However, the benefits of ICS for those with severe asthma likely outweigh any negative effects on adult height.

Challenges to implementation: What to tell patients

The message we convey to patients (and parents) about ICS use is a nuanced one. We can stress that ICS remain very important in the treatment of asthma and, while it appears that their use causes a slight decrease in adult height, most children with persistent asthma benefit from ICS.

Acknowledgement

The PURLs Surveillance System is supported in part by Grant Number UL1RR024999 from the National Center for Research Resources, a Clinical Translational Science Award to the University of Chicago. The content is solely the responsibility of the authors and does not necessarily represent the official views of the National Center for Research Resources or the National Institutes of Health.

Practice changer

Before prescribing inhaled corticosteroids (ICS) for a child with asthma, tell the patient—and parents—that their use could lead to a small but permanent effect on adult height.1

STRENGTH OF RECOMMENDATIONS

B: Based on one prospective study.

Kelly HW, Sternberg AL, Lescher R, et al; CAMP Research Group. Effect of inhaled glucocorticoids in childhood on adult height. N Engl J Med. 2012;367:904-912.

 

Illustrative case

A 10-year-old boy is brought in by his father for asthma follow-up. The child uses an albuterol inhaler, but has had increased coughing and wheezing recently. You are ready to step up his asthma therapy to include ICS. But the patient’s father questions this, noting that he recently read that steroids may reduce a child’s growth. How should you respond?

Inhaled corticosteroids (ICS) are a mainstay in the treatment of asthma ranging from mild persistent to severe. Standards of care for asthma treatment involve a stepwise approach, with ICS added if symptoms are not controlled with short-acting beta antagonists alone.2 In addition, monotherapy with ICS is more effective for controlling symptoms than leukotriene inhibitors or other controller medications, while also decreasing hospitalizations and nocturnal awakenings and improving quality of life—with few side effects.3

What we know about ICS and children’s growth
One adverse effect of ICS, however, is that of “decreased linear growth velocity”4—ie, slowing the rate at which children grow. Until recently, children were thought to “catch up” later in life, either by growing for a longer period of time than they would had they not taken ICS or by growing at an increased velocity after ICS medications are discontinued.4-6

Study summary: The effect on growth is small, but long-lasting

Kelly et al conducted a prospective observational cohort study that followed 943 (90.7%) participants in the Childhood Asthma Management Program (CAMP) in the years after the randomized controlled trial (RCT) ended.

A double-blind, placebo-controlled RCT, CAMP studied the linear growth of 1041 children with mild-to-moderate persistent asthma who were divided into 3 treatment groups: One group received 200 mcg inhaled budesonide twice daily; a second group received 8 mg inhaled nedocromil twice daily; and a third group received placebo. Albuterol was used symptomatically by all 3 groups.7 The children ranged in age from 5 to 13 years at the start of the study; 98 patients—split evenly among the 3 treatment arms—were lost to follow-up.

During the 4 to 6 years of the CAMP trial, the budesonide group received a mean total of 636 mg ICS, whereas the nedocromil and placebo groups received an average of 88.5 and 109.4 mg ICS, respectively. After the RCT ended, all participants had standardized asthma treatment, receiving mean adjusted total doses of ICS of 381 mg for the budesonide group, 347.9 mg for the nedocromil group, and 355 mg for the placebo group.

Patients’ height was measured every 6 months for the next 4.5 years, and once or twice a year thereafter until they reached adult height (at a mean age of 24.9±2.7 years).

ICS users were a half inch shorter
Long-term ICS use was linked to a lower adult height. The adjusted mean height was 171.1 cm for the budesonide group vs 172.3 cm for those on placebo, a difference of 1.2 cm, or 0.47 inch (95% confidence interval [CI], −1.9 to −0.5; P=.001); the mean adult height in the nedocromil group (172.1 cm) was similar to that of the placebo group (−0.2 cm; 95% CI, −0.9 to 0.5; P=.61).

The lower adult height in the ICS group did not vary significantly based on sex, age at trial entry, race, or duration of asthma prior to trial entry; however, dose was a key factor. A larger daily dose of budesonide—particularly in the first 2 years of the RCT—was associated with a lower adult height (about −0.1 cm for each mcg/kg in that 2-year time frame). This was consistent with results from studies that looked at other types of ICS (beclomethasone, fluticasone, and mometasone).8-11

The study also showed that growth velocity was reduced in the first 2 years of assigned treatment with budesonide, and this was primarily among prepubertal participants. After the initial 2-year slowing in growth rate, the children resumed growing at normal speeds.

 

 

 

What’s new: Now we know: Children don’t “catch up"

Retrospective studies have reported that children on ICS for mild persistent to moderate asthma would have an initial slowing in growth velocity but then “catch up” by growing for a longer period of time.3-5 This is the first prospective study with good follow-up to show that ICS use affects long-term growth and adult height. While the effect is not large, some children and their families might be concerned about it.

Caveats: ICS use was atypical

The randomized controlled portion of the study used a prescribed dose of budesonide without regard to symptoms. This is not the typical pattern of ICS use. In addition, compliance with ICS varies significantly.12 Because the effect on adult height appears to be dose dependent, however, we think the results of this study are valid.

In addition, there was a placebo control group (and big differences in exposure to ICS) only for the duration of the RCT. During the subsequent study, all patients received equivalent doses of ICS. This means that the variation in mean adult height achieved can be primarily ascribed to participants’ use of ICS during the 4- to 6-year CAMP trial. Of note, the effect of ICS was greatest in prepubertal participants, so there may be a diminished effect as teens approach their final height.

The study did not look at the effect of ICS use in patients with severe asthma—the group most likely to use ICS. However, the benefits of ICS for those with severe asthma likely outweigh any negative effects on adult height.

Challenges to implementation: What to tell patients

The message we convey to patients (and parents) about ICS use is a nuanced one. We can stress that ICS remain very important in the treatment of asthma and, while it appears that their use causes a slight decrease in adult height, most children with persistent asthma benefit from ICS.

Acknowledgement

The PURLs Surveillance System is supported in part by Grant Number UL1RR024999 from the National Center for Research Resources, a Clinical Translational Science Award to the University of Chicago. The content is solely the responsibility of the authors and does not necessarily represent the official views of the National Center for Research Resources or the National Institutes of Health.

References

1. Kelly HW, Sternberg AL, Lescher R, et al; CAMP Research Group. Effect of inhaled glucocorticoids in childhood on adult height. N Engl J Med. 2012;367:904-912.

2. Expert Panel Report 3: Guidelines for the Diagnosis and Management of Asthma. National Institutes of Health National Heart, Lung and Blood Institute: National Asthma Education and Prevention Program, 2007. Available at: http://www.nhlbi. nih.gov/guidelines/asthma/asthgdln.pdf. Accessed August 15, 2013.

3. Chauhan BF, Ducharme FM. Anti-leukotriene agents compared to inhaled corticosteroids in the management of recurrent and/ or chronic asthma in adults and children. Cochrane Database Syst Rev. 2012;(5):CD002314.

4. Agertoft L, Pedersen S. Effect of long- term treatment with budesonide on adult height in children with asthma. N Engl J Med. 2000;343:1064-1069.

5. Van Bever HP, Desager KN, Lijssens N, et al. Does treatment of asthmatic children with inhaled corticosteroids affect their adult height? Pediatr Pulmonol. 1999;27:369-375.

6. Silverstein MD, Yunginger JW, Reed CE, et al. Attained adult height after childhood asthma: effect of glucocorticoid therapy. J Allergy Clin Immunol. 1997;99:466-474.

7. The Childhood Asthma Management Program Research Group. Long-term effects of budesonide or nedocromil in children with asthma. N Engl J Med. 2000;343:1054-1063.

8. Tinkelman DG, Reed CE, Nelson HS, et al. Aerosol beclomethasone dipropionate compared with theophylline as primary treatment of chronic, mild to moderately severe asthma in children. Pediatrics. 1993;92:64-77.

9. Verberne AA, Frost C, Roorda RJ, et al. One year treatment with salmeterol compared with beclomethasone in children with asthma. Am J Respir Crit Care Med. 1997;156:688-695.

10. Allen DB, Bronsky EA, LaForce CF, et al. Growth in asthmatic children treated with fluticasone propionate. J Pediatr 1998;132: 472-477.

11. Skoner DP, Meltzer EO, Milgrom H, et al. Effects of inhaled mometasone furoate on growth velocity and adrenal function: a placebo-controlled trial in children 4-9 years old with mild persistent asthma. J Asthma. 2011;48:848-859.

12. Cochrane MG, Bala MV, Downs KE, et al. Inhaled corticosteroids for asthma therapy: patient compliance, devices, and inhalation technique. Chest. 2000;117:542-550.

References

1. Kelly HW, Sternberg AL, Lescher R, et al; CAMP Research Group. Effect of inhaled glucocorticoids in childhood on adult height. N Engl J Med. 2012;367:904-912.

2. Expert Panel Report 3: Guidelines for the Diagnosis and Management of Asthma. National Institutes of Health National Heart, Lung and Blood Institute: National Asthma Education and Prevention Program, 2007. Available at: http://www.nhlbi. nih.gov/guidelines/asthma/asthgdln.pdf. Accessed August 15, 2013.

3. Chauhan BF, Ducharme FM. Anti-leukotriene agents compared to inhaled corticosteroids in the management of recurrent and/ or chronic asthma in adults and children. Cochrane Database Syst Rev. 2012;(5):CD002314.

4. Agertoft L, Pedersen S. Effect of long- term treatment with budesonide on adult height in children with asthma. N Engl J Med. 2000;343:1064-1069.

5. Van Bever HP, Desager KN, Lijssens N, et al. Does treatment of asthmatic children with inhaled corticosteroids affect their adult height? Pediatr Pulmonol. 1999;27:369-375.

6. Silverstein MD, Yunginger JW, Reed CE, et al. Attained adult height after childhood asthma: effect of glucocorticoid therapy. J Allergy Clin Immunol. 1997;99:466-474.

7. The Childhood Asthma Management Program Research Group. Long-term effects of budesonide or nedocromil in children with asthma. N Engl J Med. 2000;343:1054-1063.

8. Tinkelman DG, Reed CE, Nelson HS, et al. Aerosol beclomethasone dipropionate compared with theophylline as primary treatment of chronic, mild to moderately severe asthma in children. Pediatrics. 1993;92:64-77.

9. Verberne AA, Frost C, Roorda RJ, et al. One year treatment with salmeterol compared with beclomethasone in children with asthma. Am J Respir Crit Care Med. 1997;156:688-695.

10. Allen DB, Bronsky EA, LaForce CF, et al. Growth in asthmatic children treated with fluticasone propionate. J Pediatr 1998;132: 472-477.

11. Skoner DP, Meltzer EO, Milgrom H, et al. Effects of inhaled mometasone furoate on growth velocity and adrenal function: a placebo-controlled trial in children 4-9 years old with mild persistent asthma. J Asthma. 2011;48:848-859.

12. Cochrane MG, Bala MV, Downs KE, et al. Inhaled corticosteroids for asthma therapy: patient compliance, devices, and inhalation technique. Chest. 2000;117:542-550.

Issue
The Journal of Family Practice - 62(9)
Issue
The Journal of Family Practice - 62(9)
Page Number
500-502
Page Number
500-502
Publications
Publications
Topics
Article Type
Display Headline
This asthma treatment has a lasting side effect in children
Display Headline
This asthma treatment has a lasting side effect in children
Legacy Keywords
Tanner Nissly; DO; Shailendra Prasad; MBBS; MPH; asthma; PURLs; inhaled corticosteroids; ICS; Childhood Asthma Management Program; CAMP;
Legacy Keywords
Tanner Nissly; DO; Shailendra Prasad; MBBS; MPH; asthma; PURLs; inhaled corticosteroids; ICS; Childhood Asthma Management Program; CAMP;
Sections
PURLs Copyright

Copyright © 2013 Family Physicians Inquiries Network. All rights reserved.

Disallow All Ads
Alternative CME
Use ProPublica
Hide sidebar & use full width
render the right sidebar.
Conference Recap Checkbox
Not Conference Recap
Clinical Edge
Display the Slideshow in this Article
Article PDF Media
Media Files

Corticosteroids for a Sore Throat?

Article Type
Changed
Tue, 12/13/2016 - 12:08
Display Headline
Corticosteroids for a Sore Throat?
One dose of a steroid can alleviate the pain—and has the potential to decrease unnecessary use of antibiotics.


PRACTICE CHANGER
Consider prescribing a single dose of corticosteroids for patients with sore throat; this has been found to provide quick pain relief and resolution of symptoms.1

STRENGTH OF RECOMMENDATION
A: Based on a meta-analysis of randomized controlled trials (RCTs) in ambulatory care settings.1

ILLUSTRATIVE CASE
A 28-year-old woman comes to your clinic because she’s had a severe sore throat and low-grade fever for the past two days. She has no associated cough. Examination reveals erythematous posterior oropharynx with exudate. A rapid strep test is negative. The patient says the sore throat is very painful and asks for medication to make it better. What should you prescribe?

Most sore throats—particularly in adults—are viral and self-limiting.2,3 Group A β-hemolytic Streptococcus infections account for just 10% of sore throats in adults and 15% to 30% in children.4 Yet US physicians have been found to prescribe antibiotics for more than half of patients who present with sore throat.5-7

Do patients want antibiotics, or simply pain relief?
Antibiotics produce only a modest reduction in symptoms of pharyngitis (fever and throat soreness), presumably in patients with bacterial infections, and increase the risk for adverse events.5,6 Research suggests that patients who request antibiotics for sore throat may be seeking primarily pain relief.8 Thus, a treatment that is more effective in alleviating symptoms of a sore throat would likely contribute to a decrease in unnecessary use of antibiotics.

A short course of corticosteroids has been used successfully and shown to be safe for conditions such as acute sinusitis, croup, and asthma.9-11 Could the anti-inflammatory effects of corticosteroids reduce pain in patients with sore throat, as well? A 2010 systematic review suggested that was the case.12 Cochrane reviewers recently took another look.1

STUDY SUMMARY
Steroids bring speedier pain relief
This meta-analysis included eight RCTs (the same eight trials used in the systematic review9) that compared corticosteroids with placebo for the symptomatic treatment of exudative or severe sore throat.1 Sore throat was defined as clinical evidence of pharyngitis and/or tonsillitis or the clinical syndrome of painful throat and odynophagia.

Five studies were conducted in the United States, and one each in Canada, Turkey, and Israel. Five studies focused on adults (n = 413); the other three studied children (n = 393). Overall, 47% of participants had exudative sore throat, and 44% were positive for group A β-hemolytic Streptococcus.

In all eight RCTs, antibiotics were given to those in both the treatment and placebo groups. In addition, all participants were allowed to use traditional analgesia (either acetaminophen or NSAIDs). Corticosteroids (oral dexamethasone, oral prednisone, or intramuscular [IM] dexamethasone) were used as an adjunctive treatment in all the RCTs.

Primary outcomes varied between studies. Four of the eight RCTs included the proportion of patients with improvement or complete resolution of symptoms within 24 to 48 hours. Mean time to onset of pain relief was the primary outcome in five of the eight studies. Some of the secondary outcomes in the individual trials included relapse rates, adverse events, and days missed from school or work.

Overall, patients who received corticosteroids were three times more likely to report complete resolution of symptoms at 24 hours (relative risk = 3.2) and had a reduced mean time to onset of pain relief of about six hours. The number needed to treat to prevent one patient from experiencing pain at 24 hours was < 4.

Adverse events were reported in only one of the trials (n = 125): Five patients (three in the steroid group and two receiving placebo) were hospitalized for fluid rehydration, and three patients (one in the steroid group and two receiving placebo) developed peritonsillar abscess.12 Three RCTs did not find any significant difference in days missed from school or work, and four trials reported no difference in recurrence of symptoms. One of the trials found that 16% of the patients in the placebo group returned to seek additional care, while none in the steroid group did.13

WHAT’S NEW
Steroids haven’t been tested as standalone treatment

Steroids are not currently recommended for routine use to treat symptoms of sore throat. This Cochrane review found that patients with severe or exudative sore throat benefit from pain reduction with corticosteroids, used as an adjunct to antibiotics and other analgesics without increased risk for harm. Nonetheless, the use of steroids in this patient population would address a practical concern of those seeking symptom relief and has the potential to decrease unnecessary use of antibiotics.

CAVEATS
Questions about effects on antibiotic use, heterogeneity

The studies in this meta-analysis did not assess whether the use of corticosteroids would reduce unnecessary use of antibiotics, so we cannot conclude that this would be the case. Because the effect was similar in all subgroups analyzed, however, it is reasonable to expect that reduced antibiotic use could be a positive effect. The main documented benefit was resolution of pain, an important patient-centered outcome that justifies consideration of treating painful pharyngitis with corticosteroids.

 

 

Corticosteroids have an immunosuppressant effect and carry the theoretical risk for exacerbation of an existing infection. That did not occur in these studies. Nor has it occurred when short courses of corticosteroids are used for other illnesses, such as croup, infectious mononucleosis, asthma, contact dermatitis, and COPD.14 Thus, this theoretical risk is not a barrier to implementation.

It is important to note that single and multiple doses of corticosteroids and oral and IM routes were effective, with only minimal differences in results.

CHALLENGES TO 
IMPLEMENTATION
Determining the severity
Acetaminophen and NSAIDs are used for pain relief in sore throat and have been shown to be effective—but may be inadequate for severe pain.15 There are no head-to-head trials that have compared steroids to NSAIDs or acetaminophen in this clinical scenario. So the challenge for clinicians will be to decide when pharyngitis is severe enough to justify the use of corticosteroids, rather than simple analgesics alone.  

REFERENCES
1. Hayward G, Thompson M, Perera R, et al. Corticosteroids as stand-alone or add-on treatment for sore throat. Cochrane Database Syst Rev. 2012;(10):CDC008268.

2. Cherry DK, Woodwell DA. National Ambulatory Medical Care Survey: 2000 summary. Adv Data. 2002;328:1-32.

3. Bisno AL. Acute pharyngitis. N Engl J Med. 2001;344:205-211.

4. Del Mar CB, Glasziou PP, Sprinks AB. Antibiotics for sore throat. Cochrane Database Syst Rev. 2006;(4):CD000023.

5. Linder JA, Stafford RS. Antibiotic treatment of adults with sore throat by community primary care physicians: a national survey, 1989-1999. JAMA. 2001;286:1181-1186.

6. Linder JA, Bates DW, Lee GM, et al. Antibiotic treatment of children with sore throat. JAMA. 2005;294:2315-2322.

7. Hong SY, Taur Y, Jordan MR. Antimicrobial prescribing in the USA for adult acute pharyngitis in relation to treatment guidelines. J Eval Clin Pract. 2011;17:1176-1183.

8. van Driel ML, De Sutter A, Deveugele M, et al Are sore throat patients who hope for antibiotics actually asking for pain relief? Ann Fam Med. 2006;4:494-499.

9. Venekamp RP, Thompson MJ, Hayward G, et al. Systemic corticosteroids for acute sinusitis. Cochrane Database Syst Rev. 2011;(12): CD008115.

10. Russell KF, Liang Y, O’Gorman K, et al. Glucocorticoids for croup. Cochrane Database Syst Rev. 2011;(1):CD001955.

11. Rowe BH, Spooner C, Ducharme F, et al. Early emergency department treatment of acute asthma with systemic corticosteroids. Cochrane Database Syst Rev. 2001;(1):CD002178.

12. Korb K, Scherer M, Cenot J. Steroids as adjuvant therapy for acute pharyngitis in ambulatory patients: a systematic review. Ann Fam Med. 2010;8:58-63.

13. Olympia RP, Khine H, Avner JR. Effectiveness of oral dexamethasone in the treatment of moderate to severe pharyngitis in children. Arch Pediatr Adolesc Med. 2005;159:278-282.

14. Manson SC, Brown RE, Cerulli A, et al. The cumulative burden of oral corticosteroid side effects and the economic implications of steroid use. Respir Med. 2009;103:975-994.

15. Wei JL, Kasperbauer JL, Weaver AL, et al. Efficacy of single-dose dexamethasone as adjuvant therapy for acute pharyngitis. Laryngoscope. 2002;112:87-93.

Acknowledgement
The PURLs Surveillance System was supported in part by Grant Number UL1RR024999 from the National Center for Research Resources, a Clinical Translational Science Award to the University of Chicago. The content is solely the responsibility of the authors and does not necessarily represent the official views of the National Center for Research Resources or the National Institutes of Health.

Copyright © 2013. The Family Physicians Inquiries Network. All rights reserved.

Reprinted with permission from the Family Physicians Inquiries Network and The Journal of Family Practice. 2013;62(7):372-374.

Author and Disclosure Information

Keri Bergeson, MD, Nina Rogers, MD, Shailendra Prasad, MBBS, MPH

Issue
Clinician Reviews - 23(8)
Publications
Topics
Page Number
18
Legacy Keywords
corticosteroids, sore throat, oropharynx, strep, Streptococcus, pharyngitis, dexamethasone, prednisone
Sections
Author and Disclosure Information

Keri Bergeson, MD, Nina Rogers, MD, Shailendra Prasad, MBBS, MPH

Author and Disclosure Information

Keri Bergeson, MD, Nina Rogers, MD, Shailendra Prasad, MBBS, MPH

One dose of a steroid can alleviate the pain—and has the potential to decrease unnecessary use of antibiotics.

One dose of a steroid can alleviate the pain—and has the potential to decrease unnecessary use of antibiotics.


PRACTICE CHANGER
Consider prescribing a single dose of corticosteroids for patients with sore throat; this has been found to provide quick pain relief and resolution of symptoms.1

STRENGTH OF RECOMMENDATION
A: Based on a meta-analysis of randomized controlled trials (RCTs) in ambulatory care settings.1

ILLUSTRATIVE CASE
A 28-year-old woman comes to your clinic because she’s had a severe sore throat and low-grade fever for the past two days. She has no associated cough. Examination reveals erythematous posterior oropharynx with exudate. A rapid strep test is negative. The patient says the sore throat is very painful and asks for medication to make it better. What should you prescribe?

Most sore throats—particularly in adults—are viral and self-limiting.2,3 Group A β-hemolytic Streptococcus infections account for just 10% of sore throats in adults and 15% to 30% in children.4 Yet US physicians have been found to prescribe antibiotics for more than half of patients who present with sore throat.5-7

Do patients want antibiotics, or simply pain relief?
Antibiotics produce only a modest reduction in symptoms of pharyngitis (fever and throat soreness), presumably in patients with bacterial infections, and increase the risk for adverse events.5,6 Research suggests that patients who request antibiotics for sore throat may be seeking primarily pain relief.8 Thus, a treatment that is more effective in alleviating symptoms of a sore throat would likely contribute to a decrease in unnecessary use of antibiotics.

A short course of corticosteroids has been used successfully and shown to be safe for conditions such as acute sinusitis, croup, and asthma.9-11 Could the anti-inflammatory effects of corticosteroids reduce pain in patients with sore throat, as well? A 2010 systematic review suggested that was the case.12 Cochrane reviewers recently took another look.1

STUDY SUMMARY
Steroids bring speedier pain relief
This meta-analysis included eight RCTs (the same eight trials used in the systematic review9) that compared corticosteroids with placebo for the symptomatic treatment of exudative or severe sore throat.1 Sore throat was defined as clinical evidence of pharyngitis and/or tonsillitis or the clinical syndrome of painful throat and odynophagia.

Five studies were conducted in the United States, and one each in Canada, Turkey, and Israel. Five studies focused on adults (n = 413); the other three studied children (n = 393). Overall, 47% of participants had exudative sore throat, and 44% were positive for group A β-hemolytic Streptococcus.

In all eight RCTs, antibiotics were given to those in both the treatment and placebo groups. In addition, all participants were allowed to use traditional analgesia (either acetaminophen or NSAIDs). Corticosteroids (oral dexamethasone, oral prednisone, or intramuscular [IM] dexamethasone) were used as an adjunctive treatment in all the RCTs.

Primary outcomes varied between studies. Four of the eight RCTs included the proportion of patients with improvement or complete resolution of symptoms within 24 to 48 hours. Mean time to onset of pain relief was the primary outcome in five of the eight studies. Some of the secondary outcomes in the individual trials included relapse rates, adverse events, and days missed from school or work.

Overall, patients who received corticosteroids were three times more likely to report complete resolution of symptoms at 24 hours (relative risk = 3.2) and had a reduced mean time to onset of pain relief of about six hours. The number needed to treat to prevent one patient from experiencing pain at 24 hours was < 4.

Adverse events were reported in only one of the trials (n = 125): Five patients (three in the steroid group and two receiving placebo) were hospitalized for fluid rehydration, and three patients (one in the steroid group and two receiving placebo) developed peritonsillar abscess.12 Three RCTs did not find any significant difference in days missed from school or work, and four trials reported no difference in recurrence of symptoms. One of the trials found that 16% of the patients in the placebo group returned to seek additional care, while none in the steroid group did.13

WHAT’S NEW
Steroids haven’t been tested as standalone treatment

Steroids are not currently recommended for routine use to treat symptoms of sore throat. This Cochrane review found that patients with severe or exudative sore throat benefit from pain reduction with corticosteroids, used as an adjunct to antibiotics and other analgesics without increased risk for harm. Nonetheless, the use of steroids in this patient population would address a practical concern of those seeking symptom relief and has the potential to decrease unnecessary use of antibiotics.

CAVEATS
Questions about effects on antibiotic use, heterogeneity

The studies in this meta-analysis did not assess whether the use of corticosteroids would reduce unnecessary use of antibiotics, so we cannot conclude that this would be the case. Because the effect was similar in all subgroups analyzed, however, it is reasonable to expect that reduced antibiotic use could be a positive effect. The main documented benefit was resolution of pain, an important patient-centered outcome that justifies consideration of treating painful pharyngitis with corticosteroids.

 

 

Corticosteroids have an immunosuppressant effect and carry the theoretical risk for exacerbation of an existing infection. That did not occur in these studies. Nor has it occurred when short courses of corticosteroids are used for other illnesses, such as croup, infectious mononucleosis, asthma, contact dermatitis, and COPD.14 Thus, this theoretical risk is not a barrier to implementation.

It is important to note that single and multiple doses of corticosteroids and oral and IM routes were effective, with only minimal differences in results.

CHALLENGES TO 
IMPLEMENTATION
Determining the severity
Acetaminophen and NSAIDs are used for pain relief in sore throat and have been shown to be effective—but may be inadequate for severe pain.15 There are no head-to-head trials that have compared steroids to NSAIDs or acetaminophen in this clinical scenario. So the challenge for clinicians will be to decide when pharyngitis is severe enough to justify the use of corticosteroids, rather than simple analgesics alone.  

REFERENCES
1. Hayward G, Thompson M, Perera R, et al. Corticosteroids as stand-alone or add-on treatment for sore throat. Cochrane Database Syst Rev. 2012;(10):CDC008268.

2. Cherry DK, Woodwell DA. National Ambulatory Medical Care Survey: 2000 summary. Adv Data. 2002;328:1-32.

3. Bisno AL. Acute pharyngitis. N Engl J Med. 2001;344:205-211.

4. Del Mar CB, Glasziou PP, Sprinks AB. Antibiotics for sore throat. Cochrane Database Syst Rev. 2006;(4):CD000023.

5. Linder JA, Stafford RS. Antibiotic treatment of adults with sore throat by community primary care physicians: a national survey, 1989-1999. JAMA. 2001;286:1181-1186.

6. Linder JA, Bates DW, Lee GM, et al. Antibiotic treatment of children with sore throat. JAMA. 2005;294:2315-2322.

7. Hong SY, Taur Y, Jordan MR. Antimicrobial prescribing in the USA for adult acute pharyngitis in relation to treatment guidelines. J Eval Clin Pract. 2011;17:1176-1183.

8. van Driel ML, De Sutter A, Deveugele M, et al Are sore throat patients who hope for antibiotics actually asking for pain relief? Ann Fam Med. 2006;4:494-499.

9. Venekamp RP, Thompson MJ, Hayward G, et al. Systemic corticosteroids for acute sinusitis. Cochrane Database Syst Rev. 2011;(12): CD008115.

10. Russell KF, Liang Y, O’Gorman K, et al. Glucocorticoids for croup. Cochrane Database Syst Rev. 2011;(1):CD001955.

11. Rowe BH, Spooner C, Ducharme F, et al. Early emergency department treatment of acute asthma with systemic corticosteroids. Cochrane Database Syst Rev. 2001;(1):CD002178.

12. Korb K, Scherer M, Cenot J. Steroids as adjuvant therapy for acute pharyngitis in ambulatory patients: a systematic review. Ann Fam Med. 2010;8:58-63.

13. Olympia RP, Khine H, Avner JR. Effectiveness of oral dexamethasone in the treatment of moderate to severe pharyngitis in children. Arch Pediatr Adolesc Med. 2005;159:278-282.

14. Manson SC, Brown RE, Cerulli A, et al. The cumulative burden of oral corticosteroid side effects and the economic implications of steroid use. Respir Med. 2009;103:975-994.

15. Wei JL, Kasperbauer JL, Weaver AL, et al. Efficacy of single-dose dexamethasone as adjuvant therapy for acute pharyngitis. Laryngoscope. 2002;112:87-93.

Acknowledgement
The PURLs Surveillance System was supported in part by Grant Number UL1RR024999 from the National Center for Research Resources, a Clinical Translational Science Award to the University of Chicago. The content is solely the responsibility of the authors and does not necessarily represent the official views of the National Center for Research Resources or the National Institutes of Health.

Copyright © 2013. The Family Physicians Inquiries Network. All rights reserved.

Reprinted with permission from the Family Physicians Inquiries Network and The Journal of Family Practice. 2013;62(7):372-374.

PRACTICE CHANGER
Consider prescribing a single dose of corticosteroids for patients with sore throat; this has been found to provide quick pain relief and resolution of symptoms.1

STRENGTH OF RECOMMENDATION
A: Based on a meta-analysis of randomized controlled trials (RCTs) in ambulatory care settings.1

ILLUSTRATIVE CASE
A 28-year-old woman comes to your clinic because she’s had a severe sore throat and low-grade fever for the past two days. She has no associated cough. Examination reveals erythematous posterior oropharynx with exudate. A rapid strep test is negative. The patient says the sore throat is very painful and asks for medication to make it better. What should you prescribe?

Most sore throats—particularly in adults—are viral and self-limiting.2,3 Group A β-hemolytic Streptococcus infections account for just 10% of sore throats in adults and 15% to 30% in children.4 Yet US physicians have been found to prescribe antibiotics for more than half of patients who present with sore throat.5-7

Do patients want antibiotics, or simply pain relief?
Antibiotics produce only a modest reduction in symptoms of pharyngitis (fever and throat soreness), presumably in patients with bacterial infections, and increase the risk for adverse events.5,6 Research suggests that patients who request antibiotics for sore throat may be seeking primarily pain relief.8 Thus, a treatment that is more effective in alleviating symptoms of a sore throat would likely contribute to a decrease in unnecessary use of antibiotics.

A short course of corticosteroids has been used successfully and shown to be safe for conditions such as acute sinusitis, croup, and asthma.9-11 Could the anti-inflammatory effects of corticosteroids reduce pain in patients with sore throat, as well? A 2010 systematic review suggested that was the case.12 Cochrane reviewers recently took another look.1

STUDY SUMMARY
Steroids bring speedier pain relief
This meta-analysis included eight RCTs (the same eight trials used in the systematic review9) that compared corticosteroids with placebo for the symptomatic treatment of exudative or severe sore throat.1 Sore throat was defined as clinical evidence of pharyngitis and/or tonsillitis or the clinical syndrome of painful throat and odynophagia.

Five studies were conducted in the United States, and one each in Canada, Turkey, and Israel. Five studies focused on adults (n = 413); the other three studied children (n = 393). Overall, 47% of participants had exudative sore throat, and 44% were positive for group A β-hemolytic Streptococcus.

In all eight RCTs, antibiotics were given to those in both the treatment and placebo groups. In addition, all participants were allowed to use traditional analgesia (either acetaminophen or NSAIDs). Corticosteroids (oral dexamethasone, oral prednisone, or intramuscular [IM] dexamethasone) were used as an adjunctive treatment in all the RCTs.

Primary outcomes varied between studies. Four of the eight RCTs included the proportion of patients with improvement or complete resolution of symptoms within 24 to 48 hours. Mean time to onset of pain relief was the primary outcome in five of the eight studies. Some of the secondary outcomes in the individual trials included relapse rates, adverse events, and days missed from school or work.

Overall, patients who received corticosteroids were three times more likely to report complete resolution of symptoms at 24 hours (relative risk = 3.2) and had a reduced mean time to onset of pain relief of about six hours. The number needed to treat to prevent one patient from experiencing pain at 24 hours was < 4.

Adverse events were reported in only one of the trials (n = 125): Five patients (three in the steroid group and two receiving placebo) were hospitalized for fluid rehydration, and three patients (one in the steroid group and two receiving placebo) developed peritonsillar abscess.12 Three RCTs did not find any significant difference in days missed from school or work, and four trials reported no difference in recurrence of symptoms. One of the trials found that 16% of the patients in the placebo group returned to seek additional care, while none in the steroid group did.13

WHAT’S NEW
Steroids haven’t been tested as standalone treatment

Steroids are not currently recommended for routine use to treat symptoms of sore throat. This Cochrane review found that patients with severe or exudative sore throat benefit from pain reduction with corticosteroids, used as an adjunct to antibiotics and other analgesics without increased risk for harm. Nonetheless, the use of steroids in this patient population would address a practical concern of those seeking symptom relief and has the potential to decrease unnecessary use of antibiotics.

CAVEATS
Questions about effects on antibiotic use, heterogeneity

The studies in this meta-analysis did not assess whether the use of corticosteroids would reduce unnecessary use of antibiotics, so we cannot conclude that this would be the case. Because the effect was similar in all subgroups analyzed, however, it is reasonable to expect that reduced antibiotic use could be a positive effect. The main documented benefit was resolution of pain, an important patient-centered outcome that justifies consideration of treating painful pharyngitis with corticosteroids.

 

 

Corticosteroids have an immunosuppressant effect and carry the theoretical risk for exacerbation of an existing infection. That did not occur in these studies. Nor has it occurred when short courses of corticosteroids are used for other illnesses, such as croup, infectious mononucleosis, asthma, contact dermatitis, and COPD.14 Thus, this theoretical risk is not a barrier to implementation.

It is important to note that single and multiple doses of corticosteroids and oral and IM routes were effective, with only minimal differences in results.

CHALLENGES TO 
IMPLEMENTATION
Determining the severity
Acetaminophen and NSAIDs are used for pain relief in sore throat and have been shown to be effective—but may be inadequate for severe pain.15 There are no head-to-head trials that have compared steroids to NSAIDs or acetaminophen in this clinical scenario. So the challenge for clinicians will be to decide when pharyngitis is severe enough to justify the use of corticosteroids, rather than simple analgesics alone.  

REFERENCES
1. Hayward G, Thompson M, Perera R, et al. Corticosteroids as stand-alone or add-on treatment for sore throat. Cochrane Database Syst Rev. 2012;(10):CDC008268.

2. Cherry DK, Woodwell DA. National Ambulatory Medical Care Survey: 2000 summary. Adv Data. 2002;328:1-32.

3. Bisno AL. Acute pharyngitis. N Engl J Med. 2001;344:205-211.

4. Del Mar CB, Glasziou PP, Sprinks AB. Antibiotics for sore throat. Cochrane Database Syst Rev. 2006;(4):CD000023.

5. Linder JA, Stafford RS. Antibiotic treatment of adults with sore throat by community primary care physicians: a national survey, 1989-1999. JAMA. 2001;286:1181-1186.

6. Linder JA, Bates DW, Lee GM, et al. Antibiotic treatment of children with sore throat. JAMA. 2005;294:2315-2322.

7. Hong SY, Taur Y, Jordan MR. Antimicrobial prescribing in the USA for adult acute pharyngitis in relation to treatment guidelines. J Eval Clin Pract. 2011;17:1176-1183.

8. van Driel ML, De Sutter A, Deveugele M, et al Are sore throat patients who hope for antibiotics actually asking for pain relief? Ann Fam Med. 2006;4:494-499.

9. Venekamp RP, Thompson MJ, Hayward G, et al. Systemic corticosteroids for acute sinusitis. Cochrane Database Syst Rev. 2011;(12): CD008115.

10. Russell KF, Liang Y, O’Gorman K, et al. Glucocorticoids for croup. Cochrane Database Syst Rev. 2011;(1):CD001955.

11. Rowe BH, Spooner C, Ducharme F, et al. Early emergency department treatment of acute asthma with systemic corticosteroids. Cochrane Database Syst Rev. 2001;(1):CD002178.

12. Korb K, Scherer M, Cenot J. Steroids as adjuvant therapy for acute pharyngitis in ambulatory patients: a systematic review. Ann Fam Med. 2010;8:58-63.

13. Olympia RP, Khine H, Avner JR. Effectiveness of oral dexamethasone in the treatment of moderate to severe pharyngitis in children. Arch Pediatr Adolesc Med. 2005;159:278-282.

14. Manson SC, Brown RE, Cerulli A, et al. The cumulative burden of oral corticosteroid side effects and the economic implications of steroid use. Respir Med. 2009;103:975-994.

15. Wei JL, Kasperbauer JL, Weaver AL, et al. Efficacy of single-dose dexamethasone as adjuvant therapy for acute pharyngitis. Laryngoscope. 2002;112:87-93.

Acknowledgement
The PURLs Surveillance System was supported in part by Grant Number UL1RR024999 from the National Center for Research Resources, a Clinical Translational Science Award to the University of Chicago. The content is solely the responsibility of the authors and does not necessarily represent the official views of the National Center for Research Resources or the National Institutes of Health.

Copyright © 2013. The Family Physicians Inquiries Network. All rights reserved.

Reprinted with permission from the Family Physicians Inquiries Network and The Journal of Family Practice. 2013;62(7):372-374.

Issue
Clinician Reviews - 23(8)
Issue
Clinician Reviews - 23(8)
Page Number
18
Page Number
18
Publications
Publications
Topics
Article Type
Display Headline
Corticosteroids for a Sore Throat?
Display Headline
Corticosteroids for a Sore Throat?
Legacy Keywords
corticosteroids, sore throat, oropharynx, strep, Streptococcus, pharyngitis, dexamethasone, prednisone
Legacy Keywords
corticosteroids, sore throat, oropharynx, strep, Streptococcus, pharyngitis, dexamethasone, prednisone
Sections
Article Source

PURLs Copyright

Inside the Article